Download as pdf or txt
Download as pdf or txt
You are on page 1of 120

CHARTERED ACCOUNTANCY PROFESSIONAL II

(CAP-II)

Suggested Answer
June 2019

The Institute of Chartered Accountants of Nepal

Disclaimer: The Suggested answers are prepared by the institute with a view to assist the students in their
study. The suggested answers are indicative and not exhaustive. Students are expected to apply their
knowledge and write the answer in the examinations taking the suggested answers as a guidance.
Paper 1: Advanced Accounting
CAP II Paper 1: Advanced Accounting

Marks
Attempt all questions. Working notes should form part of the answer.
1. The summarized balance sheet of Shyam Limited as on 32nd Ashadh,
2075 is as under:
Particulars Amount (Rs.)
Liabilities
Share Capital
500,000 equity shares of Rs. 10 each fully paid up 5,000,000
9%, 20,000 Preference shares of Rs. 100 each fully paid up 2,000,000
Reserves & Surplus
Profit & Loss Account (1,460,000)
Non-current Liabilities
10 % secured debentures 1,600,000
Current Liabilities
Interest due on debentures 160,000
Trade Payables 500,000
Loan from Directors 100,000
Bank Overdraft 100,000
Provision for Tax 100,000
Total 8,100,000
Assets
Non- Current Assets
Fixed Assets:
a. Tangible Assets
Land & Building 3,000,000
Plant & Machinery 1,250,000
Furniture & Fixtures 250,000
b. Intangible Assets
Goodwill 1,000,000
Patents 500,000
Current Assets
Trade Investments 500,000
Trade Receivables 500,000
Inventory 1,000,000
Discount on issue of Debentures 100,000
Total 8,100,000
Note: Preference dividend is in arrears for last two years.
Mr. Ram holds 60% of debentures and Mr. Shyam holds 40% of
debentures. Moreover, Rs. 100,000 and Rs. 60,000 were also payable to
Mr. Ram and Mr. Shyam respectively as trade payable.

The following scheme of reconstruction has been agreed upon and duly approved.
i) All the equity shares to be converted into fully paid equity shares of
Rs. 5 each.
ii) The preference shares to be reduced to Rs. 50 each and the
preference shareholders agreed to forego their arrears of preference

© The Institute of Chartered Accountants of Nepal 2


CAP II Paper 1: Advanced Accounting

dividends, in consideration of which 9 % preference shares are to be


converted into 10% preference shares.
iii) Mr. Ram and Mr. Shyam agreed to cancel 50 % each of their
respective total debt including interest on debentures. Mr. Ram and
Mr. Shyam also agreed to pay Rs. 100,000 and Rs. 60,000
respectively in cash and to receive new 12% debentures for the
balance amount.
iv) Persons related to trade payables, other than Mr. Ram and Mr.
Shyam also agreed to forego their 50% claims.
v) Directors also waived 60% of their loans and accepted equity shares
for the balance.
vi) Capital commitments of Rs. 300,000 were cancelled on payment of
Rs. 15,000 as penalty.
vii) Directors refunded Rs. 100,000 of the fees previously received by
them.
viii) Reconstruction expenses paid Rs. 15,000.
ix) The taxation liability of the company was settled for Rs. 75,000 and
was paid immediately.
x) The assets were revalued as under:
Land & Building Rs. 3,200,000; Plant & machinery Rs. 600,000;
Inventory Rs. 750,000; Trade Receivables Rs. 400,000; Furniture &
Fixtures Rs. 150,000; Trade Investments Rs. 450,000.
You are required to prepare necessary journal entries for all the above
mentioned transactions including written off of intangible assets. And
also prepare Bank Account and Reconstruction Account. 20
Answer:
Journal Entries in the books of Shyam Limited
Dr. Cr.
i. Equity Share Capital (Rs. 10 each) A/c Dr. 50,00,000
To, Equity Share Capital (Rs. 5 each) A/c 25,00,000
To, Reconstruction A/c 25,00,000
(Being conversion of 5,00,000 equity shares of Rs. 10
each fully paid into same number of fully paid equity
shares of Rs. 5 each as per the scheme of
reconstruction)
ii. 9 % Preference Share Capital (Rs.100 each) A/c Dr. 20,00,000
To, 10 % Preference Share Capital (Rs.50 each) A/c 10,00,000
To, Reconstruction A/c 10,00,000

(Being conversion of 9% preference shares of Rs. 100


each fully paid into same number of fully paid
Preference shares of Rs. 50 each and claims of
preference dividends settled as per the scheme of
reconstruction )
iii. 10% Secured Debentures A/c Dr. 9,60,000
Trade Payables A/c Dr. 1,00,000

© The Institute of Chartered Accountants of Nepal 3


CAP II Paper 1: Advanced Accounting

Interest on Debentures Outstanding A/c Dr. 96,000


Bank A/c Dr. 1,00,000
To, 12% Debentures A/c 6,78,000
To, Reconstruction A/c 5,78,000
(Being Rs. 11,56,000 due to Ram, including Trade
Payables, cancelled and 12% debentures allotted for
the amount after waiving 50% as per scheme of
reconstruction )
iv. 10% Secured Debentures A/c Dr. 6,40,000
Trade Payables A/c Dr. 60,000
Interest on Debentures Outstanding A/c Dr. 64,000
Bank A/c Dr. 60,000
To, 12% Debentures A/c 4,42,000
To, Reconstruction A/c 3,82,000
(Being Rs. 7,64,000 due to Shyam, including Trade
Payables, cancelled and 12% debentures allotted for
the amount after waiving 50% as per scheme of
reconstruction.
v. Trade Payables A/c Dr. 1,70,000
To, Reconstruction A/c 1,70,000
(Being remaining payables sacrificed 50% of their
claim)
vi. Directors Loan A/c Dr. 1,00,000
To, Equity Share Capital A/c 40,000
To, Reconstruction A/c 60,000
(Being Director's loan claim settled by issuing 12,000
equity shares of Rs. 5 each as per scheme of
reconstruction)
vii. Reconstruction A/c Dr. 15,000
To, Bank A/c 15,000
(Being payment made towards penalty of 5% for
cancellation of capital commitments of Rs. 3 lakhs)
viii. Reconstruction A/c Dr. 15,000
To, Bank A/c 15,000
(Being payment made of reconstruction expenses)
ix. Bank A/c Dr. 1,00,000
To, Reconstruction A/c 1,00,000
(Being refund of fees by directors credited to
reconstruction account)
x. Provision for Tax A/c Dr. 1,00,000
To, Bank A/c 75,000
To, Reconstruction A/c 25,000
(Being payment of tax liability in full settlement
against provision for tax)
xi. Land & Building A/c Dr. 2,00,000
To, Reconstruction A/c 2,00,000
(Being appreciation in value of land & building
recorded)
xii. Reconstruction A/c Dr. 50,15,000
To, Goodwill A/c 10,00,000

© The Institute of Chartered Accountants of Nepal 4


CAP II Paper 1: Advanced Accounting

To, Patent A/c 5,00,000


To, Profit & Loss A/c 14,60,000
To, Discount on issue of debenture A/c 1,00,000
To, Plant & Machinery A/c 6,50,000
To, Furniture & Fixture A/c 1,00,000
To, Trade Investment A/c 50,000
To, Inventory A/c 2,50,000
To, Trade Receivable A/c 1,00,000
To, Capital Reserve A/c (Balancing Figure) 7,75,000
(Being writing off of losses and reduction in value of
assets as per scheme of reconstruction , balance of
reconstruction account transferred to Capital Reserve)

Bank Account
Particulars Amount Particulars Amount
To, Reconstruction 1,00,000 By, Balance b/d 1,00,000
(Ram) 60,000 By, Reconstruction A/c- Penalty 15,000
To, Reconstruction 1,00,000 By, Reconstruction A/c- Expenses 15,000
(Shyam) By, Provision for Tax A/c 75,000
To, Reconstruction A/c By, Balance c/d 55,000
(Refund from director)
Total 2,60,000 2,60,000

Reconstruction Account
Particulars Amount Particulars Amount
To, Bank- Penalty 1,00,000 By, Equity Share Capital A/c 25,00,000
To, Bank- Expenses 30,000 By, 9%Preference Share Capital 10,00,000
To, Goodwill A/c 10,00,000 A/c 5,78,000
To, Patent A/c 5,00,000 By, Mr. Ram A/c 3,82,000
To, Profit & Loss A/c 14,60,000 By, Mr. Shyam A/c 1,70,000
To, Discount on issue By, Trade payables A/c 60,000
of debenture A/c 1,00,000 By, Director's Loan A/c 1,00,000
To, Plant & Machinery 6,50,000 By, Bank A/c 25,000
A/c To, Furniture & 1,00,000 By, Provision for Tax A/c 2,00,000
Fixture A/c 50,000 By, Land & Building A/c
To, Trade Investment 2,50,000
A/c 1,00,000
To, Inventory A/c 7,75,000
To, Trade Receivable
A/c
To, Capital Reserve A/c
(Balancing Figure)

Total 50,15,000 50,15,000

© The Institute of Chartered Accountants of Nepal 5


CAP II Paper 1: Advanced Accounting

2.
a) Pokhrel and Paudel were carrying on hosiery business sharing profits
and losses equally. The firm’s Balance Sheet as at 32nd Ashadh 2075
was as follows:
Liabilities Amount (Rs.) Assets Amount (Rs.)
Sundry Creditors 6,000,000 Stock 6,000,000
Bank Overdraft 3,500,000 Machinery 15,000,000
Capital Accounts: Debtors 7,000,000
Pokharel 14,000,000 Joint Life Policy 900,000
Paudel 13,000,000 Leasehold Premises 3,400,000
Profit and Loss Account 2,600,000
Drawing Accounts:
Pokharel 1,000,000
Paudel 600,000
Total 36,500,000 Total 36,500,000
The business was carried on till Poush end 2075. The partners
withdrew in equal amounts half the amount of profits made during
the period of six months after charging depreciation at 10% p.a. on
machinery and after writing off 5% on leasehold premises. In the
half year, sundry creditors were reduced by Rs. 1,000,000 and bank
overdraft by Rs. 1,500,000.

On Poush end 2075, stock was revalued at Rs. 7,500,000 and debtors
at Rs. 6,000,000, the joint life policy had been surrendered for Rs.
900,000 before Poush end 2074 and other items remained the same
as at 32nd Ashadh 2075.

On Poush end 2075, the firm sold the business to Global Apparels
Ltd. by fixing value of goodwill at Rs. 10,000,000 and the rest of the
assets were valued on the basis of the Balance Sheet as at Poush end
2075.
Global Apparels Ltd. paid the purchase consideration in equity
shares of Rs. 10 each.
You are required to prepare: (5+5=10)
i) Balance Sheet of the firm as at Poush end 2075.
ii) Partners’ Capital Account showing the final settlement between
them.

b) Following particulars are extracted from the books of Big Bank Ltd.
As the Head of Finance Department of the bank, you are required to
report to the CEO the level of Capital Adequacy Ratios (Tier I and
Total Capital Ratio) for the third quarter ending on Chaitra 2075. 10
Particulars Amount (Rs. 000’)
Paid up Equity Share Capital 2,409,000
Gross income 6,067,000
Net Interest Income 4,550,000

© The Institute of Chartered Accountants of Nepal 6


CAP II Paper 1: Advanced Accounting

Securities Premium 11,000


Exchange Equalization Reserve 37,000
Statutory General Reserve 1,324,000
Investment in equity arising out of underwriting commitment 58,000
Subordinated term debt 1,050,000
Retained Earning up to previous year 255,000
Current Year cumulative profit (Unaudited) 294,000
Discount Value of 5 year Debenture (337,000)
General Loan Loss Provision 421,000
Debenture redemption reserve 480,000
Investment in equity of institutions having financial interest 78,000
Cumulative Redeemable preference shares 256,000
Irredeemable non-cumulative Preference Shares 263,000
Fictitious Assets 16,000
Other reserve 31,000
Risk Weighted Exposures:
For Credit Risk 54,416,000
For Operational Risk 3,860,000
For Market Risk 368,000
Supervisory Response:
During onsite inspection of the bank, Nepal Rastra Bank was not
satisfied with the quality of risk management and control procedures
and instructed the bank management to adjust the risk weighted
exposures as follows:
(a) Asset liability management practices to effectively manage
market risk of the bank is not satisfactory and an additional risk
weight of 1% of Net Interest Income shall be added to the risk
weight for market risk.
(b) The bank has not adopted sound practice of operational risk
management, therefore, an additional capital charge of 2% of
Gross Income shall be charged for operational risks.
(c) Bank has not made desired level of disclosure requirements; the
total risk weighted exposures of the bank shall be increased up to
3%.
Answer:
a) M/S Pokhrel and Paudel
Balance Sheet
As at Poush end 2075
Liabilities Amount Assets Amount
Rs. Rs.
Sundry Creditors 5,000,000 Stock 7,500,000
Bank Overdraft 2,000,000 Machinery 15,000,000
Capital Accounts: Less: Dep. 750,000 14,250,000

© The Institute of Chartered Accountants of Nepal 7


CAP II Paper 1: Advanced Accounting

Pokharel 12,290,000 23,980,000 Debtors 6,000,000


Paudel 11,690,000 Leasehold Premises 3,400,000
Less:Dep. 170,000 3,230,000
30,980,000 30,980,000

Pokharel = 14,000,000 -1,000,000-1,300,000+1,180,000-590,000 = 12,290,000


Paudel= 13,000,000-600,000-1,300,000 +1,180,000-590,000 = 11,690,000

Partners’ Capital Account


Particulars Pokhrel Paudel Particulars Pokhrel Paudel
To P/L A/C 1,300,000 1,300,000 By Balance b/d 14,000,0 13,000,000
To Drawing 1,000,000 600,000 By P/L A/C 00 1,180,000
To Drawing (Profit) 1,180,00 5,000,000
(Out of Profit) 590,000 590,000 By Realization 0 300,000
To Equity Shares in 16,990,000 16,990,000 A/C 5,000,00
Global Apparel ltd. - By Cash 0
To Cash 300,000 -
20,180,000 19,480,000 20,180,0 19,480,000
00

Working Notes:
Computation of Profit earned:
Balance Sheet as at Poush end 2075
Liabilities Amount Rs. Assets Amount Rs.
Sundry Creditors 5,000,000 Stock 7,500,000
Bank Overdraft 2,000,000 Machinery 15,000,000
Capital Accounts:# Less: Dep. 750,000 14,250,000
Pokharel 11,700,000 Debtors 6,000,000
Paudel 11,100,000 Leasehold Premises 3,400,000
Retained Earnings (BF)* 1,180,000 Less: Dep. 170,000 3,230,000

Total 30,980,000 Total 30,980,000


 Since half the profit was withdrawn, total profit for the six month was Rs. 2,360,000.
Capital Accounts:#
Pokharel = 14,000,000 -1,000,000-1,300,000 = 11,700,000
Paudel= 13,000,000-600,000-1,300,000 = 11,100,000

Realization Account
To Assets (Transfer) By Sundry Creditors 5,000,000
Stock 7,500,000 By Bank Overdraft 2,000,000
Machinery 14,250,000 By Global Apparel Ltd 33,980,000
Debtors 6,000,000 (Purchase Consideration)
Premises 3,230,000
To Capital A/C
(Profit on Realization)
Pokhrel 5,000,000 10,000,000
Paudel5,000,000
40,980,000 40,980,000

© The Institute of Chartered Accountants of Nepal 8


CAP II Paper 1: Advanced Accounting

b) As per the Capital Adequacy Framework issued by Nepal Rastra Bank, the
capital adequacy ratios of the Big Bank Ltd. can be calculated as follows:

Big Bank Ltd.


Capital Adequacy Table
For the 3rd Quarter ending on Chaitra 2075
Amount
Particulars (Rs. in 000’)
Core Capital (Tier I)
Paid up Equity Share Capital 2,409,000
Irredeemable Non-cumulative Preference Shares 263,000
Securities Premium 11,000
Statutory General Reserve 1,324,000
Retained Earning up to previous year 255,000
Current Year profit 294,000
Debenture Redemption Reserve 480,000
Less:
Investment in equity of institutions having financial interest (78,000)
Fictitious Assets (16,000)
Investment in equity arising out of underwriting commitment (58,000)
Total Core Capital 4,884,000
Supplementary Capital (Tier II)
Cumulative and redeemable Preference Share 256,000
Subordinated Term Debt 1,050,000
Less: 20% discount value to 5 year Debenture (337,000)
General Loan Loss Provision 421,000
Exchange Equalization Reserve 37,000
Other Reserve 31,000
Total Supplementary Capital 1,458,000
Total Capital Fund (Tier I + Tier II) 6,342,000
Risk Weighted Exposure
Risk Weighted Exposure for Credit Risk 54,416,000
Risk Weighted Exposure for Operational Risk 3,860,000
Risk Weighted Exposure for Market Risk 368,000
Risk weighted Exposure before Supervisory Adjustment 58,644,000
Add: 3% of the total RWE due to non-compliance to disclosure 1,759,320
2% Capital Charge for operational risk 121,340
1% Capital Charge for NII 45,500
Total Risk Weighted Exposure (After Supervisory Adj. of Pillar II) 60,570,160
Core Capital to Total Risk Weighted Exposure 8.06%
Total Capital to Risk Weighted Exposure 10.47%

© The Institute of Chartered Accountants of Nepal 9


CAP II Paper 1: Advanced Accounting

3.
a. XYZ Limited has given the following information for the
preparation of cash flow statement for the year ended 32nd
Ashadh, 2075.

Particulars Rs. in 000’


Net profit after tax 50,000
Dividend (Including Dividend Tax) 17,070
Provision for Income Tax 10,000
Income Tax paid during the year 8,496
Loss on sale of assets (Net) 80
Book value of the assets sold 370
Depreciation charged to Profit & Loss 40,000
Amortization of government grant 12
Profit on sale of investment 200
Carrying amount of the investment sold 55,530
Interest received on investments 5,012
Interest Expenses of the year 20,000
Interest paid during the year 21,040
Increase in working capital (excluding cash & bank balance) 112,150
Purchase of fixed assets 106,300
Proceeds from calls in arrear 4
Receipt of grant for capital projects 24
Proceeds from long-term borrowing 51,960
Proceeds from short-term borrowing 41,150
Opening Cash & Bank balance 10,006
Closing Cash & Bank Balance 13,976

You are required to prepare the Cash Flow Statement for the year
ended 32nd Ashadh, 2075. 10

b. From the following information as on 32nd Ashadh, 2075 of


Xeta General Insurance Co. Ltd., prepare the Revenue Account,
reserving 50% of the net premiums for unexpired risk and an
additional reserve of Rs. 700,000. 5

© The Institute of Chartered Accountants of Nepal 10


CAP II Paper 1: Advanced Accounting

Particular Amount (Rs.)


Reserve for unexpired risk on 31st Ashadh, 2074 1,500,000
Additional reserve on 31st Ashadh, 2074 300,000
Claims paid 1,920,000
Estimated liability in respect of outstanding claims on 31st
Ashadh, 2074 195,000
Estimated liability in respect of outstanding claims on 32nd
Ashadh, 2075 270,000
Expenses of management (including Rs. 90,000 incurred in
connection with claims) 840,000
Re-Insurance premium paid 225,000
Re- Insurance recoveries 60,000
Premiums 3,360,000
Interest and dividend (gross before TDS) 150,000
Profit of sale of investments 30,000
Commission 50,000

Answer:
a)
XYZ Limited
Cash Flow Statement
For the year ended 32nd Ashadh, 2075
Particulars Rs. in 000’ Rs. In 000’
Cash Flow From Operating Activities
Net Profit before Tax (50,000+10,000) 60,000
Adjustments for:
Depreciation 40,000
Loss on sale of assets (net) 80
Amortization of government grant (12)
Profit on sale of Investments (200)
Interest income on Investments (5,012)
Interest Expenses 20,000
Cash Flow from Operating Activities before changes in
Working Capital 1,14,856
Less: Increase in working capital (excluding cash &
bank balance) (1,12,150)
Add: Proceeds from short-term borrowing 41,150
Cash Flow from Operating Activities before income tax
paid 43,856
Less: Income Tax paid (8,496)
Cash Flow from Operating Activities 35,360

Cash Flow from Investing Activities


Sale of assets (370-80) 290
Sale of investments (55,530+200) 55,730

© The Institute of Chartered Accountants of Nepal 11


CAP II Paper 1: Advanced Accounting

Interest Income on Investments 5,012


Purchase of fixed assets (1,06,300)
Cash Flow from Investing Activities (45,268)

Cash Flow from Financing Activities


Proceeds from calls in arrears 4
Receipts of grant for capital projects 24
Proceeds from long-term borrowing 51,960
Interest paid (21,040)
Dividend (including dividend tax paid) (17,070)
13,878
Cash Flow from Financing Activities
Net increase in Cash & Cash Equivalent 3,970
Cash & Cash Equivalent at the beginning of the period 10,006
Cash & Cash Equivalent at the end of the period 13,976

b) Xeta General Insurance Company Limited


Revenue Account for the year ended 32nd Ashadh, 2075
Particulars Schedule Amount Rs.
Premium earned (Net) 1 26,67,500
Profit on sale of investment 30,000
Others -
Interest and dividend (gross) 1,50,000
Total (A) 28,47,500
Claims incurred (Net) 2 20,25,000
Commission 3 50,000
Operating expenses related to insurance 4 7,50,000
Total (B) 28,25,000
Operating profit from insurance business (A)-(B) 22,500

Schedule-1 Premium earned (net)


Rs
Premium received 33,60,000
Less: Premium on reinsurance ceded (2,25,000)
Net premium 31,35,000
Less: Adjustment for change in Reserve for Unexpired risk (as per W.N.) (4,67,500)
Total premium earned 26,67,500

Schedule-2 Claims incurred (Net)


Rs
Claims paid 19,20,000
Add: Expenses regarding claims 90,000
20,10,000
Less: Re-insurance recoveries (60,000)
19,50,000
Add: Claims outstanding as on 32nd Ashadh, 2075 2,70,000
22,20,000
Less: Claims outstanding as on 31st Ashadh, 2074 (1,95,000)
20,25,000

© The Institute of Chartered Accountants of Nepal 12


CAP II Paper 1: Advanced Accounting

Schedule -3 Commission
Rs
Commission paid 50,000

Schedule -4 Operating expenses related to Insurance Business


Rs
Expenses of management (Rs 8,40,000-Rs 90,000) 7,50,000

Working Note:
Calculation for change in Reserve for Unexpired risk:
Rs.
Reserve for Unexpired Risk as on 32nd Ashadh, 2075 15,67,500
Additional Reserve as on 32nd Ashadh, 2075 7,00,000 22,67,500
Less: Reserve for Unexpired Risk as on 31st Ashadh, 2074 1500,000
Additional Reserve as on 31st Ashadh, 2074 3,00,000 (18,00,000
)
4,67,500

4.
a) The Receipts and Payments account of Dadhikot Club prepared on
32nd Ashadh, 2075 is as follows:
Receipts and Payments Account

Dr. Cr.
Receipts Amount (Rs.) Payments Amount (Rs.)
To Balance b/d 450 By Expenses (including
To Annual income payment for sports
from subscription 4,590 material Rs. 2,700) 6,300
Add: Outstanding of By Loss on sale of
last year received this furniture (cost price
year 180 Rs. 450) 180
4,770 By Balance c/d 90,450
Less: Prepaid of last
year (90) 4,680
To Other fees 1,800
To Donation for
building 90,000
96,930 96,930

Additional information:
The club had balances as on 1/4/2074:
Furniture Rs. 1,800; investment at 5% Rs. 27,000; Sports material
Rs. 6,660;
Balance as on 32.3.2075;
Subscription receivable Rs. 270; Subscription received in advance

© The Institute of Chartered Accountants of Nepal 13


CAP II Paper 1: Advanced Accounting

Rs. 90;
Stock of sports material Rs. 1,800

Do you agree with above receipts and payments account? If not,


prepare correct receipts and payments account and income and
expenditure account for the year ended 32nd Ashadh, 2075 and
balance sheet as on that date. 10

b) Mahesh acquired on 1st Shrawan 2074, a machine under a hire


purchase agreement which provides for five half-yearly installments
of Rs. 6,000 each, the first installment being due on 1st Magh 2074.
Assuming that the applicable rate of interest is 10% per annum,
calculate the cash value of the machine. All working should form
part of the answer. 5

Answer:
a) Corrected Receipts and Payments Account of Dadhikot Club for the year
ended 32nd Ashadh, 2075

Receipts Amount Payments Amount


Rs. Rs. Rs.
To Balance b/d 450 By Expenses (Rs. 3,600
To Subscription Annual 4,590 6,300-Rs 2,700)
income By Sports material 2,700
Less: Receivable as on (270) By Balance c/d(cash 90,720
31.3.2075 in hand and at bank
Add: Advance received 90
for the year 2075-2076
Add: Receivable as on 180
31.3.2074
Less: Advance received a (90) 4,500
on 31.3.2074
To Other fees 1,800
To Donation for building 90,000
To Sale of furniture 270
97,020 97,020

© The Institute of Chartered Accountants of Nepal 14


CAP II Paper 1: Advanced Accounting

Income and Expenditure Account of Dadhikot Club for the year


ended 32nd Ashadh, 2075

Expenditure Amount Income Amount


To Sundry expenses 3,600 By Subscription 4,590
6,660 1,800
To Sports material Balance By Other fees
as on 1.4.2074 2,700 1,350
Add: Purchases (1,800) 7,560 By Interest on investment
Less: Balance as on (5%on Rs 27,000)
180
32.3.2075 By Deficit : Excess of 3,600
To Loss on sale of furniture --------- expenditure over income
11,340 11,340

Balance sheet of Dadhikot Club as on 32nd Ashadh, 2075


Liabilities Amount Assets Amount
(Rs) (Rs)
Capital fund 36,000 Furniture 1,800
Less: Excess of (3,600) 32,400 Less: Sold (450) 1,350
expenditure 5% Investment 27,000
over income Interest accrued on 1,350
investment
Building fund 90,000 Sports material 1,800
Subscription Subscription receivable 270
received in advance 90 Cash in hand and at 90,720
1,22,490 bank 1,22,490
Balance Sheet of Dadhikot Club as on 1st Shrawan , 2074
Liabilities Amount Assets Amount
Rs Rs
Subscription received in advance 90 Furniture 1,800
Capital Fund(balancing figure) 36,000 Investment 27,000
Sports material 6,660
Subscription receivable 180
Cash in hand and at 450
36,090 bank 36,090

© The Institute of Chartered Accountants of Nepal 15


CAP II Paper 1: Advanced Accounting

b)
Installment Interest @5% half Principal Amount
Amount yearly= 5/105=1/21 (In each installment)
5th Installment 6,000 286 5714
Less: Interest (286)
5,714
Add: 4th Installment 6,000
11,714
Less: Interest (558) 558 5442
11,156 (11,156-5,714)
Add: 3rd Installment 6,000
17,156 817 5183
Less: Interest (817) (16,339-11,156)
16,339
Add: 2nd Installment 6,000
22,339 1,063 4,937
Less: Interest (1,063) (21,276-16,339)
21,276
Add: 1st Installment 6,000
27,276 1,299 4701
Less: Interest 1,299 (25,977-21,276)
25,977 4023 25,977

The cash price of machinery is Rs. 25,977.

5.
a) Kathmandu Ltd. is a company that prepares accounts in accordance
with Nepal Financial Reporting Standards (NFRS). A meeting of the
Directors of Kathmandu Ltd. is scheduled to discuss the following
matters with a view to finalizing the accounts for the year ending
32nd Ashadh 2075:
i) A fire occurred in one of the warehouses of Kathmandu Ltd. on
3rd Shrawan 2075, destroying inventory which had a cost price of
Rs. 100,000 and a net realisable value of Rs. 150,000.
ii) On 9th Shrawan 2075, Kathmandu Ltd. received information that
one of their largest customers had gone bankrupt. At 32nd Ashadh
2075, this customer owed Kathmandu Ltd. Rs. 235,000. It is
anticipated that Kathmandu Ltd. can only receive 10 paisa for
every Rs. 1 they were owed.
iii) In Shrawan 2075, Kathmandu Ltd. sold inventory which had
been in one of their warehouses for the past two years, for Rs.
75,000. This had been included in the financial statements, for
the year ended 32nd Ashadh 2075, at its cost price of Rs. 105,000.

iv) On 30th Ashadh 2075, an employee of Kathmandu Ltd. fell and


injured her arm at work. This employee has commenced legal action.
The solicitors for Kathmandu Ltd. informed the company on 10th

© The Institute of Chartered Accountants of Nepal 16


CAP II Paper 1: Advanced Accounting

Ashwin 2075, that it is probable they will be found liable and have to
pay this employee Rs. 33,000.
Required: Advise the board on the accounting treatment of these
issues. 5

b) Alex Ltd. intends to set up a solar plant. Alex Ltd. has acquired a
dilapidated factory, having an area of 7,500 acres at a cost of Rs.
70,000 per acre. Alex Ltd. has incurred Rs. 5,000,000 on
demolishing the old factory building thereon. A sum of Rs.
4,357,500 (including 13% VAT) was realized from sale of material
salvaged from the site. Alex Ltd. also incurred Registration Charges
of 5% of Land Value, paid legal and consultancy charges Rs.
500,000 for land acquisition and incurred Rs. 200,000 on Title
Guarantee Insurance. Compute the value of land acquired to be
booked in books of the company. 5

c) On 1st Shrawan 2071, Dillibazar Furniture sold some furniture to his


regular customer M/S Zebra for Rs. 400,000 with three years
interest-free credit. Dillibazar Furniture’s cost of capital is 8%. You
are required to advise how much revenue should be recognized in
fiscal year ending on Ashadh 2072, 2073 and 2074 in accordance
with NAS 18? 5

Answer:
a)
i) This is a non-adjusting event as it occurred after the statement of financial
position date. If it is material it should be disclosed in the financial
statements, but it should not be recognized in the financial statements.
ii) This is an adjusting event as it provides evidence of conditions that existed
at the statement of financial position date. The company should recognize
this in the accounts by debiting bad debts Rs. 211,500 (90/100*235,000)
and crediting trade receivables Rs. 211,500.
iii) This is also an adjusting event as it confirms the net realizable value of the
goods that were in stock at 32nd Ashadh 2075. The company will have to
recognize this by reducing the value of closing inventory in the financial
statements.
iv) This is an adjusting event. As the injury took place prior to the year end it
has now been confirmed that the company will probably have to pay out
Rs. 33,000 in compensation. The company will have to recognize a
provision of Rs. 33,000 in the financial statements by debiting provision
expense Rs. 33,000 and crediting provision statement of financial position
Rs. 33,000.

b) Computation of value of land acquired as Nepal Accounting


Standard:-

© The Institute of Chartered Accountants of Nepal 17


CAP II Paper 1: Advanced Accounting

Property, Plant and Equipment

Particular Rs in lakhs
Purchase price @ Rs 70,000 per acre for 7,500 acres 5,250.00
Stamp duty & registration charges @5% 262.50
Legal fees 5.00
Title guarantee insurance 2.00
Demolition
expenses
50 11.44
Less: Sale of salvaged materials (net of VAT)
(43,57,500× 100/113) = (38.56)
5,530.94
Value of land

c) On Ashadh 2072, sales revenue will be Rs. 317,532.90 i.e.


(400,000×1/ . Also, Dillibazar Furniture will recognize interest
income of Rs. 25,402.63 i.e. 317,532.90 ×8%.

In 2073 Ashadh, interest income will be Rs. 27,434.84 i.e.


(317,532.90+25,402.63)×8%.

In 2074 Ashadh, remaining amount out of Rs. 400,000 i.e. Rs. 29,629.63 will be
recognized as interest income.

6. Write short notes on: (5×3=15)


a) Financial Leverage Multiplier (FLM)
b) Matching Concept
c) Fund Based Accounting Technique
d) Public Expenditure and Financial Accountability (PEFA)
e) Average Clause

Answer:
a) Financial Leverage Multiplier (FLM)
Financial Leverage Multiplier (FLM) is the connection between return on assets
and return on equity. The FLM is one of the several ways of looking at the
relative amounts of debt and equity the organization is using to finance its
assets. An important feature of FLM is the relationship:
ROA*FLM=ROE
That implies that if ROE is important to investors in an organization, then the
relative level of ROE can be managed by changes in the FLM once ROA results
can be anticipated.
FLM= Total Assets/ Equity
Alternatively
FLM= (Debt+Equity)/Equity

b) Matching Concept
The matching concept is an essential part of accrual accounting, these two are
often used interchangeably. Like accrual concept, the matching concepts also

© The Institute of Chartered Accountants of Nepal 18


CAP II Paper 1: Advanced Accounting

result from periodicity concept. The matching concept requires that the
expenses for an accounting period should be matched against related incomes.
The matching concept states that the revenue and the expenses incurred to
earned the revenues must belong to the same accounting period. So once the
revenue is realized, the next step is to allocate it to the relevant accounting
period.
Significance
 It guides how the expenses should be matched with revenue for determining
exact profit or loss for a particular period.
 It is very helpful for the investors/ shareholders to know the exact amount of
profit or loss of the business.
 It requires proper allocation of costs into appropriate period so that relevant
income and expenses are matched.

c) Fund based accounting technique


Fund based accounting involves preparation of financial statements fund-wise.
In case of institutions like school, college, university etc. separate ledgers are
maintained for each fund. Fund may be created for purchase, acquisition,
construction of fixed assets or for any specific activities of the organization. For
example, a building fund may be created with a view to construct building. All
receipts in connection with the construction of the building are separated from
the main transaction of the organization and shown under building fund.
Expenditures in construction of the building will be made out of this fund.
When building is completed, building will be shown as an asset and
consequently that portion of the building fund which has been utilized for the
construction of the building should be transferred to general fund.

d) Public Expenditure and Financial Accountability (PEFA)


PEFA is the framework for improving public expenditure system of a country.
Since public funds hold utmost importance, PEFA pays high priority in
transparency and accountability in managing the fund. PEFA initiatives has
developed a robust tool for measuring public financial management (PFM)
performances and providing sound assessment of the quality of PFM for
countries of all income level. Nepal has assessed PEFA indicators and adopted
action plan that serve as the national policy for the overall improvement in PFM
system, processes and institutions. Effective implementation of action plan
contribute to improve PFM performances and eventually help to achieve better
service delivery and bringing efficiency in public expenditure management.

e) Average Clause
Some unscrupulous businessmen may resort to under- insurance of stocks in
order to save some amount of premium. Under-insurance means insuring for a
lesser value. Under- insurance is resorted to because, usually the loss will not be
total and therefore, in spite of under- insurance the business men can recover his
loss. For example, stocks worth Rs. 1,00,000 may be insured for, say, Rs.
60,000, because the insured knows, from experience, that in the event of fire not
all his stocks are likely to be lost. (Of course there can be exceptions) So, if
there is a fire and the actual loss is Rs. 50,000, the insured can recover the

© The Institute of Chartered Accountants of Nepal 19


CAP II Paper 1: Advanced Accounting

amount in the absence of an 'average clause'. To prevent such misuse of


insurance, the policy incorporates an 'average clause'.

By inserting average clause, the insured is called upon to bear a portion of loss
himself in the event of under-insurance. The main object of this clause is to
discourage under- insurance, to encourage full-insurance and above all to
impress upon the property owner the necessity of having his property valued
accurately before insurance. Under this clause the loss is suffered by both
insurer and insured proportionately. This is based on the principle that, in case
of under-insurance, the owner of the property himself acts as an insurer to the
extent the property has not been insured with the insurance company.
For example,
A building of Rs. 60,000 is insured for Rs. 50,000 then to the extent of Rs.
10,000, the owner himself is acting as insurer. Insurance company will bear
only Rs. 50,000 and the owner of the building will bear Rs. 10,000. If the loss is
less than Rs. 60,000, then the share of the insurance company is reduced
proportionately. The formula, therefore may be laid down as follows:
Loss to be borne by the insurance company=

© The Institute of Chartered Accountants of Nepal 20


CAP II Paper 1: Advanced Accounting

Specific Comments on the performance of the students


Batch: - June 2019
Level: - CAP-II
Subject: Advanced Accounting
Question No. 1
Some of the students are confused while calculating issue of new debentures. Many
failed to answer at reconstruction amount and debentures for dues settlement of Ram
& Shyam. Students have less knowledge of journal entries.
Question No. 2
Most of the candidates failed to calculate profit for the six month purchase
consideration which led to mistakes in preparing balance sheet and capital account.
Most of the students fail to answer the first part. Students were unable to calculate
profit during the period. Most of the students have problem in calculation of purchase
consideration and profit during the period.
Question No. 3
Classification of items in operating, financing and investing activities is a problem for
students. None of the students has solved the problem of insurance profit as there is
absence of conceptual knowledge.
(a) Most of the students fairly attempted this part. However, majority of candidates
were unable to treat short term borrowing as operating activity.
(b) Most of them failed to provide proper solution. Students are confused about
revenue account of the insurance company.
Question No. 4
(a) Most of the candidates attempted fairly except in calculation of subscription.
(b) Most of the candidates fairly attempted this part.
Question No. 5
The performance is ok. Exclusion of VAT in salvage value is ignored, but should be
accounted as a part of cost as well.
(a) Majority are not up to the mark regarding treatment in adjusting and adjusting
event after balance sheet.
(b) Most of them failed to calculate salvage value.
(c) Most of the case, they are unable to understand the question.
Question No. 6
Majority of the candidates prepared poorly in theoretical part. Students have very
poor knowledge of financial leverage multiplier. For other, notes are satisfactory.

© The Institute of Chartered Accountants of Nepal 21


Paper 2: Audit and Assurance
CAP II Paper 2: Audit and Assurance

Marks
Attempt all questions.

1. As an auditor, give your opinion with explanations on the following cases: (45=20)
a) Surendra Clothing Pvt. Ltd. has been assessed to Income-tax, in which a demand of
Rs. 10 lakhs has been made. The company has gone in appeal. The company has
deposited Rs. 6 lakhs against the demand, on being pursued by the department. The
company has been advised by its counsel that there is 80% chance of losing in respect
of one of the grounds which may end up confirming the demand of rest Rs. 4 lakhs.
How the company should treat the same while preparing the final accounts for the
year ending Asadh end, 2076?
b) At the year-end (2075-03-31), Chitwan Biscuits P. Ltd. revealed an inventory of Rs.
3.5 crores at its godown. Due to a fire on 2075-04-01, inventory worth Rs. 2 crores
were destroyed. The salvage value and insurance claim were estimated at Rs. 1.25
crores before the commencement of audit. No provision was made in the books of
company for the year ended 2075-03-31 for Rs. 0.75 crore.

c) During the previous year ABC Limited has followed the straight line method of
depreciation. During the current year it has been changed to written down value
method.

d) The ABC Ltd., while valuing its finished inventory at the year-end wants to include
interest on Bank Overdraft as an element of cost, for the reason that overdraft has
been taken specifically for the purpose of financing current assets like inventory and
for meeting day to day working expenses.
Answer:
a) As per paragraph 14 of NAS 37, an entity shall recognise a provision if and only
if:
 a present obligation (legal or constructive) has arisen as a result of past event (the
obligating event),
 payment is probable („more likely than not‟)
 the amount can be estimated reliably
Here, the obligating event is an event that creates a legal or constructive obligation and
therefore results in an entity having no realistic alternative but to settle the obligation.
Contingent liability is a possible obligation depending on whether some uncertain future
event occurs or a present obligation but payment is not probable or the amount cannot be
measured reliably. Contingent liability is not recognised; rather it is disclosed in the
notes to accounts in financial statements.

In the given case, there is an assessment by income tax authority of Rs. 10 lakhs. This
situation has arisen as a result of past event resulting in present obligation of the entity
and there is an 80% chance of losing the case. This extent of probability is substantial.
The amount of obligation can also be estimated reliably. Therefore the company shall
make provision for Rs. 10 lakhs. If the provision of Rs. 6 lakhs already deposited has

© The Institute of Chartered Accountants of Nepal 2


CAP II Paper 2: Audit and Assurance

been accounted for as provision, additional provision of Rs. 4 lakhs should be made and
presented in financial statements accordingly.

b) NSA 560 "Subsequent events" requires that the auditor should consider the effects of
subsequent events on the financial statements and the auditor's report. However, the exact
manner and treatment would depend upon whether the event falls in the category of
adjusting event or non-adjusting event as mentioned in NAS 10 Events after Reporting
Date. The event took place after the close of accounting year and does not relate to
conditions existing at the Balance Sheet Date. Thus, it is a non-adjusting event after the
reporting period as per NAS 10. Therefore, an entity shall not adjust the amounts in
Financial Statements for this case. However, an entity shall disclose for each material
category of non-adjusting events the nature of event and its financial effect.
In this case, as the company has correctly accounted by not providing provision, the
auditor is required to ensure whether the proper disclosure of the event has been made in
financial statements.

c) As per NAS 8 “Accounting Policies, Changes in Accounting Estimates and Errors”


defines accounting policy and a change in accounting estimates.
An entity shall change the accounting policy only if the change:
 is required by Standards; or
 results in the financial statements providing reliable and more relevant information
about the effects of transactions, other events or conditions on the entity‟s financial
position, financial performance or cash flows.
A change in accounting estimate is an adjustment of carrying amount of an asset or
liability, or an amount of the periodic consumption of assets, which results from the
assessment of present status of, and expected future benefits and obligations associated
with, assets and liabilities.
As per NAS 16 Property, Plant and Equipment para 5, the residual value and the useful life
of an assets shall be reviewed at least at each financial year-end, if expectation differs from
previous estimates, the changes shall be accounted for as a change in an accounting
estimate in accordance with NAS 8. Para 60 and 61 further describe that depreciation
method used shall reflect the pattern in which the asset‟s future economic benefits are
expected to be consumed by the entity and shall be reviewed at least at each financial year-
end, if there has been a significant change in the expected pattern of consumption of the
future economic benefits embodied in the assets, the method shall be changed to reflect the
changed pattern. Such a change shall be accounted for as a change in accounting estimate
in accordance with NAS 8.
In light of provisions laid down in NASs as discussed above, the change in depreciation
method by the entity is a change in accounting estimate, not the changes in accounting

© The Institute of Chartered Accountants of Nepal 3


CAP II Paper 2: Audit and Assurance

policy. Therefore, as per NAS 8, effect of such changes in accounting estimate shall be
recognised prospectively by including it in profit or loss in the period of the change or in
the period of change and future periods, if such change affects both. The entity shall
disclose the nature and amount of such change in an accounting estimate that has an effect
in current period or is expected to have an effect in future periods.
d) As per NAS 2 “Inventories”, cost of inventories comprises all costs of purchase, costs of
conversion and other costs incurred in bringing the inventories to their present location
and condition. NAS 23 borrowing costs identifies circumstance where borrowing costs
can be included in the cost of inventories. Such borrowing cost shall be directly
attributable to the inventories and would have been avoided if the expenditure on
inventory had not been made. In light of these provisions, in given case, overdraft was
taken for financing current assets and meeting day to day working expenses, not
necessarily directly for inventory only. Therefore, the proposal of ABC Ltd. to include
interest on bank overdraft as an element of cost of inventory is not acceptable because
it does not form part of cost of production.

2. Give your comments on the following cases: (45=20)


a) Binaya Bhandari & Co. is appointed as statutory auditor of Sagarmatha
Development Bank Ltd. by an Annual General Meeting. As a partner at
Binaya Bhandari & Co., describe the process Binaya Bhandari & Co. should
undertake to assess whether the preconditions for an audit are present when
accepting the audit of Sagarmatha Development Bank Ltd.

b) Mr. Kumar, a practicing Chartered Accountant was ordered to surrender his


Certificate of Practice and he was suspended for one year on certain
professional misconduct against him. During the period of suspension, Mr.
Kumar, designating himself as Tax Consultant, did the work of filing of tax
returns and made appearance as a consultant before various related authorities.
He contended that there is nothing wrong in it as he, like any other tax
consultant, could take such work and his engagement as such in no way
violates the order of suspension inflicted on him.
c) A Ltd. has the total Assets of Rs. 1.2 Arab up to Chaitra end 2075 for financial
year 2075/76. It has been estimated that its total assets would be Rs.1.5 Arab
for FY 2075/76. A Ltd. has appointed Ramesh & Associates, the B class audit
firm for the audit of financial year 2075/76, during its AGM held on 2nd
Baishakh 2076.

d) H Ltd. declared dividend amounting to Rs. 3 lakhs out of profits for the year ended
2074/75. Subsequently, it was noticed that company had failed to make provisions for
outstanding expenses of Rs. 4.2 lakhs and the stock was also overvalued, which was
not reported by auditors of the company. Management of H Ltd. held auditors
responsible for this situation.
Answer:
a) NSA 210 prescribes the procedures to establish whether the pre-conditions for an audit
are present in order to accept or continue an audit engagement. Following steps have to be
followed by partner of Binaya Bhandari & Co. to ascertain the same:
(a) Determine whether the financial reporting framework to be applied in the preparation
of the financial statements is acceptable;

© The Institute of Chartered Accountants of Nepal 4


CAP II Paper 2: Audit and Assurance

(b) Obtain the agreement of management that it acknowledges and understands its
responsibility:
(i) for the preparation of the financial statements in accordance with the applicable
financial reporting framework, including where relevant their fair presentation;
(ii) for such internal control as management determines is necessary to enable the
preparation of financial statements that are free from material misstatement,
whether due to fraud or error; and
(iii) to provide Binaya Bhandari & Co. with:
a. Access to all information of which management is aware that is relevant to the
preparation of the financial statements such as records, documentation and
other matters;
b. Additional information that Binaya Bhandari & Co. may request from
management for the purpose of the audit; and
c. Unrestricted access to persons within the entity from whom Binaya Bhandari
& Co determines it necessary to obtain audit evidence

b) A chartered accountant not holding certificate of practice cannot take up any other work
in the capacity of Chartered Accountant in practice because it would amount to
violation of the relevant provisions of the Nepal Chartered Accountants Act, 2053 and
Code of Ethics. In case a member is suspended and is not holding Certificate of
Practice, he cannot in any other capacity to practice as a member of the Institute. This is
because once a person becomes a member of the Institute; he is bound by the provisions
of the Act and its Regulations.
In the instant case, Mr. Kumar was a practicing chartered accountant and he was ordered
to surrender his certificate of practice and was suspended for one year. Mr. Kumar is
doing the work of filing tax returns and has appeared as a consultant before various
related authorities as tax Consultant which is not in capacity of a practicing chartered
accountant rather in capacity of authorized representative. Any person who has been
authorized to act as a tax practitioner on behalf of the concerned registered person can
become authorized representative. Thus, filing tax return and appearing as tax consultant
by Mr. Kumar is not professional misconduct under Nepal Chartered Accountants Act,
2053 and Regulation therein and the code of ethics. Therefore, Mr. Kumar will not be
held guilty for misconduct.

c) As per rule 53 of ICAN Rule 2061 (amended in 2075), the B class registered auditor is
allowed to carry out the audit of company having total assets or liabilities upto Rs.
1Arab. In the light of such changes in ICAN Rule, since A Ltd. has actual total Assets
of Rs. 1.2 arab during the financial year 2075/76 till Chaitra end 2075 and total
estimated assets would be Rs. 1.5 Arab for FY 2075/76, the appointment of Ramesh &
Associates, the B class audit firm for the audit of financial year 2075/76 is invalid.

d) Failure to detect incorrect financial position of a company: In the given case, profit of
the company has been inflated by non-provisioning for expenses of Rs. 4.2 lakhs and by
overvaluation of stock and based on such inflated profit, the company has declared and
paid dividend of Rs. 3 Lakhs. Thus, it can be said that dividend has not been paid out of

© The Institute of Chartered Accountants of Nepal 5


CAP II Paper 2: Audit and Assurance

real profit. If there is insufficient profit after adjustment of outstanding expenses and
correctness of stock valuation and there is no past reserve, it would amount to payment
of dividend out of capital.
It was the duty of auditor to ascertain whether the Balance sheet and Statement of Profit
and Loss of the company show a true and fair view of the financial position and its
performance. For that, he has to exercise proper audit procedure of substantive test and
evaluation of various items of Balance sheet and Statement of profit and loss. The auditor
should have checked whether all the outstanding expenses have been provided or not and
whether closing stock has been properly valued as per NAS 2. If he was not satisfied, he
should have issued a qualified report or adverse report.
In the instant case, the auditor has failed to do so; he will be guilty of gross negligence in
the performance of his duty.

3. Answer the following: (35=15)


a) Mr. Ashish Basnet is a partner at Basnet & Shrestha Associates which is the
external auditor of PQR Ltd., public company. During the audit, he identified
a regulatory non-compliance. He is considering reporting it to the audit
committee. Suggest him in the light of NSA 250, what should be taken into
consideration for reporting the same to the audit committee.
b) What are the factors auditors need to consider while evaluating the adequacy
of the auditor‟s expert‟s work?
c) Explain what is meant by “Written Representations” and indicate to what extent an
auditor can place reliance on such representations.
Answer:
a) Mr. Basnet should follow the procedures prescribed by NSA 250 Consideration of Laws
and Regulations in the Audit of Financial Statements.
If Mr. Basnet becomes aware of information concerning an instance of non-compliance or
suspected non-compliance with laws and regulations, he shall obtain:
a. An understanding of the nature of the act and the circumstances in which it has
occurred; and
b. Further information to evaluate the possible effect on the financial statements.

Following points should be considered for reporting identified non-compliance to audit


committee:-
 Mr. Basnet should evaluate the implications of non-compliance in relation to other
aspects of the audit, including the auditor‟s risk assessment and the reliability of
written representations, and take appropriate action.
 Unless all the members of audit committee are aware of such non-compliance, Mr.
Basnet should communicate non-compliance with laws and regulation that come to
his attention during the course of audit.

© The Institute of Chartered Accountants of Nepal 6


CAP II Paper 2: Audit and Assurance

 If in Mr. Basnet‟s judgment, the non-compliance is believed to be intentional and


material, he should communicate the matter to the audit committee as soon as
practicable.
 If the Mr. Basnet suspects that members of the audit committee are involved in non-
compliance, he should communicate the non-compliance to other members of the
Board of Directors. Where he suspects that everyone in position of governance
including Board of Directors are involved or the communication may not be acted
upon, he should consider the need to obtain legal advice.

b) As per NSA 620, the auditor should evaluate the adequacy of auditor‟s expert‟s work as
audit evidence regarding the assertions being considered. This will involve evaluation
of whether the substance of the expert‟s finding is properly reflected in the financial
statements or supports the assertions, and consideration of :
 The relevance and reasonableness of expert‟s findings and conclusions and their
consistency with other audit evidence,
 If expert‟s work involves use of significant assumptions and methods, the relevance
and reasonableness of those assumptions and methods in the circumstances,
 Source data used,
 Results of expert‟s work in the light of the auditors‟ overall knowledge of the business
and of the result of other audit procedures.
While considering whether the expert has used source data which is appropriate in the
circumstances, the auditor consider the following procedure
 Making inquiries regarding any procedures undertaken by the expert to establish
whether the source data is relevant and reliable.
 Reviewing or testing the data used by the expert.

The auditor need to obtain an understanding of the assumptions and methods used and to
consider whether they are appropriate and reasonable based on the auditors knowledge of
the business and the results of other audit procedures. If the result of the expert‟s work
does not provide sufficient appropriate audit evidence or if the results are not consistent
with other audit evidence the auditor should resolve the matter by applying additional
audit procedures including possibly engaging another expert or modifying the auditor‟s
report.

c) Written Representation: The management is responsible for the preparation and


presentation of financial statements. Thus it is quite natural that during the course of
audit, management would be required to make several representations on various matters
relating to financial statements. The representation by management constitutes
acknowledgement by the management about its responsibility for the preparation and
approval of the financial information. A written representation may either take the form
of a letter from the management or letter by auditor outlining auditor‟s understanding
and confirmation of the same by the management.

© The Institute of Chartered Accountants of Nepal 7


CAP II Paper 2: Audit and Assurance

Extent of Reliance: Written representations are necessary information that auditor


requires in connection with the audit of the entity‟s financial statements. Accordingly, this
is similar to responses to inquiries, hence, written representation are audit evidence.
Although they provide necessary audit evidence, they do not provide sufficient appropriate
audit evidence on their own about any of the matter with which they deal. Furthermore, the
fact that management has provided reliable written representation does not affect the
nature or extent of other audit evidence that the auditor obtains about the fulfillment of
management‟s responsibilities, or about specific assertions.
Therefore, the auditor should:
(a) seek corroborative evidence from sources inside or outside the entity,
(b) evaluate whether the representations made by the management appear reasonable and
consistent with other audit evidence obtained, including other representations; and
(c) consider whether the individuals making the representations are expected to be well
informed on the matter.

It must be noted that representations by the management cannot be the substitute for other
audit evidence that the auditor could reasonably expected to be available. For example, a
representation by the management as to existence, quantity and cost of inventories is not
substitute for adopting audit procedures regarding verification and valuation of
inventories. If a representation by management is contradicted by other evidence, the
auditor should examine the circumstances and, when necessary, reconsider the reliability
of other representations made by the management as well.

4. Answer/Comment on the following: (35=15)


a) Dayahang & Co. is being considered as external auditor for audit of Nischal
Pvt. Ltd. Dayahang quoted an audit fee of Rs. 3 Lakhs plus 5% of profit.
b) Write down the provision for "Marketing Professional Services" for
professional accountant in public practice.
c) A Chartered Accountancy Firm has known that before issuing audit report of certain
client "the independence" has breached. How the audit firm shall ensure the
significance of such breach?
Answer:
a) In the given case, Dayahang & Co. is quoting a fee which is based on percentage of
profit. Contingent fees may create a self-interest threat to objectivity. The existence
and significance of such threats will depend on factors such as the nature of the
engagement, range of possible fee amounts, basis for determining the fee, whether
the outcome or result of the transaction is to be reviewed by an independent third
party etc.
Code of ethics requires that professional accountant in public practice shall not
quote contingent fees for any professional services.
Also as per Section 34 of Nepal Chartered Accountants‟ Act, Members holding
Certificate of Practice shall not base their remuneration as a percentage on the profit
or on any other uncertain results. Hence, quoting of contingent fee is not allowed by
code of ethics and Nepal Chartered Accountant‟s Act. If the Dayahang & Co. does
so they will be liable for professional misconduct.

© The Institute of Chartered Accountants of Nepal 8


CAP II Paper 2: Audit and Assurance

b) Section 250 of ICAN Code of Ethics has stated the provision regarding Marketing
Professional Services". Such provisions are:
When a professional accountant in public practice solicits new work through
advertising or other forms of marketing, there may be a threat to compliance with the
fundamental principles. For example, a self-interest threat to compliance with the
principle of professional behavior is created if services, achievements, or products are
marketed in a way that is inconsistent with that principle.

A professional accountant in public practice shall not bring the profession into
disrepute when marketing professional services. The professional accountant in public
practice shall be honest and truthful, and not:
(a) Make exaggerated claims for services offered, qualifications possessed, or
experience gained; or
(b) Make disparaging references or unsubstantiated comparisons to the work of
another.

If the professional accountant in public practice is in doubt about whether a proposed


form of advertising or marketing is appropriate, the professional accountant in public
practice shall consider consulting with the relevant professional body.
Notwithstanding anything mentioned hereinbefore, no professional accountant in
public practice shall solicit clients through any manner such as advertisement,
designing web site etc. except in accordance with the Guidelines on Ethical Marketing
& Publicity Practices By Professional Accountants issued by the ICAN.

c) Section 290 (42) of ICAN Code of Ethics deals with the matter. Accordingly, the
audit firm shall ensure the significance breach of independence considering the
following factors:
 The nature and duration of the breach;
 The number and nature of any previous breaches with respect to the current audit
engagement;
 Whether a member of the audit team had knowledge of the interest or relationship
that caused the breach;
 Whether the individual who caused the breach is a member of the audit team or
another individual for whom there are independence requirements;
 If the breach relates to a member of the audit team, the role of that individual;
 If the breach was caused by the provision of a professional service, the impact of
that service, if any, on the accounting records or the amounts recorded in the
financial statements on which the firm will express an opinion; and
 The extent of the self-interest, advocacy, intimidation or other threats created by
the breach.

5. Answer/Comment on the following: (25=10)


a) Explain the disqualifications of auditors as per Section 64 of Banks and
Financial Institutions Act, 2073.

© The Institute of Chartered Accountants of Nepal 9


CAP II Paper 2: Audit and Assurance

b) RGS & Co. a firm of Chartered Accountants has three partners, namely, R, G & S.
The firm is allotted the audit of BY Ltd. Mr. R, partner in the firm subsequently holds
100 shares in BY Ltd. Comment.
Answer:
a) Disqualifications for appointment as auditor:
Following persons or any firm or company in which such person is a promoter or partner
shall not be eligible to be appointed as an auditor of a licensed institution and shall in de
facto cease to hold office of auditor even though such person is already appointed:
i) A promoter, director, chief executive officer of bank or financial institution or his or
her family member;
ii) An officer, employee, or internal auditor of the bank or financial institution;
iii) A person working as a partner of any director or employee or chief executive officer
of the bank of financial institution;
iv) A borrower, substantial shareholder or associated person or any person with
financial interest of the bank or financial institution;
v) A person who has been punished in an offense relating to audit, and a period of five
years has not elapsed after he or she has served the punishment;
vi) A person who is insolvent in Nepal or foreign country;
vii) A person, firm, company or institution having subscribed one percent or more of the
shares in the concerned bank of financial institution;
viii) A person who has been punished by the court for a criminal offense involving moral
turpitude, and a period of five years has not elapsed after he or she has served the
punishment; and
ix) A person who is disqualified to be appointed as an auditor as per relevant laws.

b) As per section 112(1)(g) of the Companies act, a substantial shareholder of the company
or a shareholder holding one percent or more of the paid up capital of the company or
his close relative is disqualified from being an auditor.
In the given case, Mr. R is a partner of RGS & Co, Chartered Accountants and RGS &
Co. has been allotted the auditor of the BY Ltd. Mr. R partner of the firm has 100 shares
of By Ltd. As per the company act, a substantial shareholder of the company or his
close relative is disqualified from being an auditor. Mr. R holds only 100 shares which
is not substantial shareholder of the company. So, RGS & Co. can be auditor of the
company.

6. Write short notes on the following: (42.5=10)


a) Limited Assurance Engagement
b) Contents of Permanent Audit File
c) External Confirmation
d) Audit Strategy

© The Institute of Chartered Accountants of Nepal 10


CAP II Paper 2: Audit and Assurance

Answer:
a) Limited Assurance Engagement
A limited assurance engagement is an assurance engagement in which the practitioner
reduces engagement risk to a level that is acceptable in the circumstances of the
engagement but in which the risk is greater than for a reasonable assurance engagement.
For a limited assurance engagement, the team must understand the subject matter
sufficiently to identify areas where a significant deviation is most likely to arise.
Further, obtaining an understanding of internal control relevant to the engagement is
usually not required. The evidence needed in a limited assurance engagement would
normally be limited to that obtained by inquiry, analytical procedures, and discussion,
to enable the practitioner to conclude that the subject matter is plausible in the
circumstances. In contrast to reasonable assurance engagements, the practitioner in a
limited assurance engagement would not normally seek to corroborate evidence
obtained as long as the information obtained from carrying out the audit procedures
appears plausible in the circumstances to the practitioner. The conclusion for a limited
assurance engagement is in the negative form, i.e. “based on the procedures performed
and evidence obtained, nothing has come to our attention (…).”
b) Contents of Permanent Audit File
Contents of Permanent Audit Files:
 Information concerning the legal and organizational structure of the entity. In the case
of a company, this includes the Memorandum and Articles of Association. In the case
of a statutory corporation, this includes the Act and Regulations under which the
corporation functions.
 Extracts or copies of important legal documents, agreements and minutes relevant to
the audit.
 A record of the study and evaluation of the internal controls related to the accounting
system. This might be in the form of narrative descriptions, questionnaires or flow
charts, or some combination thereof.
 Copies of audited financial statements for previous years.
 Analysis of significant ratios and trends.
 Copies of management letters issued by the auditor, if any.
 Record of communication with the retiring auditor, if any, before acceptance of the
appointment as auditor.
 Notes regarding significant accounting policies.
 Significant audit observations of earlier years.

c) External confirmation
An external confirmation represents audit evidence obtained by the auditor as a direct
written response to the auditor from a third party (the confirming party), in paper form, or
by electronic or other medium. External confirmation procedures frequently are relevant
when addressing assertions associated with certain account balances and their elements.
However, external confirmations need not be restricted to account balances only. For
example, the auditor may request confirmation of the terms of agreements or transactions
an entity has with third parties; the confirmation request may be designed to ask if any
modifications have been made to the agreement and, if so, what the relevant details are.

© The Institute of Chartered Accountants of Nepal 11


CAP II Paper 2: Audit and Assurance

External confirmation procedures also are used to obtain audit evidence about the absence
of certain conditions, for example, the absence of a “side agreement” that may influence
revenue recognition.

d) Audit Strategy
Audit strategy is concerned with designing optimized audit approaches that seek to
achieve the necessary audit assurances at the lowest cost within the constraints of the
information available.
Audit procedures should be relevant to the important assertions, and as cost effective as
possible to perform. Audit strategy generally involves the following steps:
i) Obtaining knowledge of business.
ii) Performing analytical procedures at initial stages.
iii) Evaluating inherent risks.
iv) Evaluating internal control system for strategy purpose.
v) Formulating the strategy – identifying risk areas, development of appropriate audit
strategy for those areas.

7. Distinguish between: (25=10)


a) Audit and Investigation
b) Test Check & Internal Check
Answer:
a) Audit and Investigation:
Basis Audit Investigation

Description Auditing is the independent Investigation means an inquiry,


examination of financial information of or is the act of detail
any entity, whether profit oriented or examination of activities so as
not, and irrespective of its size or legal to achieve certain objectives.
form, when such an examination is
conducted with a view to expressing an
opinion thereon

Owners Audit is conducted for accountability Investigation may be conducted


purposes on behalf of owners who either by owner of the
make the appointment. undertaking or by an outsider.

Purpose To determine the true and fair view. Varies from business to
business

Process Routine process Investigation is not a regular


process

Scope It includes only an examination of the It covers an examination of the


accounts of a business accounts bur also covers an
inquiry into other matter that
are connected with the purpose
for which it is undertaken

© The Institute of Chartered Accountants of Nepal 12


CAP II Paper 2: Audit and Assurance

Employees Does not examine personally May examine personally

Person Audit is to be conducted by a chartered Investigation may be take on


performing accountant or person recognized by even by a non-chartered
work regulatory body. accountant

Legal Audit is mandatory under law There is no such legal


Obligations obligations with regard to
investigation

b) Distinction between Test Check & Internal Check


Sn. Particulars Test Check Internal check
1 Meaning It stands for the method of It refers to a system of
auditing when instead of a book-keeping and
complete examination of arrangement of staff
all the transaction duties in the organization
recorded in the books of in such a manner that no
account only some of the one person can
transaction are selected completely carry through a
and verified. transaction and record
every aspect thereof.
2 Instituted It is an audit procedures It is a series of procedures
by performed by the auditor laid down by the
in respect of only selected management.
group of transactions.
3 Objectives The purpose is to aid Its objective is to facilitate
auditors to check and management functions.
draw conclusions about
the voluminous
transactions.
4 Fraud & It helps the auditor to It is instituted to prevent
Errors unearth frauds and errors frauds and errors.
without checking all the
transactions.
5 Management Management has no Internal check are
Control control over the test subject to review,
checks carried out by the appraisal and changed by
auditors. the management.

© The Institute of Chartered Accountants of Nepal 13


CAP II Paper 2: Audit and Assurance

Specific Comments on the performance of the students


Batch: - June 2019
Level: - CAP-II
Subject: Audit and Assurance
Question No. 1
Some of the students were confused with events occurring after balance sheet date.
Some of the students mentioned it as accounting policy and opted for retrospective
effect. Provision of NAS 2 was not explained properly. Few students were confused
on retrospective or prospective effect of depreciation. Majorities of answers were
unbalanced, few has written provision of NAS 2 or NAS 23.
Question No. 2
Instead of precondition, terms of engagement is explained. Limit relating to
assets/liabilities were tagged with capital in few cases. Most of the students explained
inherent limitations of audit. Few students are confused on auditing and tax
consultancy services. Majorities were unaware for audit threshold for B/C/D class
auditors. Majorities were unaware of auditors‟ duty to qualify the report.
Question No. 3
Most of the students did not answer three parts of this question. They have not
answered as per the requirement of the question. Majorities were unable to write the
proper consideration. Majorities were unable to test for reliance on written
representation (under 580).
Question No. 4
Although most of the students attempted all three parts, they lacked the proper
understanding of the requirement of laws. Consideration to reporting to audit
committee not explained. Assessment of significance of breach was not explained.
Question No. 5
Most of the students though answered both part, majority of them did not write the
content of Section 64 of BAFIA. Few students are confused 1% or more than 1%
holding.
Question No. 6
Some tagged it to negative opinion. Few are confused with contents of permanent
audit file.
Question No. 7
Concept of test check and internal check was not clear in majority of the cases.

© The Institute of Chartered Accountants of Nepal 14


Paper 3: Corporate and Other Laws
CAP II Paper 3: Corporate and Other Laws

Marks
Attempt all questions.
1. Answer the following questions:
a) Group of 75 students doing MBA from Kathmandu University are eager to establish
a startup business in IT sector. But they are not well aware about the provision of
the Companies Act regarding the number of shareholders required for establishing a
company. So they approached you for advice regarding minimum and maximum
number of shareholders required for registration of a company. Give your advice as
an expert quoting the relevant provisions of the Companies Act, 2063.
b. Mr. CP Adhikari, director of ABC Hospitality Limited has signed on the prospectus
of the company for public issue. State the liability of the Mr. Adhikari for the
matters mentioned in that prospectus by referring the provision of the Companies
Act, 2063. 5
c. Surya Nepal Ltd. decided to call its AGM within 15 days for the winding up of
company. A shareholder retaining 1% of its share objected and challenged the
validity of the AGM though the boards of directors hold 95% shares of the
company. Answer the following matters in the given issue:
i) The required notice for convening AGM. 4
ii) If the AGM becomes valid? 1
d. A resolution was passed by the general meeting of Civil Powers Pvt. Ltd. amending
some clauses of its memorandum and articles. The company by next day gave
information of the amendment to the office of the company Registrar. Some of its
shareholders who were dissenting to the resolution intend to challenge the
resolution. Advise them in the following matters as per the Companies Act, 2063:
i) When the amendment takes place effectiveness? 2
ii) Can the amendment be challenged? 3
e. Board of Directors of the Koshi Company Ltd. is in view that the company is over
capitalized and advisable to consider to buy back its own share. Give your opinion
to the company's Board of Directors regarding the provisions of the Companies Act,
2063 about buy back its own shares by the company. 5

Answer:
a. Where persons are interested to carry out certain business under the name of a
company it is mandatory to register them. Incorporation of a company means
establishment or registration of the company or a corporate body as a legal person
or legal entity. According to Sec 5(5) of the Companies Act, 2063 a person cannot
use the name of company to carry any kind of transaction by the name of any
institution or firm. Thus registration of a company is compulsory to carry any
business under its name.

The process regarding registration of a company has been prescribed in Sections 3, 4


and 5 of the Companies Act. The first amendment made in 2074.1.19 allows
incorporation of a company via electronic transmission with digital recording.

© The Institute of Chartered Accountants of Nepal 2


CAP II Paper 3: Corporate and Other Laws

Any person or persons desiring to incorporate a company which may be either


private, public or company not distributing profit under chapter IX can incorporate
them (Sec 3).

Here in the given situation the group of students doing MBA can carry business in IT
sector by incorporating a company.

So far the required minimum number or maximum number of shareholders are


concern, it depends upon the nature of the company that they are going to establish a
company. Section 9 of the Companies Act, 2063 prescribes the provision regarding
the number of shareholders as follows:

(1) The number of shareholders of a private company shall not be more than one
hundred one.
(2) Subject to the proviso to Sub-section (2) of section 3, the number of shareholders
of a public company shall be seven in minimum and a maximum of any number.
(3) Notwithstanding anything contained in Sub-section (1), any employee who has
purchased a share of a company under scheme of selling shares to employees or any
employee who has already purchased a share under such scheme but is not in service
of the company for the time being shall not be counted as a shareholder.

Hence, as per the provision of the Section 9 of the Act, the 75 students intending to
carry business in IT sector can incorporate either as a private company or as a public
company as per their interest and scope of the business.

b) Section 24 of the Companies Act, 2063 clearly prescribes the provision regarding the
liability for matters contained in prospectus published thereof.

According to Section 24(1) it will be the duty of the concern company to be abide by
the matters contained in prospectus as per the Act. Regarding liability for the matters
contained in the prospectus, it has been stated in Section 24(2) and (3) of the Act.
As per the Section 24(2) of the Companies Act, 2063; the directors who have signed
the prospectus shall be liable for the matters mentioned in that prospectus.

Also as per sub-section (3), if any published prospectus contains false statements
made maliciously or deliberately and any person sustains any loss or damage by
reason of his/her subscription of securities on the faith of that prospectus, the
directors who have signed that prospectus shall be personally liable to pay
compensation for the actual loss or damage so sustained.
Provided, however, that a promoter who resigns before the decision made by the
company to publish the prospectus or whom on becoming aware of any false
statement in the prospectus, publishes a notice of that matter to the information of the
general public prior to the sale or allotment of securities or who proves that he/she did
not know that the prospectus contained any false statement shall not be liable to bear
such compensation.

© The Institute of Chartered Accountants of Nepal 3


CAP II Paper 3: Corporate and Other Laws

Conclusion: Thus in the given issue, Mr. CP Adhikari, director of ABC Hospitality
Limited, who has signed on the prospectus of the company for public issue will
personally liable for the matters contained in the prospectus as per Section 24 (2) and
(3) of the Companies Act, 2063 and also under the Section 33 of the Securities Act,
2063.

c. i. Before convening annual general meeting (AGM) of the company, it is required to


send notice regarding the meeting of the company to all its shareholders. This
provision has been addressed by the Section 67(2) of the Companies Act, 2063.
According to Section 67(2) of the Act, a public company should send a notice
specifying the place, date and agenda of meeting to every shareholders at the address
supplied by that shareholders to the company in advance of at least twenty one days
to hold the annual general meeting, and in advance of at least fifteen days to hold
extra-ordinary general. A notice thereof should also be published at least twice in a
national daily newspaper. Thus the objection made by the shareholders retaining 1%
of its shares prevails.

ii. Regarding the validity of the meeting as per the case, the meeting has been clearly
specified as AGM, where there is clear requirement of 21 days notice in advance,
failure to comply the notice period renders the meeting becomes invalid. The short
period of notice provided in the given case defeats the provision of the law and
becomes ultra vires, hence the majority rule is ineffective.

d. i. Section 21 of the Companies Act, 2063 provides that the general meeting of a
company may amend the memorandum of association or articles of association, by
adopting a special resolution to that effect. As per Section 21(2) after passing such
resolution, the company should give information of any amendment made to the
memorandum of association or articles of association to the Office within thirty days;
and the amendment shall take effect when the Office shall record the same and give
information thereof to the concerned company, within seven days after the receipt of
such information.
As per the provision of the Section 21(2) the amendment made thereof will take place
effectiveness when the Office will record the same and give information thereof to
the concerned company.

ii. As per Sub-section (4) of Section 21, if a shareholder or shareholders of the company
holding at least 5% percent shares of the paid-up capital, except the shareholders who
consent to or vote for the amendment or alteration of a public company who is not
satisfied with an amendment made to the objectives of the company may file a
petition, setting out the reasons therefore, in the court to have that amendment declared
null and void. Provided that the petition has to be filed within 21 days after the
adoption of the resolution to amend the objectives of the company.
Further, where a petition is filed in the court, the amendment made to the objectives of
the company shall not be effective pending the final decision or order by the court in
that matter.

© The Institute of Chartered Accountants of Nepal 4


CAP II Paper 3: Corporate and Other Laws

e. Section 61(1) of Company Act 2063 prohibits company to Buy-back its own shares or
provide loan against the guarantee of own shares.
However subsection (2) of Section 61 of the Companies Act 2063 permits company to
buy back own shares by notifying the Company Registrar Office from the accumulated
profit that could be distributed as dividend on following conditions.
i) Where issued capital of the company is fully paid up.
ii) Where share of public company is listed in Securities Board.
iii) Where Buy-back is authorized by the Company’s Articles.
iv) Where A special resolution has been passed in general meeting
of the company authorizing buy-back.
v) Where the ratio of debt owed by the company is not more than
double of the capital and general reserve fund after such buy back
vi) Where the amount of buy-back is not more than 20% of
company’s total paid up share capital and general reserve fund.
vii) Such buy back is not in contravention with the directives of
Company Registrar Office regarding buy back of share.

Conclusion: In the given case, it is advisable to the company regarding the buyback
of its share that if the company is in a position to comply all the required conditions
as prescribed by the Section 61 (2) and possible to pass its special resolution by
convening the AGM/EGM as per the case, can arrange to buy back its own shares
by the company.

2. Answer the following questions: (3×5=15)


a) ICFC Bank of India is eager to establish a branch in Nepal. Could the
bank establish a branch in Nepal? How it is possible? Answer the
questions, relying on the Banks and Financial Institution Act, 2073.
b) State the acts which cannot be carried out by NRB under the Nepal
Rastra Bank Act, 2058.
c) Peoples Bank, China under its strategic plan decided to incorporate and
operate banking business in Nepal in joint venture with Investment
Bank and some Nepali citizens. The representatives of the Peoples
Bank carried out detail feasibility study and asked you for the
preparation of documents. Advise them about the documents to be
submitted for the prior approval to incorporate the banks and financial
institutions in Nepal with foreign investment.
Answer:
a) Section 40 of the Bank and Financial Institution Act, 2073 has provided special
provision on Branch of Foreign Bank. The ICFC bank may carry on banking and
financial transaction as per the provision given below.

(1) In case the Rastra Bank has granted license to an internationally classified
foreign bank to carry on banking and financial transactions through a branch
in Nepal, the branch so established in Nepal as per the prevailing law shall be
deemed as equivalent to the bank or financial institution established under
this Act and unless otherwise provided by this Act or Rules, Byelaws, order
or directives framed under this Act, all other provisions shall be applicable
equally even in case of such a branch.

© The Institute of Chartered Accountants of Nepal 5


CAP II Paper 3: Corporate and Other Laws

(2) The Rastra Bank may issue necessary directives with regard responsibility
of the officials and employees taking responsibilities of the functions, duties,
powers, responsibility, liability, assets, accounts, etc. of the branch of the
foreign bank and carrying out such functions.
(3) The branch of a foreign bank located in Nepal shall, while using its own
assets in the course of bearing own responsibility, give first priority to its
liability towards Nepal subject not to make adverse effect in any other legal
provisions.
(4) Notwithstanding anything contained elsewhere in this Act, branch of the
foreign bank may carry on wholesale banking transactions.
(5) Function and activities of branch office of foreign bank shall be as prescribed
by the Rastra Bank.

b) Section 7 (1) of the NRB Act, 2058 the Nepal Rastra Bank shall not carry out the
following functions:
a. Providing any loan, accepting any type of deposit or making any type of
financial gift;
b. Purchasing shares of any commercial bank, financial institution, public
corporation or a company or acquiring any type of proprietary right in any
financial, commercial, agricultural, industrial or other institution;
c. Carrying out any type of business; and
d. Acquiring right over movable and immovable property by way of purchase,
lease or in any manner whatsoever. Provided that the Bank may acquire such
property as required for carrying out its function or for achieving its
objectives.
However, under Section 7 (2), NRB can provide loan to its employees and
other companies or institutions which carry out functions helping to attain its
objectives.

c) Section 5 of the Bank and Financial Institution Act, 2073 has provided a
mandatory provision that no bank and financial institutions shall be
incorporated with foreign investment without obtaining a prior approval from
the Nepal Rastra Bank. Subsection (1) states that to incorporate a bank and
financial institution in Nepal by a foreign bank and financial institution in joint
venture with a body corporate established in Nepal or Nepali citizens or to
incorporate a bank and financial institution as a subsidiary company the
following documents other than stated under Section 4(1) of the Act shall be
submitted-
a. Copy of Memorandum and Articles of Association and certificate of
incorporation of the foreign bank and financial institution and the capital
structure.
b. Copy of business license obtained by the bank and financial institution to
carry on banking and financial transaction in the concerned country and
the details of the main place of business.
c. Certified copy of audited balance sheet and profit and loss account of last
three years of the foreign bank and financial institution.
d. Details of business plan, business strategy and nature of business, internal
controls and risk management.

© The Institute of Chartered Accountants of Nepal 6


CAP II Paper 3: Corporate and Other Laws

e. Decision of the foreign bank and financial institution to incorporate bank


and financial institution in Nepal and approval granted by the concerned
regulatory body of the said country.

3. Answer the following questions: (2×5=10)


a) Kastamandap Insurance Ltd. has been undertaking insurance business
since 2068 B.S. Office of the Auditor General audited its account and
has published a report showing that the liability of the Insurer exceeds
its assets within Nepal. Board cancelled the registration of license
fulfilling other formalities. State the validity of the cancellation
grounds on the basis of the Insurance Act, 2049.
b) The committee formed under the convenorship of the member of
National Planning Commission for recommending a name for the
purpose of appointment of Chairperson of the Securities Board of
Nepal, has recommended Mr. Kumar Dhital along with other two
names. Mr. Dhital is a Ph.D. holder in management and does not have
any experience in such field. Comment on the recommendation of the
committee relying upon the relevant provision of the Securities Act,
2063.
Answer:
a) Section 13 of the Insurance Act, 2049 has provided the circumstances which
fulfill the cancellation of the registration of an Insurer with providing a written
notice with effect from the date prescribing in the same notice:
(a) If the Insurance Business is not started within six months from the date of
obtaining the certificate,
(b) If it is felt that the liability of the Insurer exceeds its assets within Nepal,
(c) If the Insurer could not fulfill the liability pursuant to the decision within
three months from the date of final decision of the court in the case filed
under the Insurance Policy issued within Nepal,
(d) If the head office of the Insurance Business of any foreign Insurer is
situated outside Nepal and in case it is felt that Nepalese Insurer has not
obtained equal facilities there which are enjoyed by the foreign Insurer
pursuant to the prevailing law of such country,
(e) If the Insurer does not open its office inside Nepal.

If the Insurer does not perform the functions to be performed or has performed
any functions which is not to be performed pursuant to this Act or the Rules
made under this Act.

The Board should furnish a reasonable time-limit to submit clarification to the


Insurer stating the reason for cancelling its registration. If, the insurer does not
submit or submit clarification is found not to be satisfactory, the Board may
cancel the registration of such Insurer with publishing a notice in two major
newspapers to be published Nepal for the information public in general.
However, mere cancellation of the registration of an insurer, rights and liabilities
of the Insurer shall not make any effect.

© The Institute of Chartered Accountants of Nepal 7


CAP II Paper 3: Corporate and Other Laws

b) As per section 10 of the Securities Act, 2063; in order to be appointed as a


Chairperson or a member, as the case may be, a person shall have to possess
the qualification as follows:-
(a) One who is a citizen of Nepal,
(b) One who has maintained high moral character,
(c) One who has gained at least seven years of professional experience in the
field of stock exchange management, capital market development,
economics, finance, commerce, management or law, and
(d) One who is not disqualified under Section 11 of the said Act.

Since Mr. Dhital doesn’t possess any professional experience in the field of
stock exchange management, capital market development, economics,
finance, commerce, management or law as required by section 10 above, the
recommendation of the committee is not valid.

4. Answer the following questions: (2×5=10)


a) What are the deductions allowed from the remuneration to be obtained
by the workers or employees as per the Labour Act, 2074?
b) Explain the various sanctions prescribed by the Industrial Enterprises
Act, 2073 for the non-compliance of the Act.
Answer:
a) As per section 38 of Labor Act, 2074, the remuneration of workers or employees
shall not be deducted except under the following circumstances:
(a) In case it is required to deduct against absence,
(b) In case it is required to deduct against loss or damage of cash or kind of the
Enterprise caused intentionally or negligently;
(c) In case it is required to deduct in respect of contribution to provident fund or
contribution amount in respect of insurance or such other contribution
amount in respect of social security,
(d) In case it is required to deduct any amount as per the order of judicial or
quasi-judicial body,
(e) In case it is required deduct in respect of providing prescribed facilities;
(f) In case it is required to deduct in respect of advance or over payment of
remuneration;
(g) In case it is required to deduct in respect of income tax or any other tax levied
under prevailing laws;
(h) In case it is required to deduct any amount from remuneration as per
collective agreement;
(i) In case it is required to deduct trade union membership fee. The limit of
amount to be deducted, the method of deduction, the period of deduction and
other related matters shall be as prescribed.

b) Chapter VIII Section 42 of the Industrial Enterprises Act, 2073 has prescribed the
list of the offences and sanctions imposed upon the industries for non-compliance
of the legal provisions.
The Act has defined various non-compliances as offences and has also imposed
sanctions. Offences that include;

© The Institute of Chartered Accountants of Nepal 8


CAP II Paper 3: Corporate and Other Laws

(a) operation of industry without registration,


(b) failure to meet the reporting/filing requirement as set out under the Act,
(c) misuse of facilities and incentives as made available under the Act,
(d) non-fulfillment of the CSR Requirement, etc.
The sanctions under the Act include:
(a) closure of industry, and
(b) fine on the basis of types of industry. For example, a large scale industry is
liable to fine ranging from NPR 100,000 to NPR 300,000 for operating
without registration. Similarly, an industry is liable to fine amounting to
0.75% of the annual transactions for non-compliance with CSR requirement.
The sanctions are levied either on, (a) industry, or (b) responsible officer of
the industry. The time limitation for filling an appeal against the imposition
of sanctions is thirty-five (35) days. Such appeal can be filed at the concerned
High Court.

5. What do you mean by standing committee of the Institute of Chartered


Accountants of Nepal? Mention the formation of Disciplinary and
Executive Committee as per the Nepal Chartered Accountants Act, 2053. 10
Answer:
Standing Committee of ICAN means the formulation of which has been
mentioned by Nepal Chartered Accountants Act, 2053.
As per Section 13 of ICAN Act, 2053; The Council may form the following
permanent committees which shall be responsible to it:

(a) Disciplinary committee;


(b) Examination committee;
(c) Executive committee;
(d) Professional guidance committee.

The Council may, as per necessity, form other committees for the attainment of
the objectives of the Institute.

As per section 14 of the ICAN Act, 2053; there shall be a disciplinary committee
as follows to inquire into a complaint and recommend the Council for necessary
action in cases where anyone lodges a complaint in the Institute that any member
has done any act or action contrary to this Act or the Rules or code of conduct
framed under this Act, or where the Institute receives such information:

(a) A fellow chartered accountant designated by the Council from amongst the
councilors referred to in Clause (a) of Sub-section (3) of Section 7 -Chairperson
(b) Three persons nominated by the Council from amongst the councilor -Member
(c) Two persons nominated by the Council from amongst the members -Member
(d) One person nominated by the Auditor General -Member

As per section 15 of the ICAN Act, 2053; there shall be formed an executive
committee comprising of the following members in order to carry out the day-to-
day business of the Institute, under the direction of the Council:
(a) Chairperson -Chairperson

© The Institute of Chartered Accountants of Nepal 9


CAP II Paper 3: Corporate and Other Laws

(b) Vice-chairperson -Vice chairperson


(c) Two persons nominated by the Council from amongst the councilors -Member
(d) Executive Director -Member-secretary

6. Answer the following questions: (5×4=20)


a) State the dispute settlement process between the foreign investor and
Nepalese investor by referring the provisions of the Foreign
Investment and Technology Transfer Act, 2049.
b) Write the provision related to “Meeting and decision of the Council” as
mentioned in the Social Welfare Act, 2049.
c) Anil has attained 17 years of age. Anil and Bikash have entered into a
contract of lease for land in Budhanilkantha, Kathmandu. However,
Bikash questioned about the legal competency of the Anil to conclude
any contract. What is the status of Anil in regard to the competency of
the contract? What types of contracts are regarded as void under the
Muluki Civil Code, 2074?
d) State the ratio of the contribution to be made to the Welfare Fund
under the Bonus Act, 2030.
e) State the role of the World Trade Organization (WTO) in the modern
trading activities.
Answer:
a) Section 7 of the Act provided the method and manner of deciding the disputes
which may arise between the foreign investor or national investor or industry. Sub
section 1 states that if there is any disputes relating to foreign investment between
the foreign investor, national investor or the concerned industry, the concerned
parties shall be required to settle the dispute by the mutual consultation in
presence of the Department.
If the dispute cannot be solved by the mutual consultation, as per sub section 2, it
shall be settled by referring the matter to arbitration according to the prevailing
arbitration rule of the United Nations Commission on International Trade Law
(UNICITRAL).
Further, the arbitration shall be held in Kathmandu and the law of Nepal shall be
applicable in the arbitration.
Similarly, Sub section 4 has given freedom to the parties to adopt the due
procedure regarding to the settlement of dispute in relation to the foreign
investment in industries in Nepal. Therefore, notwithstanding anything contained
in sub section 1, 2 and 3, the parties of foreign investment may provide provisions
for the settlement of dispute at the time of agreement for the foreign investment.

b) As per the section 11 of the Social Welfare Act, 2049,


1. The meeting of the Council shall take place in the date, time and place
specified by the chairman at least twice a year.
2. The quorum for the meeting of the Council shall be at least fifty percent of
total number of the member.
3. The Chairman shall preside the Council meeting in case of default of the
chairman, vice-chairman, and defaulted of both, the member selected among
themselves shall preside over the meeting.

© The Institute of Chartered Accountants of Nepal 10


CAP II Paper 3: Corporate and Other Laws

4. The opinion of the majority shall be credible in the meeting of the Council and
in case of equal vote chairman shall give the decisive vote.
5. The decision of the Council shall be verified by the member-secretary.
6. Other procedures relating to the meeting of the Council shall be determined by
the Council.

c) It is clear that the party of the contract of this contract is attained 17 years. It
should be reaffirmed by the Muluki Civil Code, 2074. Section 506 has mentioned
that any person, who is a minor, is not competent to enter to any contract. Section
2(e) has declared that a person who has not attained the 18 years age is said to a
minor. As, the age of 17, in this case concern, he is deemed to be a minor and said
to be incompetent to the contract. Further, the subsection (3) of section 506 has
given the right to a person who is a guardian or head (Mathabar) can enter into the
1 part of any contract and he is represented by his guardian and head for the
purpose of the contractual matter.
The following contracts shall be void in accordance with the section 507.
(a) A Contract preventing anyone from engaging him/herself in any occupation,
profession or trade which is not prohibited by prevailing law. Provided that a
contract shall not be deemed to have been concluded in preventing profession
or trade. :
(b) A contract restraining marriages other than those prohibited by the prevailing
law.
(c) A contract preventing any one from enjoying the facilities already being
enjoyed by the general public.
(d) A contract seeking to prevent the legal rights of any person from being
enforced by any government office or court.
(e) A contract concluded in matters, contrary to or prohibited by the prevailing
law.
(f) A contract concluded for immoral purpose or against Public morality or
public interest.
(g) A contract which cannot be performed because the parties thereto do not
exactly know about the matter in relation to which it has been concluded.
(h) A contract which is considered impossible to fulfill even at the time is
concluded.
(i) A contract which is vague as it does not provide reasonable meaning thereof.
(j) A contract concluded by an incompetent person to conclude such contract.
(k) A contract concluded with an unlawful consideration or objective.

d) Section 13 of the Bonus Act, 2030 has provided the provision for the Welfare Fund
where 70% of the residuary amount after distribution of bonus from the allocated
amount for bonus pursuant to Section 5 of the Act shall be deposited with the
Welfare Fund Established in accordance with the Labour Act, 2074 and remaining
30% shall be deposited with the National Level Welfare Fund, established by
Government of Nepal for the interest of the employees of the enterprises. The
operation of the welfare funds shall be in participation of employees as prescribed.

e) The World Trade Organization (WTO) is the only global international organization
dealing with the rules of trade between nations. At its heart are the WTO
agreements, negotiated and signed by the bulk of the world’s trading nations and

© The Institute of Chartered Accountants of Nepal 11


CAP II Paper 3: Corporate and Other Laws

ratified in their parliaments. The goal is to help producers of goods and services,
exporters, and importers conduct their business.

World Trade organization (WTO) has a crucial role to play in the international trade,
global economics, political and legal issues arising in the international business because of
the globalization.

WTO has emerged as a world’s most powerful institutions for reducing trade related
barriers between the countries and opening new markets.

The goal of WTO is to provide a fair platform for its member countries to help in services
like exports, imports and conduct their business in a peaceful manner.

Role of WTO in international business has been listed as follows:

 WTO facilitates implementation, administration and smooth operations of trade


agreements between the countries.
 It provides a forum for the trade negotiations between its member countries.
 WTO plays an important role for the settlements of disputes between the member
countries through the established rules and regulations.
 It cooperates with the IMF (International Monetary Fund) and World Bank in terms of
making cohesiveness in making global economic policies.

Overall WTO was set up to play a very important role in the world economics though
settling trade related disputes through rules, regulations and consensus based agreement
mechanisms that would prevent trade related wars between powerful countries.

7. Write short notes on the following: (2×5=10)


a) Establishment of the Nepal Securities Board.
b) Explain about the payment of insurance claims after the cancellation of
the insurer in the light of Insurance Act, 2049.
Answer:
a) Nepal Securities Board has been established by the Securities Act, 2063 in its
section 3. It is established in order to regulate and manage the activities of the
securities markets and persons involved in securities business by regulating the
issue, purchase, sale and exchange of securities in order to develop capital market
and protect the interests of investors in securities.
The Board shall consist the members as follows:-
(a) A person appointed by the Government of Nepal -Chairperson
(b) Joint Secretary, Ministry of Finance -Member
(c) Joint Secretary, Ministry of Law, Justice and Parliamentary Affairs -
Member
(d) Representative, Nepal Rastra Bank -Member
(e) Representative, Institute of Chartered Accountants of Nepal -Member
(f) Representative, Federation of Nepalese Chambers of Commerce and
Industries -Member
(g) A person nominated by the Government of Nepal from amongst the experts
who have obtained at least master's degree in economics, management,

© The Institute of Chartered Accountants of Nepal 12


CAP II Paper 3: Corporate and Other Laws

finance, commerce or law from a recognized university and gained at least


seven years of experience in stock exchange, management, capital market
development, finance and economic sector -Member

The concerned organization shall, in nominating its representative pursuant to


Clauses (e) and (f) of Sub-section (2), nominate a person who has obtained at least
bachelor's degree and gained at least seven years of experience in accounts,
industry, commerce, finance, banking, economics or law matters. The tenure of
office of the member nominated pursuant to clause (g) of Sub-section (2) shall be
three years. No act or proceeding of the Board shall be affected merely on the
reason of any vacancy in office of any member. An officer employee designated
by the Board shall act as the Secretary of the Board. If the Board thinks it
necessary, it may invite any native or foreign expert, adviser to attend its meeting
as an observer. The central office of the Board shall be situated in Kathmandu
Valley; and the Board may, as required, open its branch or contact office within or
outside of Nepal.

b) Section 16 of the Insurance Act, 2049 provides the following about the payment
of insurance claims after the cancellation of the insurer. After the cancellation of
the insurer, the insurer, dissolved by the cause of the cancellation of its
registration pursuant to section 13, shall refund the amount received by it for
insurance to the person, organization or the Insurance Board (Board), within the
period and method specified by the Board.

It shall refund the principal amount along with bonus as specified by the Board in
the case of life insurance, and it shall refund the principal amount as specified by
the Board on a proportional basis in the case of non-life insurance.

© The Institute of Chartered Accountants of Nepal 13


CAP II Paper 3: Corporate and Other Laws

Specific Comments on the performance of the students


Batch: - June 2019
Level: - CAP-II
Subject: Corporate & Other Laws
Question No. 1
Most of the students have attempted and answered in satisfactory way. Students found
the question normal and easy. Students have Confusion on IT industry and
Telecommunication industry. Students know the number of shareholders but not
mentioned the process of registration. There is lack of knowledge on writing skills
how to answer or respond on question. This is practical type of question but only
some students have written specified systematic answer. Whereas in the answer of Q.
1 (a) – provision of Section 9 mentioned and other answers found in general rather
than explanation as per demand of the question.
Question No. 2
Students have not focused on the legal provisions of the NRB and BAFIA. Most of
them have not answered in well manner. Many students have not mentioned specific
provision in regard to question no 2(a) and 2(b) almost have written same answer for
the question. In case of 2(b) restrictive clause provided that missing of clause.
Question No. 3
Students have understand the facts and done in satisfactory ways. This is practical
type of question but only some students have written systematic answer in form of
issue case raise, legal provision, conclusion and suggestion.
Question No. 4
Most of the students have not answered the questions because of lack of
understanding the legal provision under the concerned laws. Complete answer to 4 (a)
not written by many students and specification is also missing. Many students have
not attended or written correct answer to question no. 4(b). In case of part (a) students
need to explain circumstances in point, answer was not written as per expectation,
whereas in part (b) answer written was more than provision which are not necessary.
Question No. 5
Students have answered but only few of them have answered well. They need to focus
on the legal provisions. Students have confusion on standing committee's formation.
Some students have not mentioned four committee and formation/composition of
committee along with activities executed is not answered. Students gave partial
answer to question.
Question No. 6
Students have not studied the relevant legal provisions of the concerned laws; hence,
the answers are not specific. There is latest update in case of minor age which was
written mistake by many students. Question was not attempted by many students and
those who answered have answered in general form. Most of answers are not relevant
as per the provision of Muluki Civil Code, 2074.
Question No. 7
Students have misunderstood Question No. 7 (b) and answered as insolvency matter.
Formation of the Nepal Securities Board answer was answered partially by many
students leaving functions and experience in detail.

© The Institute of Chartered Accountants of Nepal 14


Paper 4: Financial Management
CAP II Paper 4: Financial Management

Marks
Attempt all questions.
Working notes should form part of the answer. Make assumptions wherever
necessary.

1.
a) Z Ltd. is a diversified company operating in different industries. The
shares of the company are traded in the stock exchange and currently
has a market price of Rs. 320 per share. The company‟s dividend
payment over the last five years are as follows:
Year 2018 2017 2016 2015 2014
Dividend Per Share 35 32 30 29 28
(Rs.)

The board of directors of Z Ltd. are currently considering two main


investment opportunities: one in Solar Energy and the other in the
Hotel and Tourism sector. Both projects have short lives and their
associated cash flows are as follows:
Year 1 2 3
Solar Energy (Rs. in million) 85 175 160
Hotel & Tourism (Rs. in million) 180 195 150
The investment in Solar Energy would cost Rs. 400 million while
that in Hotel and Tourism would cost Rs. 405 million.
The management of the company has identified the industry beta of
Solar Energy and Hotel and Tourism as 1.2 and 1.6 respectively.
However, a research conducted by management revealed that Z
Ltd.‟s beta is 1.5. The average return on the companies, listed on the
stock exchange, is 25% and the yield on Treasury bill is 20%. The
growth rate is 5.7%.
Required: (5+5=10)
i) Compute the net present values of both projects using the
company‟s weighted average cost of capital as a discount rate.
ii) Compute the net present values using a discount rate which take
into account the risk associated with the individual projects.
b) BCD Ltd. specializes in the production of “spring table water” for
which it has distributors both in the Northern and Southern parts of
Nepal. The consumers of the product in the East and West parts of
Nepal are clamoring for more branches in each of these areas to
enable them have this product within their easy reach.
The Managing Director of the company formed a project team to
study the feasibility of the branch expansion project as well as its
overall financial requirement. The team, after serious meetings and
deliberations, submitted its report containing the following
information relating to the branch to be opened in the Eastern part of
Nepal:
 Initial investment - Rs. 350,000 with nil scrap value
 Expected life span - 10 years

© The Institute of Chartered Accountants of Nepal 2


CAP II Paper 4: Financial Management

 Sales volume - 20,000 units per annum


 Selling price - Rs. 20 per unit
 Direct variable cost - Rs. 15 per unit
 Fixed cost excluding depreciation - Rs. 25,000 per annum.
 IRR - 17%.
The Managing Director is concerned about the viability of the
project as the IRR is close to the company‟s hurdle rate of 15%.
Therefore, he wanted you to evaluate the project very well so that it
does not run into a loss. Present value of annuity at company's hurdle
rate for 10 years is 5.0188.
Required: (8+2=10)
i) Compute the sensitivity of the NPV to each of the following
variables:
a. Sales price
b. Sales volume
c. Initial outlay
d. Variable cost
ii) From your calculation in (i) above, determine the two most
sensitive variables and interpret the result.
Answer:
a)
(i) Computation of NPV for both projects Using WACC of company
Solar Energy
Year Cash flow (Rs. in Discount Factor PV(Rs. in
million) @17% million)
0 -400 1 -400.00
1 85 0.855 72.68
2 175 0.731 127.93
3 160 0.625 100.00
NPV (99.39)
Hotel and Tourism
Year Cash flow (Rs.in Discount Factor PV (Rs.in
million) @17% million)
0 -405 1 -405.00
1 180 0.855 153.90
2 195 0.731 142.55
3 150 0.625 93.75
(14.80)
(ii) Projects NPV using CAPM
Solar Energy
Year Cash flow (Rs.in Discount Factor @ PV (Rs.in
million) 26% million)
0 -400 1 -400.00
1 85 0.794 67.49
2 175 0.630 110.25
3 160 0.500 80.00
NPV (142.26)

© The Institute of Chartered Accountants of Nepal 3


CAP II Paper 4: Financial Management

Hotel and Tourism


Year Cash flow (Rs.in Discount Factor PV (Rs.in
million) @28% million)
0 -405 1 -405
1 180 0.781 140.58
2 195 0.610 118.95
3 150 0.476 71.40
NPV (74.07)
Working Notes:
1. WACC of the company, here, represents the cost of equity calculates as:
Ke = 35(1.057)/320 + 0.057

= 17.26 % (Approx)
2.Cost of Capital Using CAPM (discount rate which takes into account the risk
factor):
Solar Energy: Rs = Rf + B (Rm – Rf)

20% + 1.2 (25 – 20) = 20% + 6% = 26%


Hotel and Tourism:
RH = Rf + B (Rm – Rf) = 20% + 1.6 (25% - 20%)
20% + 8% = 28%
Alternate Solution

WACC can be used by taking discount factor @ 27.50% (from CAPM)

WACC (ke) = Rf + β(Rm-Rf)

= 20% + 1.5(25-20)

= 27.5%
b)
i) Calculation of NPV
Year Items NCF PVIFA@ PV
(NRs.) 15% (Rs.)
0 Initial Outlay (350,000) 1.0000 (350,000)
1 - 10 Relevant Fixed
Cost (25,000) 5.0188 (125,470)
1 - 10 Variable Cost (300,000) 5.0188 (1,505,640)
1 - 10 Sales 400,000 5.0188 2,007,520
NPV 26,410
Contribution = Sales – Variable Cost
= 400,000 – 300,000
= Rs. 100,000
PV of Contribution Rs. 100,000 x 5.0188

© The Institute of Chartered Accountants of Nepal 4


CAP II Paper 4: Financial Management

= Rs. 501,880
Sensitivity Analysis:
Sales Price = (NPV/ PV of Sales) x 100 =(26,410/2,007,520) x 100 =
1.32%
Sales Volume = (NPV/ PV of Contribution) x 100 = (26,410/501,880) x 100 =
5.26%
Initial Outlay = (NPV/ PV of Outlay) x 100 = (26,410/350,000) x 100 =
7.55%
Variable Cost = (NPV/ PV of Variable Cost) x 100 = (26,410/1,505,640) x 100 =
1.75%
ii) The two most sensitive variables are:
Sales price at 1.32%
Variable Cost at 1.75%
These are derived from the sensitivity analysis workings above, as these are the two
least NPVs in terms of sensitivity.
The sales price must not fall by more than 1.32% and the variable cost must not
increase by more than 1.75%. Otherwise, the NPV starts going down.

2.
a) SSC Ltd. is considering the immediate purchase of some, or all, of
the share capital of one of two firms- SG Ltd. and CG Ltd. Both SG
and CG have one million ordinary shares issued and neither
company has any debt capital outstanding.
Both SG Ltd. and CG Ltd. are expected to pay a dividend in one
year‟s time. SG's expected dividend amounts to Rs. 30 per share and
that of CG is Rs. 27 per share. Dividends will be paid annually and
are expected to increase over time. SG‟s dividends are expected to
display perpetual growth at a compound rate of 6% per annum. CG‟s
dividend will grow at the annual compound rate of 33⅓% until a
dividend of Rs. 64 per share is reached in year 4. Thereafter CG‟s
dividend will remain constant.
If SSC is able to purchase all the equity capital of either company,
then the reduced competition would enable SSC to save some
advertising and administrative costs which would amount to Rs.
225,000 per annum indefinitely and, in year 2, to sell some office
space for Rs. 800,000. SSC would change some operations of any
company completely taken over, the details are:
SG – No dividend would be paid until year 3. Year 3 dividend would
be Rs. 25 per share and dividends would then grow at 10% per
annum indefinitely.
CG – No change in total dividends in years 1 to 4, but after year 4
dividend growth would be 25% per annum compounded until year 7.
Thereafter annual dividend per share would remain constant at the
year 7 amount.
An appropriate discount rate for the risk inherent in all the cash
flows mentioned is 15%.
Required: (4+6=10)
i) Calculate the value per share for a minority investment in each of

© The Institute of Chartered Accountants of Nepal 5


CAP II Paper 4: Financial Management

the companies, SG and CG, which would provide the investor


with a 15% rate of return.
ii) Calculate the maximum amount per share which SSC should
consider paying for each company in the event of a complete
takeover.
b) A firm provides you the following information:
Total earnings = Rs. 600,000
No. of Equity share of Rs. 100 each = 40,000
Dividend Paid = Rs. 160,000
Price-Earnings (P/E) Ratio = 10
The firm is expected to maintain its rate of return on fresh
investment.
Required: (3+2=5)
i) Ascertain whether the firm is following an optimal dividend
policy as per Walter's model and determine the payout ratio at
which dividend policy will have no effect on the value of the
share.
ii) Will your decision change if the P/E ratio is 5 instead of 10?
Answer:
a)
i) Using the dividend valuation model, the value of ordinary shares
is given by;
Vs = D0 (1+g)
Ke-g
Where D0 (1 + g) is the dividend due in one year.
Ke is the cost of equity or required return.
g is the anticipated growth in dividends.
Now,
Value per share of SG =30/(0.15-0.06)
= Rs. 333.33
The model must be modified slightly to estimate the value per share of CG
as follows:
= D1/(1+Ke)1 + D2/(1+ Ke)2+ D3/(1+ Ke)3+ D4/(1+ Ke)4 × 1/ Ke
= 27/(1.15)1 + 27(1.33)/(1.15)2 +27(1.33)2/(1.15)3 +64/(1.15)3 ×1/0.15
=23.48+27.15+31.40+243.95
= Rs. 362.8
ii) Maximum price in the event of a complete take-over:
Present value of cost savings:
Administrative costs= 225,000/0.15
= Rs. 1,500,000
Sale of office space = 800,000 x 0.7562
= Rs. 604,960
Total = Rs. 2,104,960
Saving per share = Rs. 2,104,960/1,000,000 = Rs.2.10
Value per share of SG (with change in operations):
Vs =25/(1.15)2×1/(0.15-0.10) = Rs.378.05

© The Institute of Chartered Accountants of Nepal 6


CAP II Paper 4: Financial Management

Maximum price = Rs. 378.05+Rs. 2.10 = Rs. 380.15


Value per share of CG:
Vc=27/1.15+(27×1.33)/(1.15)2 +

27×(1.33)2/(1.15)3+64/(1.15)4+(64×1.25)/(1.15)5+64×(1.25)2/(1.15)6+64×(1.
25)3/(0.15) ×(1/(1.15)6
=23.48+27.15+31.40+36.59+39.77+43.23+313.25
=Rs. 562.13
Maximum price = 562.13+2.10
=Rs. 564.23
b)
i) Rate of return on firm's investment (r) = 600,000 / (40,000 x 100) x100 =15%
Cost of Capital (Ke) = 1 / P/E ratio = 1/10 = 10%
Calculation of Payout ratio at which dividend policy will have no effect on the
value of the share:
Firm's dividend payout ratio = Rs.160,000/ Rs. 600,000 = 0.2667 or 26.67%
Rate of return of the firm (r) is 15%, which is more than its cost of capital (Ke)
10%.
Therefore, by distributing 26.67% of earnings, the firm is not following an
optimal dividend policy. The optimal dividend policy for the firm would be to pay
zero dividend and in such case, the market value of share under Walter„s model
would be as follows:
P = ((0 + (0.15 / (0.1) x (15- 0)) / 0.1
= Rs. 225 (Optional – Not Compulsory)
The market value of the share would increase by not paying dividend and by
retaining all the earnings of the company.
ii) Calculation of market value of share when P/E ratio is 5 instead of 10
The Ke of the firm is the inverse of P/E ratio i.e. 1/5 = 0.20. In such case Ke > r
Under the situation P/E ratio is 5, the optimum dividend policy for the company
would be 100% dividend payout at which the value of the firm would be
maximum.
3.
a) Following is the abridged balance sheet of Everest Co. Ltd. as at 31/03/2074:
Capital & labilities Rs. Assets Rs.
Paid up share capital 500,000 Freehold property 400,000
Profit and loss a/c 85,000 Plant & machinery 250,000
Current liabilities 200,000 (-) Depreciation 75,000
175,000
Stock 105,000
Debtor 100,000
______ Bank 5,000
785,000 785,000
The following information is also available:
i) The composition of total of the capital and liabilities side of the company‟s
balance sheet as at 32/03/2075 (the paid up share capital remaining the same as of
31/03/2074) was:
Share capital = 50% Profit & loss a/c = 15%

© The Institute of Chartered Accountants of Nepal 7


CAP II Paper 4: Financial Management

7% Debenture = 10% Creditors = 25%


The debenture were issued on 01/04/2074, Interest being paid on half yearly basis.
ii) During the year ended on 32/03/2075, additional plant & machinery had been
bought and a further Rs. 25,000 depreciation written off. Freehold property
remained unchanged. Total fixed assets constituted 60% of total of Fixed assets
and Current assets.
iii) Current ratio was 1.6 times and Quick ratio was 1 time.
iv) Debtor (4/5th of Quick assets) to sales ratio revealed a credit period of 2 months.
v) Gross Profit was 15% and Return on net worth was 10%. Ignore tax.
Requred: 8
Prepare Protit & loss account and Balance sheet of the company as at
32/03/2075.
b) Mr. X is to invest his funds in two securities, P & Q. The relevant
information is as follows:
P Q
Expected return 12% 20%
Standard deviation 10% 18%
Coefficient of correlation „r‟ between P & Q = 0.15
He has decided to consider only five portfolios of P & Q as follows:
(a) All funds invested in P
(b) 50% of funds in each of P & Q
(c) 75% funds in P and 25% in Q
(d) 25% funds in P and 75% in Q
(e) All funds invested in Q
Required: 7
i) Calculate return under different portfolios.
ii) Calculate Risk factor associated with these portfolios.
iii) Which portfolio is best for him from the risk point of view?
iv) Which portfolio is best for him from the return point of view?
Answer:
a)
Balance sheet of Everest Co. Ltd as on 32/03/2075

Liabilities & Capital Amount(Rs.) Assets Amount(Rs.)


Share Capital 500,000 Freehold Property 400,000
Profit & Loss A/C 150,000 Plant & Machinery
7% Debenture 100,000 300,000 200,000
Creditors 250,000 Less: Depreciation
(100,000) 150,000
Current Assets: 200,000
Stock 50,000
Debtors
Cash & Bank
Total 1,000,000 Total 1,000,000
Profit & Loss Account for the year ended 32/03/2075

Particulars Amount(Rs.) Particulars Amount(Rs.)


To Opening Stock 105,000 By Sales 1,200,000
To Purchase (Bal. figure) 1,065,000 By Closing Stock 150,000

© The Institute of Chartered Accountants of Nepal 8


CAP II Paper 4: Financial Management

To Gross Profit 180,000


1,350,000 1,350,000

To Expenses (Bal. Figure) 83,000 By Gross Profit 180,000


To Debenture Interest 7,000
To Depreciation 25,000
To Net Profit 65,000

180,000 180,000

Working Notes:
i) Total of the liabilities side = Rs. 500,000/ 0.5 = Rs. 1,000,000
ii) Profit & Loss a/c = 15% of Rs. 1,000,000 = Rs. 1,500,000
iii) 7% Debenture = 10% of Rs. 1,000,000 = Rs. 100,000
iv) Creditors = 25% of Rs. 1,000,000 = Rs. 250,000
v) Net Fixed Assets = 60% of Rs. 1,000,000 = Rs. 600,000
vi) Net Plant & Machinery = Rs. 600,000 – Rs. 400,000 = Rs. 200,000
vii) Gross Plant & Machinery = Rs. 200,000 + (Rs. 75,000 + Rs. 25,000) = Rs.
300,000
viii) Current Assets = Rs. 250,000 x 1.6 = Rs. 400,000
ix) Liquid Assets = Rs. 250,000 x 1 = Rs. 250,000
x) Stock = Rs. 400,000 – Rs. 250,000 = Rs. 150,000
xi) Debtors = Rs. 250,000/ 5 x 4 = Rs. 200,000
xii) Sales = Rs. 200,000 x 12/2 = Rs. 1200,000
xiii) Gross Profit = 15% of Rs. 1,200,000 = Rs. 180,000
xiv) Net worth = Rs. 500,000 + Rs. 150,000 = Rs. 650,000
xv) Net Profit = 10% of Rs. 650,000 = Rs. 65,000
b) Given that,
Return of P (Rp) = 12%
Return of Q (Rq) = 20%
Std Deviation of P (σp) = 10%
Std Deviation of Q (σq) = 18%
Coefficient of correlationbetween P & Q (r) = 0.15
i) Expected Return under different portfolio
Portfolio Return = Rp x Wp + Rq x Wq Return
All funds invested in P = 12% x 1 + 20% x 0 12%
50% of funds in each of P & = 12% x 0.50 + 20% x 0.50 16%
Q
75% funds in P and 25% in Q = 12% x 0.75 + 20% x 0.25 14%
25% funds in P and 75% in Q = 12% x 0.25 + 20% x 0.75 18%
All funds invested in Q = 12% x 0 + 20% x 1 20%
ii) Risk Factor associated under different portfolio
Risk =
All funds invested in P
=
= 10%

© The Institute of Chartered Accountants of Nepal 9


CAP II Paper 4: Financial Management

50% of funds in each of P &Q


=
= 10.93%
75% funds in P and 25% in Q
=
= 9.31%
25% funds in P and 75% in Q

=
= 14.09%
All funds invested in Q
=
= 18%
iii) Portfolio of investment of 75% in P and 25% in Q is best for him from
the point of risk as this portfolio has lowest risk of 9.31%
iv) Portfolio of investment of 100% in Q is best for him from the point of
return as this portfolio has highest return of 20%

4.
a) PQR Ltd. has current sales of Rs. 1.50 million per year. Cost of sales
is 75 percent of sales and bad debts are one percent of sales. Cost of
sales comprises 80 percent variable costs and 20 percent fixed costs,
while the company‟s required rate of return is 12 percent. PQR Ltd.
currently allows customers 30 days credit, but is considering
increasing this to 60 days in order to increase sales. It has been
estimated that this change in policy will increase sales by 15 percent
and bad debts will increase from one percent to four percent. It is
expected that the policy change will not result in an increase in fixed
costs, and creditors and stock will be unchanged.
Required: 8
Advise whether PQR Ltd. should introduce the proposed policy.
b) The valuation of a company has been done by an investment analyst.
Based on an expected free cash flow of Rs. 5.40 million for the
following year and an expected growth rate of 9 percent, the analyst
has estimated the value of the company to be Rs. 180 million.
However, he committed a mistake of using the book values of debt
and equity.
The book value weights employed by the analyst are not known, but
you know that the company has a cost of equity of 20 percent and
post-tax cost of debt of 10 percent. The market value of equity is
thrice its book value, whereas the market value of its debt is nine-
tenth of its book value.
Required: 7
Calculate the correct value of the company.

© The Institute of Chartered Accountants of Nepal 10


CAP II Paper 4: Financial Management

Answer:
a)
Rs. Rs.
Proposed investment in debtors = (1725000 x 60% + 1500000 x 15%) x
60/365 207,123.88
Less: Current investment in debtors = (1,500,000 x 75%) x 30/365 92,466.00
Increase in investment debtors 114,657.88
Increase in contribution = 15% x 1,500,000 x 40% 90,000.00
New level of bad debts = 1,725,000 x 4% 69,000.00
Current level of bad debts = 1,500,000 x 1% 15,000.00
Increase in bad debts (54,000.00)
Additional financing costs = 114,657.88 x 12% (13,758.87)
Savings by introducing change in policy 22,241.12
Decision
The financing policy is financially acceptable, although the savings are not
significant.
b) Cost of capital by applying Free Cash Flow to Firm (FCFF) Model is
as follows:
Value of Firm (V0) = FCFF1 / (Kc - gn)
Where, FCFF1 = Expected FCFF in year 1
Kc = Cost of Capital
gn = Growth rate =9%
Thus Rs. 180 m = 5.4 m / (Kc - gn)
Since, g = 9%
Kc -9% = 5.4/180
Kc = 0.03 +0.09 = 12%
Now, let X be the weight of debt and given cost of equity = 20% and cost of
debt=10%,
Then 20% (1-X) + 10%X = 12%
Hence, X=0.80, so book value weight of debt was 80% and accordingly
book value weight of equity was 20%
Thus, correct weight should be 60 (thrice of book value of equity) and 72
(nine-tenth of book value) of debt
Cost of capital = Kc= 20% (60/132) + 10% (72/132) = 14.55%
Correct value of the firm = Rs. 5.4 m / (0.1455 - 0.09) = Rs. 97.3 m
5.
a) Z Ltd. is presently financed entirely by equity shares. The current
market value is Rs. 600,000. A dividend of Rs. 120,000 has just been
paid. This level of dividend is expected to be paid indefinitely. The
company is thinking of investing in a new project involving an outlay
of Rs. 500,000 now and is expected to generate net cash receipts of
Rs. 105,000 per annum indefinitely. The project would be financed by
issuing Rs. 500,000 debentures at the market interest rate of 18%.
Ignore tax consideration.
Required: 5
i) Calculate the value of equity shares and the gain made by the
shareholders if the cost of equity rises to 21.6% after investment in
new project.

© The Institute of Chartered Accountants of Nepal 11


CAP II Paper 4: Financial Management

ii) Prove that weighted average cost of capital is not affected by


gearing.
b) Honda Ltd., a producer of turbine generators, is in the following
situation:
EBIT = Rs. 4 million
Tax rate = 35%
Debt outstanding = Rs. 2 million
Interest rate = 10%
Cost of equity (ke) =15%
Number of shares outstanding = 600,000
Book value per share = Rs. 10
Since Honda‟s product market is stable and the company expects no
growth, all earnings are paid out as dividends. The debt consists of
perpetual bonds.
Required: 5
i) What are Honda‟s earnings per share and price per share?
ii) Honda can increase its debt by Rs. 8 million. The new debt can be
used to buy back and retire some of its shares at the current price.
Its interest rate on debt will be 12% including the existing one and
its cost of equity will rise from 15% to 17%. EBIT will remain
constant. Should Honda change its capital structure?
c) A company has a maximum of Rs. 800,000 available to invest in
new projects. Three alternatives are available and the business
manager of the company has calculated net present value of each of
the alternatives as below:
Investment Initial Cash Outlay (Rs.) NPV (Rs.)
Alternative 1 540,000 100,000
Alternative 2 600,000 150,000
Alternative 3 260,000 58,000
Required: 5
Which investment alternative should the company invest in, if the
projects can be divided?

Answer:
a)

i) Current Cost of Equity


20%

© The Institute of Chartered Accountants of Nepal 12


CAP II Paper 4: Financial Management

Statement Showing Value of Equity Holders and Gain made by them


Particulars Existing Proposed
EBIT 120,000 225,000
Less: Interest ( 18% x 500,000) - 90,000
EBT 120,000 135,000
Less: Tax - -
Earning for Equity Holders 120,000 135,000
Cost of Equity 20% 21.6%
Value of Equity 600,000 625,000
Gain made by equity holders in terms of - 15,000
earning
Gain made by equity holders in terms of Value - 25,000
Value of Debt - 500,000
Value of Firm [ Value of Debt + Value of 600,000 1,125,000
Equity]
ii) At present Z Limited is unleveraged firm, therefore the Cost of Equity of the
firm as well as the weighted average cost of capital of the firm is 20%
The WACC of Z Limited after infusion of Rs. 500,000 of Debt will be;
WACC Cost of Equity] Cost of Debt]

21.6%] 18%]
= 20%
OR
Ko = EBIT/Value of firm

= 225,000/1,125,000
= 20%
Hence WACC is not affected by gearing.

b)
i) Calculation of EPS and Current Market Price
Earnings Before Interest and Taxes 4,000,000.00
Less: Interest [2,000,000*10%] 200,000.00
Earnings Before Tax 3,800,000.00
Tax @ 35% 1,330,000.00
Profit After Tax 2,470,000.00

© The Institute of Chartered Accountants of Nepal 13


CAP II Paper 4: Financial Management

Number of Outstanding Shares 600,000.00


Earnings Per Share 4.12
Cost of Equity 0.15
Price Per share under Zero Growth Model ( 4.12 ÷ 0.15) 27.44
ii) Additional Loan Financing 8,000,000.00
Number of Shares that can be bought back =8,000,000/27.44
291,545.00
Number of Shares Outstanding after buy back 308,455.00
Calculation of EPS and Current Market Price after Buy Back
Earnings Before Interest and Taxes 4,000,000.00
Less: Interest [10,000,000*12%] 1,200,000.00
Earnings Before Tax 2800,000.00
Tax @ 35% 980,000.00
Profit After Tax 1,820,000.00
Number of Outstanding Shares 308,455.00
Earnings Per Share 5.90
Cost of Equity 17%
Price Per share under Zero Growth Model ( 5.90 ÷ 0.17) 34.71
As the price per share increase from Rs. 27.44 to Rs. 34.71, the company should change its
capital structure.
c) Since we have limited fund available for investment, the normal NPV rule
of accepting investment decision with the highest NPV cannot be adopted
straight way. Further, as the projects are divisible, a Profitability Index (PI)
can be utilized to provide the most beneficial combination of investment for
the company.
Alternative PI Ranked as per PI
Alternative 1 640,000 / 540,000 = 1.185 III
Alternative 2 750,000 / 600,000 = 1.250 I
Alternative 3 318,000 / 260,000 = 1.223 II
Therefore, the company should invest as below:
Rs. 600,000 in alternative 2 as it has ranked first as per PI and
Rs. 200,000 in alternative 3 as it has been ranked second as per PI.
6. Write short note/ answer on: (4×2.5=10)
a) Liquidity preference theory
b) Repurchase agreements
c) Financial restructuring
d) Ploughing back of profits
Answer:
a) The liquidity preference theory of the term structure addresses the
shortcomings of the pure expectations theory by proposing that forward
rates reflect investors' expectations of future spot rates plus a liquidity
premium to compensate them for exposure to interest rate risk. Furthermore,
the theory suggests that this liquidity premium is positively related to

© The Institute of Chartered Accountants of Nepal 14


CAP II Paper 4: Financial Management

maturity: a 25-year bond has a larger liquidity premium than a 5-year bond.
The liquidity theory says that forward rates are biased estimates of the
market's expectation of future rates because they include a liquidity
premium. Therefore, a positive-sloping yield curve may indicate that either:
(1) the market expects future interest rates to rise; or (2) that rates are
expected to remain constant (or even fall), but the addition of the liquidity
premium results in a positive slope. A downward-sloping yield curve
indicates falling short term rates according to the liquidity theory.
The size of the liquidity premiums need not be constant over time. They
may be larger during periods of greater economic uncertainty, when risk
aversion among investors is higher.
b) A repurchase agreement is an agreement to buy any securities from a seller
on the understanding that they will be repurchased at some specified price
and time in the future. However, since the length of any repurchase
agreement (or „repo‟) is likely to be short, a matter of months at most, it is
customary to think of repos as a form of short-term finance and therefore,
logically, as being an alternative to other money market transactions. The
effect of the repo deal falls upon money market prices and yields, it is
normal to regard such repos as money market deals.
In a repo, the seller is the equivalent of the borrower and the buyer is the
lender. The repurchase price is higher than the initial sale price, and the
difference in price constitutes the return to the lender. Deals are quoted on a
yield basis.
Some repo deals are genuine sales. In these circumstances, the lender owns
the securities and can sell them in the case of default. In some repo
contracts, however, what is created is more strictly a collateralized loan with
securities acting as collateral while remaining in the legal ownership of the
borrower. In the case of default, the lender has only a general claim on the
lender and so the margin is likely to be greater.
c) Financial restructuring is carried out internally in the firm with the consent
of its various stakeholders. Financial restructuring is a suitable mode of
restructuring of corporate firms that have incurred accumulated sizable
losses for / over a number of years. As a sequel, the share capital of such
firms, in many cases, gets substantially eroded / lost; in fact, in some cases,
accumulated losses over the years may be more than share capital, causing
negative net worth. Given such a dismal state of financial affairs, a vast
majority of such firms are likely to have a dubious potential for liquidation.
Can some of these Firms be revived? Financial restructuring is one such a
measure for the revival of only those firms that hold promise/prospects for
better financial performance in the years to come. To achieve the desired
objective, such firms warrant / merit a restart with a fresh balance sheet,
which does not contain past accumulated losses and fictitious assets and
shows share capital at its real/true worth.
d) Long-term funds may also be provided by accumulating the profits of the
company and ploughing them back into business. Such funds belong to the
ordinary shareholders and increase the net worth of the company. A public
limited company must plough back a reasonable amount of its profits each
year keeping in view the legal requirements in this regard and its own
expansion plans. Such funds also entail almost no risk. Further, control of
present owners is also not diluted by retaining profits

© The Institute of Chartered Accountants of Nepal 15


CAP II Paper 4: Financial Management

7. Distinguish between: (4×2.5=10)


a) Mezzanine debt and Subordinated debt
b) Risk aversion and Risk diversification
c) Financial lease and Operating lease
d) William J Baumal and Miller- Orr cash management model
Answer:
a) Mezzanine debt is subordinated debt with some forms of equity
enhancement attached. Regular subordinated debt just requires the
borrowing company to pay interest and principal. With mezzanine debt, the
lender has a piece of the action in the company's business. The equity kicker
in a mezzanine loan can be in the form of attached stock warrants or bonus
payments to the lender based on the valuation of the company. Warrants are
used with publicly traded companies and other forms of equity participation
will be attached to the mezzanine debt of closely held corporations.
The use of mezzanine debt provides benefits to both lenders and the
borrowing company. The company taking out the debt gets to borrow at a
lower rate than it would pay on straight subordinated debt. The lender gets
the benefit of participating in the better results forecast by the additional
borrowing. If the borrower is successful in putting the newly borrowed
mezzanine financing to work, the return to the lender will be much higher
than just providing a subordinated loan.
Mezzanine debt can also be used to orchestrate the orderly transfer of
ownership of a company headed for bankruptcy. In bankruptcy, the equity
shareholders are typically wiped out and lose the ownership in the company.
The equity position attached to the mezzanine debt then becomes the
primary equity ownership in the company. This strategy allows a lender to
become the owner of a troubled company using the bankruptcy courts.

b) Risk aversion is the reluctance of a person to accept a bargain with an


uncertain payoff rather than another bargain with more certain, but possibly
lower, expected payoff. For example, a risk-averse investor might choose to
put his or her money into a bank account with a low but guaranteed interest
rate, rather than into a stock that may have high returns, but also has a
chance of becoming worthless. An investor is said to be risk averse if he
prefers less risk to more risk, all else being equal.
Risk Diversification refers to minimization of risk which an investor may
choose by investing in various types of securities. An investor may not want
to concentrate his investment in a single risky security, as a result of which
he may choose to invest in various other securities to minimize his level of
risk and harmonize his returns.

c) In the case of finance lease, risk and reward incident to ownership are
passed on the lessee. The lessor only remains the legal owner of the asset.
The lessee bears the risk of obsolescence. The lease is non-cancellable by
either party under it. The lessor does not bear the cost of repairs,
maintenance or operations. The lease is usually full payout.
In the case of operating lease, lessee is only provided the use of the asset for
a certain time. Risk incident to ownership belongs only to the lessor. The
lessor bears the risk of obsolescence. The lease is kept cancellable by the

© The Institute of Chartered Accountants of Nepal 16


CAP II Paper 4: Financial Management

lessor. Usually, the lessor bears the cost of repairs, maintenance or


operations. The lease is usually non-payout.

d) William J Baumal vs Miller- Orr Cash Management Model


According to William J Baumal‟s Economic order quantity model
optimum cash level is that level of cash where the carrying costs and
transactions costs are the minimum. The carrying costs refer to the cost
of holding cash, namely, the interest foregone on marketable securities.
The transaction cost refers to the cost involved in getting the marketable
securities converted into cash. This happens when the firm falls short of
cash and has to sell the securities resulting in clerical, brokerage,
registration and other costs.
The optimum cash balance according to this model will be that point
where these two costs are equal. The formula for determining optimum
cash balance is:
2UP
C = ,
S
Where
C = Optimum cash balance
U = Annual (monthly) cash disbursements
P = Fixed cost per transaction
S = Opportunity cost of one rupee p.a. (or p.m)

Miller-Orr cash management model is a net cash flow stochastic model.


This model is designed to determine the time and size of transfers
between an investment account and cash account. In this model control
limits are set for cash balances. These limits may consist of h as upper
limit, z as the return point, and zero as the lower limit.
When the cash balances reach the upper limit, the transfer of cash equal to
h-z is invested in marketable securities account. When it touches the lower
limit, a transfer from marketable securities account to cash account is made.
During the period when cash balance stays between (h,z) and (z, o ) i.e high
and low limits no transactions between cash and marketable securities
account is made. The high and low limits of cash balance are set up on the
basis of fixed cost associated with the securities transactions, the
opportunity cost of holding cash and the degree of likely fluctuations in cash
balances. These limits satisfy the demands for cash at the lowest possible
total costs.

Specific Comments on the performance of the students


Batch: - June 2019
Level: - CAP-II
Subject: Financial Management
Question No. 1
Most of the student did not answer the sensitivity part of 1(b). Students are
able to calculate cost of capital (rate) based on CAPM. However, in most

© The Institute of Chartered Accountants of Nepal 17


CAP II Paper 4: Financial Management

cases, rate based on Gordon has not been calculated. In most of the cases,
students calculated NPV correctly. However, sensitivity calculation is
incorrect in every case. Many students fails to calculate the cost of the
company. Students did not understand the sensitivity analysis. There is wrong
calculation of overall cost of capital in many cases.
Question No. 2
(a) Cost savings calculation is not correct in every case. Majority students fails to
consider the cost saving. PV of cost saving is not calculated by many students
value per share and maximum price is not calculated properly in case of complete
takeover. Almost all the students were not able to treat correctly savings for
complete takeover.
(b) Students were unable to compare return with cost in most of the cases. Calculation
of rate of return was not done by majority of students. Some of the student did not
able to compute return.
Question No. 3
(a) Students prepared the financial statement (B/S and P&L) correctly in most of the
cases. Ratios were not calculated properly. Some students wrongly calculate paid
up share capital. Most of the student solved the question but made mistake
specially in computing return on net worth sales.
(b) Concept of portfolio management is clear for majority of students. Students fail to
calculate the risk factor properly. Some made the mistake in computing risk.
Question No. 4
(a) Majority of students were able to calculate benefit under change in receivable
policy. Calculation of additional financial cost was not done properly by majority
of students. Some students were confused to calculate opportunity cost of funds
blocked in receivables. Calculation mistake on cost of sales and opportunity cost
of investment in receivables. Made mistake in the computation of opportunity
cost, specially the total cost of debtors for the proposed.
(b) Very few students could answer the valuation part correctly. Students did not
understand the use of weight while calculating cost of capital. Most of the students
were not able to solve the question.
Question No. 5
(a) Satisfactory answer was given by students. Weighted average cost of capital was
not calculated properly by majority of students. Some student used divided price
approach.
(b) Impact of Buy-Back of share was not correctly answered by majority of students.
Students failed to calculate the shares that can be brought back. Most students
were not able to compute number of outstanding share after the bought back.
(c) Answered correctly in most of the question. However, concept of PI is not clear in
most of the cases. Students lacks concept of application of PI technique.
Question No. 6
Students were unable to interpret liquidity preference theory and repurchase
agreements. There is lack of conceptual knowledge. Maximum students answered this
question as liquidity analysis. Some write it as buy back of share. Answer is not
specific.
Question No. 7
Mezzanine debt and subordinated debt was not answered by any of the students.
There is lack of conceptual knowledge. Even answer was not specific most able to
write it. Most answers were written from the accounting standard provision.

© The Institute of Chartered Accountants of Nepal 18


Paper 5: Cost and Management Accounting
CAP II Paper 5: Cost and Management Accounting

Marks
All questions are compulsory. Working notes should form part of the answer.
Make assumptions wherever necessary.
1. From the cost record of the APEX Company for the year 2075 of product
A, the information given is extracted.
This period actual
Sales (unit) 10,000
Profit (loss) Rs. 10,000
Fixed Cost Rs. 30,000
Variable cost per unit Rs. 8
On the basis of the information determine: (3+4+4+4+5=20)
a) What is present cost structure and break event unit?
b) What increased sales volume is required to cover an extra attractive
packaging cost of Rs. 0.5 per unit, to increase the sales at the existing
sales price, to yield zero profit?
c) What increase in sales volume is required, at the present sale price, to
cover additional publicity expenses of Rs. 5,000 for that period, while
yielding a profit of Rs. 5,000?
d) What increased sales volume is required to reach a profit of Rs.
12,500 while reducing the selling price by 3% per unit?
e) What impact in profit and loss and BEP if selling price increase to Rs.
12 and fixed cost Rs. 50,000
Answer:
a. Present Cost Structure
Variable Cost 10,000*8 80,000.00
Fixed Cost 30,000.00
Total Cost 110,000.00
Profit 10,000.00
Total Sales (Rs) 120,000.00
Actual Sales in Unit 10,000.00
Selling Price Per Unit 12.00
Variable Cost Per Unit 8.00
Contribution Per Unit 4.00

Break event sale unit = fixed cost/contribution


=30,000/4
=7,500 units
b. Breakeven point if extra packing cost is incurred to yield zero profit.
Rs.
Fixed Cost 30,000.00
Sales Price per unit 12.00
Variable Cost per unit including Rs 0.5 for extra packaging cost 8.50
Contribution per unit 3.50

© The Institute of Chartered Accountants of Nepal 2


CAP II Paper 5: Cost and Management Accounting

Break Even Sale Units


= Fixed Cost / Contribution per unit = 30000/3.5 = 8,571.43 units

Since actual sales at 10,000 units is already in excess of the BEP i.e. 8,571.43 units, no
increase in sales volume would be required to cover an extra attractive package to yield zero
profit.

c. Sales volume required for profit of Rs 5,000 when additional publicity Expenses of
Rs 5,000 is incurred.
Rs.
Fixed Cost at Present 30,000.00
Add: Additional Publicity Expenses 5,000.00
Total Fixed Cost 35,000.00
Profit Required 5,000.00
Total Contribution Required 40,000.00
Volume of Sales (units) to be achieved to attain
40000/4 10,000.00
contribution at Rs. 40,000

Since the actual sales is already 10,000 units, no increase in sale volume would be
required, at present sale price, to cover additional publicity expenses of Rs 5,000 and also
profit of Rs 5,000.

d. Volume of sales to have profit of Rs 12,500 with reduced sales price of 3%


Rs.
Reduced Sales Price per unit 97% of 12 11.64
Variable Cost per unit 8.00
Contribution per unit 3.64

Total Contribution Required Rs


Fixed Cost 30,000.00
Profit 12,500.00
Total Contribution Required 42,500.00
Sales Volume (units) Required to attain contribution 42500/3.64 11,675.82
of Rs 42,500
The sales volume should be increased by 1675.82 units to reach a profit
of Rs 12500.

e. Impact in Profit and Loss if selling price increase to Rs 12 and fixed cost to Rs 50,000
Break Even Point = Fixed Cost/(Selling price - Variable Cost)
= Rs. 50,000/(12-8)
= 12,500

© The Institute of Chartered Accountants of Nepal 3


CAP II Paper 5: Cost and Management Accounting

Note : Selling price is already Rs 12 per unit.


Sales 10000*12 120,000.00
Fixed Cost 50,000.00
Variable Cost 80,000.00
Total Cost 130,000.00
Loss 10,000.00

Total impact due to increase in Fixed would be reduction of profit by Rs 20,000 from
Profit of Rs. 10000 to loss of Rs 10,000. Also, Breakeven point will increase by 5,000
from current level of 7,500 units to 12,500 Units.

2.
a) The following standards have been set to manufacture a product:
Direct Materials Amount(Rs.)
2 Units of X at Rs. 40 per Unit 80
3 Units of Y at Rs. 30 per Unit 90
15 Units of Z at Rs. 10 per Unit 150
320
Direct Labour 3 hours @ Rs. 55 per Unit 165
Total Standard Prime Cost 485
The company manufactured and sold 6,000 units of the product during
the year 2018.
Direct Material costs were as follows:
12,500 units of X at Rs. 44 per unit.
18,000 units of Y at Rs. 28 per unit.
88,500 units of Z at Rs. 12 per unit.
The company worked 17,500 direct labour hours during the year
2018. For 2,500 of these hours the company paid Rs. 58 per hour
while for the remaining hours the wages were paid at the standard
rate.
Required: 10
Compute the following Variances:
i) Material Price,
ii) Material Usage,
iii) Material Mix,
iv) Material Yield,
v) Labour Rate and
vi) Labour Efficiency.
b) Sales Managers of two companies (say company X and company Y)
compare notes and find that their sales turnover for last year was the
same viz, Rs. 10 lakhs and the profits they made also were the same
being 10% of turnover. In one company the fixed cost were double the
variable costs while in the other, it was quite opposite, the variable
costs were double the fixed costs. As an accountant, do you think that
they are equally profitable? If not, explain their relative vulnerability
to market conditions. 10

© The Institute of Chartered Accountants of Nepal 4


CAP II Paper 5: Cost and Management Accounting

Answer:
a) Material Price Variance = Actual Quantity (Std. Price – Actual Price)
X = 12,500 Units (Rs. 40 – Rs. 44) = 50,000 (A)
Y = 18,000 Units (Rs. 30 – Rs. 28) = 36,000 (F)
Z = 88,500 Units (Rs. 10 – Rs. 12) = 1,77,000 (A)
1,91,000 (A)
Material Usage Variance = Std. Price (Std. Qty – Actual Qty.)
X = Rs.40 (6,000 × 2 – 12,500) = 20,000 (A)
Y = Rs.30 (6,000 × 3 – 18,000) = Nil
Z = Rs.10 (6,000 × 15 – 88,500) = 15,000 (F)
5,000 (A)
Material Mix Variance = Std. Price (Revised Std. Qty. – Actual Qty.)
X = Rs.40 ( - 12,500) = 24,000 (A)

Y = Rs.30 ( - 18,000) = 4,500 (A)

Z = Rs.10 ( - 88,500) = 7,500 (F)


21,000 (A)
Material Yield Variance = Std. Price (Std. Qty. – Revised Std. Qty.)
X = Rs.40 (6,000 x 2 - ) = 4,000 (F)

Y = Rs.30 (6,000 x 3 - ) = 4,500 (F)

Z = Rs.10 (6,000 x 15 - ) = 7,500 (F)


16,000 (F)
Labour Rate Variance = Actual Hours (Std. Rate – Actual Rate)
= 2,500 hours (Rs.55 – Rs.58) =7,500 (A)
Labor Efficiency Variance = Std. Rate (Std. Hours – Actual Hours)
= Rs.55 (6,000 x 3 – 17,500) = 27,500 (F)
b)
STATEMENT OF PROFITABILITY
(Rs in
lakhs)
Company X Company
Y
Sales 10 10
Less: Variable Costs 3 6
Contribution 7 4
Less: Fixed Costs 6 3
Profits 1 1
P/V Ratio 70%
40%
Break -even point [Fixed cost/PV Ratio (Rs.)] 8.57 7.5
Margin of Safety
(Actual Sales – BE sales) / Actual sales *100%
14.3%
25%

© The Institute of Chartered Accountants of Nepal 5


CAP II Paper 5: Cost and Management Accounting

It is clear from the above analysis that the Company X and Company Y are
not equally profitable. Company X has a higher P/V Ratio as compared to
company Y. But at the same time, its fixed costs are also high .As a result
company X is more vulnerable to market fluctuations as compared to
company Y. For example if the sales fall by 20% the company X will suffer a
loss of Rs 40,000 while company Y will still make a profit of Rs 20,000. This
shows that a company with a low fixed cost and high variable cost is less
vulnerable to market fluctuations as compared to a company with low
variable costs and high fixed costs.
Working Notes:
(i) Computation of Variable Cost for Company X
Let the variable costs for company X be 'X'
Fixed costs for company X will be 2X
Total costs=9 lakhs
Hence X+2X=9 lakhs
or X=3 lakhs.
The Fixed costs in company X will therefore be Rs. 6 lakhs.
(ii) Computation of Variable Costs for company Y
Let the variable costs of Y be 'Y'
The fixed Cost will be 0.5 Y
Total Costs= 9 lakhs.
Hence, Y+0.5Y= 9lakhs.
or Y=6 lakhs.
In Company Y, the Variable Costs are therefore Rs. 6 lakhs and Fixed Costs
are Rs. 3 lakhs.
3.
a) The annual demand for an item of raw material is 4,000 units and the
purchase price is expected to be Rs. 90 per unit. The incremental cost of
processing an order is Rs. 135 and the annual cost of storage is estimated to
be Rs. 12 per unit. Compute the optimal order quantity and total relevant cost
of this order quantity.
Suppose that Rs. 135, as estimated to be the incremental cost of processing
an order, is incorrect, and should have been Rs. 80. All other estimates are
correct. Estimate the difference in cost on account of this error.
Assume at the commencement of the period that a supplier offers 4,000 units
at a price of Rs. 86. The materials will be delivered immediately and placed
in the stores. Assume that the incremental cost of placing the order is zero
and original estimate of Rs. 135 for placing an order for the economic batch
is correct. Analyze, should the order be accepted? 8
b) The boiler house is one of the service departments of a company. Steam is
raised and then transferred to production departments and other service
departments as required.
The basic monthly budget figures for 2014 are as follows:
Boiler operating hours: 480
Steam raised: 8,000,000 kg.
Costs:
Fuel (V) Rs.19,200
Chemicals (V) Rs. 960

© The Institute of Chartered Accountants of Nepal 6


CAP II Paper 5: Cost and Management Accounting

Wages (F) Rs. 2,400


Sundry overheads (F) Rs. 3,000
The actual figures for February 2014 are as follows:
Boiler operating hours: 432
Steam raised: 6,750,000 kg.
Costs:
Fuel (V) Rs. 18,000
Chemicals (V) Rs. 990
Wages (F) Rs. 2,200
Sundry overhead (F) Rs. 3,000
It is expected that the price of chemicals for all output will fall by 2%
where the boiler operates in excess of 480 hours per month. Sundry
fixed (F) costs are expected to fall by Rs. 200 where the boiler is
operated for less than 425 hours and to increase from the normal level
by Rs. 250 where the boiler is operated for more than 480 hours.
Variable (V) costs vary in proportion to boiler hours.
Required:
i) Prepare a budget summary which shows the cost of the boiler
house in total and per „ kg. steam for boiler operating levels of
400, 432, 480 and 540 hours. 6
ii) Prepare a control statement which compares budget with actual
cost of the boiler house for February where a flexible budgeting
system is in operation. 2

c) Explain “Escalation and De-escalation clause” in the context of


contract costing. 4
Answer:
a)
i) Optimal order quantity i.e. E.O.Q.
=
=
= 300 units
Relevant Cost of this order quantity Rs.
Ordering Cost = 4,000/300 13.33 at Rs.135 1,800
Carrying Cost = ½ x 300 x 12 1,800
Relevant Cost 3,600

ii) Revised EOQ = = 231 units


Ordering Cost = 4,000/231 = 17.32 at Rs.80 1,386
Carrying Cost = ½ x 231 x 12 1,386
2,772
Different in cost on account of this error = 3,600 – 2,772 =Rs.828

iii) In case of discount in purchase price, the total cost of purchase cost,
ordering cost and carrying cost should be compared.
Original offer at Rs.90 per unit Supplier offered at Rs.86 per unit
Rs. Rs.
Purchase cost 3,60,000 Purchase cost 4,000 x 86 3,44,000

© The Institute of Chartered Accountants of Nepal 7


CAP II Paper 5: Cost and Management Accounting

Ordering cost 1,800 Ordering cost Nil


Carrying cost 1,800 Carrying cost ½ x 4,000x12 24,000
Total cost 3,63,600 3,68,000
This special offer at Rs.86 per unit should not be accepted as its total cost is
higher by Rs.4,400 (3,68,000 – 3,63,600) as compared to original offer.
b)
i) Flexible Budget Summary
Boiler operating 400 432 480 540
hours:
Steam 6667 7200 8000 9000
raised(„ kg)
Costs
Fuel (V) Rs. 16,000 Rs. 17,280 Rs. 19,200 Rs. 21,600
Chemicals(V) 800 864 960 1,058.4
Wages(F) 2400 2400 2400 2,400.0
Sundry 2800 3000 3000 3,250.0
overheads (F)
Total 22,000 23,544 25,560 28,308.4
Cost/‟ kg 3.30 3.27 3.195 3.145

ii) Control statement for the month


Budget Actual Variances
Boiler operating hours 432 432
Steam raised („ kg) 7,200 6,750 450 (Adverse)
Costs: Rs Rs Rs.
Fuel(V) 17,280 18,000 720 (Adverse)
Chemical(V) 864 990 126 (Adverse)
Wages(F) 2,400 2,200 200 (Favourable)
Sundry overhead(F) 3,000 3,000 nil
23,544 24,190 646 (Adverse)
c) Escalation clause in a contract provides that if during the period of execution
of a contract, the price of materials, rate if labour etc. rise beyond a specified
limit, the contract price will be increased by specified rate or amount. It does
not cover that part of increase in costs which is caused due to inefficiency or
wrong estimation.
Thus, an escalation clause is meant to safeguard the interest of the contractor
against unforeseen rise in cost. There may be a De-escalation clause or
Reverse clause to provide for any future decreases in price etc., so that benefit
may be passed on to the contractee.

4.
a) Following costs were incurred in producing 800 MT of M.S. Rods:
Amount ( Rs.)
Materials 280,000
Labor 100,000
Processing Charges 100,000
Total cost 480,000

© The Institute of Chartered Accountants of Nepal 8


CAP II Paper 5: Cost and Management Accounting

Of the total output, 10% was defective and had to be sold after a
discount of 10% of the normal price. The scrap arising out of the
production realized a sum of Rs. 8,760. The sale price is calculated
to yield 15% profit on sales. You are required to find out the normal
price as well as the discounted price of per MT of M.S. Rods. 5
b) A manufacturer introduces new machinery into his factory, as a
result of which, production per worker has increased. The workers
are paid by results, and it is agreed that for every 2% increase in
average individual output, an increase of 1% on the rate of wages
will be paid. Just after installation of machinery the selling price of
the products falls by 8 1/3 %.
Before New Machinery After New Machinery
(Actual) (Planned)
Number of workers 175 125
Number of articles produced 8,400 7,000
Wages paid Rs. 16,800
Total Sales Rs. 37,800
Show the net saving in production costs which would be required to
offset the losses expected from reduced turnover and bonus to be paid
to workers. 5
c) From the following figures, calculate cost of production and profit
for the month of March 2019. 5
Amount (Rs.) Amount (Rs.)
st
Stock on 1 March, 2019 Purchase of raw 2,857,000
materials
Raw materials 606,000 Sale of finished goods 13,400,000
Finished goods 359,000 Direct wages 3,750,000
st
Stock on 31 March, 2019 Factory expenses 2,125,000
Raw materials 750,000 Office and 1,034,000
administration expenses
Finished goods 309,000 Selling and distribution 750,000
expenses
Work-in-process: Sale of scrap 26,000
On 1st March, 2019 1,256,000
On 31st March, 2019 1,422,000
Answer:
a)
Cost of Production of M.S. RODS.
Materials Rs. 280,000
Labor 100,000
Processing Charges 100,000
Total cost 480,000
Less: Sale value of scrap materials 8,760
Net cost of Production 471,240

© The Institute of Chartered Accountants of Nepal 9


CAP II Paper 5: Cost and Management Accounting

Profit @ 15% of sales or 15/85 of cost 83,160


Total Sales Value 554,400
Equivalent Good Production MT 792
Price of good production=554400/792 =Rs 700
Discounted price of Defective Production (Rs. 700 - 70) Rs. 630
Working Notes
M.T.
Equivalent unit of sales 800
Less: Defective production @ 10% 80
Total production ( good only) 720
Add: Equivalent of defective production @ 90% of 80 72
Equivalent Good Production 792
b)
Relevant cost/sales Rs.
Fall in sales value 2,625.00
Increase in wages cost 1,000.00
Total expected Loss 3,625.00
Therefore, net saving in production costs that would be required to offset the
expected loss is Rs. 3,625.
Working Notes:
If machinery After
was not Installation of
Description installed Machinery
Sales of 8400 Units
Sales of 7000 Units before fall in sales value
(37800/8400)*7000 31,500.00
Fall in sales value 2,625.00
(31500*0.25/3)

125 workers would produce


(8400/175)*125 6000 7000
Excess production 1000
Increase in labour efficiency
(1000/6000 * 100) 16.67%
Increase in wages rate 8.33%
('16.67%/2)
Wages cost for 125 workers before
increasing wages rate
(16800/175)*125 12000
Wages cost for 125 workers after increasing
wage rate 13,000.00
(12000*108.33%)
Increase in wages cost 1,000.00

© The Institute of Chartered Accountants of Nepal 10


CAP II Paper 5: Cost and Management Accounting

Alternative Answer

No. of Article/Worker = = 48
Revised No. of Article/Worker = = 56
% Increase in efficiency = ×100%
= 16.67%
% Increase in rate of wages = ×16.67% = 8.33%
Existing Rate/Unit = =2
Therefore, Revised Piece Rate = 2×108.33% = 2.1666

Therefore, Bonus to worker = Revised wages – Existing wages


= (7000×2.1666) – (7000×2)
= 15,166.2 – 14,000
= 1166.2

ii)
Fall in Sales Value = existing Sales × % decrease in S.P
= × 8.333%
= 2625

Therefore, Total expected loss = Fall in sales value + Bonus


= 2625 + 1166.2
= Rs. 3791.2

Therefore Net saving in production cost required to offset expected loss is Rs. 3791.2

c) Calculation of cost of production and profit for the month ended March, 2019
Particulars Amount (Rs.) Amount (Rs.)

Materials consumed
-Operating stock 6,06,000
-Add: Purchases 28,57,000
34,63,000
-Less: Closing stock (7,50,000) 27,13,000
Direct Wages 37,50,000
Prime cost 64,63,000
Factory expenses 21,25,000
Less: Sale of scrap (26,000) 20,99,000
Factory Cost 85,62,000
Add: Opening W-I-P 12,56,000
Less: Closing W-I-P (14,22,000)
Net Factory cost 83,96,000
Office and administration expenses 10,34,000
Cost of Production 94,30,000
Add: Opening stock of finished goods 3,59,000

© The Institute of Chartered Accountants of Nepal 11


CAP II Paper 5: Cost and Management Accounting

Less: Closing stock of finished goods (3,09,000)


Cost of goods sold 94,80,000
Selling and distribution expenses 7,50,000
Cost of sales 1,02,30,000
Profit (balancing figure) 31,70,000
Sales 1,34,00,000
5.
a) A factory which was allocating 'overhead expenses' to jobs on the
basis of 'prime cost' found the resulting cost ludicrous. The
management, therefore, decided that overheads should be allocated
either on the basis of direct labor hours or direct labor cost. Scrutiny
of a normal month‟s accounts showed the following:
Factory cost incurred Rs. 5,000
Direct labor hours worked 5,000 man hours
Direct labor costs Rs. 2,500

The Direct costs incurred on two typical jobs were as under:


Job Order 847-A Job Order 848-B
Direct materials Rs. 35 Rs. 35
Direct labor cost 4 4.5
Direct labor hours 8 6
You are required to compute the cost of each of these two jobs on the
basis of „Direct labor cost rate‟ and „Direct labor hour rate‟ and give
your views as to which of these two rates are more equitable for
adoption. 7
b) Outline the Method of Accounting of Administrative Overhead. 4
c) You have been asked to install a costing system in a manufacturing
company. What practical difficulties you expect and how would you
overcome those difficulties? 4
Answer:
a) Direct Labor Cost Rate: Factory expenses/ Direct labor costs * 100
= Rs. (5,000/2,500) *100 = 200 %
Direct Labor Hours Rate: Factory expenses/ Direct labor hours * 100
= Rs. (5,000/5,000) *100 = Rs. 1 per hour

Statement of Cost Of The Jobs (3 Marks)


Job Order 847-A Job Order 848-B
A B
Direct
labor Direct
Direct labor cost labor
Direct labor hour rate rate hour rate
cost rate (Rs.) (Rs.) (Rs.) (Rs.)
Direct Materials 35 35 35 35

© The Institute of Chartered Accountants of Nepal 12


CAP II Paper 5: Cost and Management Accounting

Direct labor cost 4 4 4.5 4.5


Overhead Expenses 8 8 9 6
Total Cost 47 47 48.5 45.5
*200% of direct labor cost **Re 1 per labor hour
The higher rate of wages (75 paise per hour) for Job Order No. 848-B
indicates that the jobs seems to have been done by workers of higher skill
than those who were engaged on the other job ( wage rate 50 paise per hour).
From this point of view, the amount of overhead charged to Job Order No.
848- B should be lower than that charged to the other job even if the time
spent were to be the same. But in case overheads are recovered as a
percentage of direct wages, more overheads are recovered from Job Order
848-B.
The direct labor hour rate method of charging overheads will give more
equitable results in such a case since overheads are mostly connected with
time.
b) There are three distinct methods of accounting of Administrative Overheads
(a) Appropriation of Administrative Overheads between Production and
Sales Department
(b) Charging to Profit and loss Account:
This method is used when the Administrative overhead are not directly
related to production and selling & distribution, and it is difficult to
determine a suitable base.
(c) Treatment of Administrative Overheads as a separate addition to cost of
Production: This method considers administration as a separate function
like production and sale.
c) The practical difficulties that arise while installing a costing system in a
manufacturing company are as follows:
i) Lack of top management support: Installation of a costing system does
not receive the adequate support of top management. They consider it as
interference in their work. They believe that such a system will involve
additional paperwork. They also have a misconception in their minds that
the system is meant for keeping a check on their activities.
ii) Resistance from cost accounting departmental staff: The staff resists
because of fear of losing their jobs and importance, after the
implementation of the new system.
iii) Non co-operation from user departments: The foremen, supervisor and
other staff members may not co-operate in providing requisite data, as
this would not only add to their responsibilities but will also increase
paper work of the entire team as well.
iv) Shortage of trained staff: Since cost accounting system‟s installation
involves specialized work, there may be a shortage of trained staff.
To overcome these practical difficulties, necessary steps required are:
 To sell the idea to top management – To convince them of the utility
of the system.
 Resistance and non-co-operation can be overcome by behavioral
approach. To deal with the staff concerned effectively.
 Proper training should be given to the staff at each level

© The Institute of Chartered Accountants of Nepal 13


CAP II Paper 5: Cost and Management Accounting

 Regular meetings should be held with the cost accounting staff, user
departments, staff and top management to clarify their doubts

6. Write short notes: (4×2.5=10)


a) Halsey and Hasley Weir Systems
b) Over and Under Absorption Overhead
c) Budget Manual
d) Sunk Costs
Answer:
a) Under this scheme, for performing a job, operation or task, a standard time is
specified. The hourly rate is fixed & the workers are guaranteed so that even
if, within the standard time specified, the job is not completed by them,
guaranteed time rate may be received by them. The worker becomes entitled
to bonus, if he is in a position to complete the job in less than the specified
time; bonus being equal to his time wage for 50% of the time saved in
addition to the time wage which he is entitled for the actual time worked.
Formula for calculating wags under Halsey and Hasley Weir Systems:
= Time taken * time rate + 50% of time saved * Time rate

Under Halsey Weir System, a worker is entitled to bonus if he/she completes


the job in less than specified time and bonus being equal to 30% of time
saved.

Earning under Halsey Weir System = (Time Taken*Hourly Rate)+30%


(Time Saved * Hourly Rate)

b) When a company uses standards costing, it derives a standard amount of


overhead cost that should be incurred in an accounting period, and applies
this standard amount of overhead to cost objects. If the actual amount of
overhead turns out to be different from the standard amount of overhead, then
the overhead is said to be either under absorbed or over absorbed.
If overhead is under absorbed, this means that more actual overhead costs
were incurred than expected, with the differences being charged to expenses
as incurred. This usually means that the recognition of expenses is
accelerated into the current period, so that the amount of profit recognized
declines. If overhead is under absorbed, this means that fewer actual overhead
costs were incurred than expected, so that more cost is applied to cost objects
than were actually incurred. This means that the recognition of expenses is
reduced in the current period, which increases profit.

c) A budget manual is defined by ICMA as „a document or manual which sets


out the responsibilities of the person engaged in budgeting that contains
details regarding budgeting organization and procedures including the routine
of and the forms and records required for budgetary control.
The budget manual thus is a schedule, document or booklet, which contains
different forms to be used, procedures to be followed, budgeting organization
details, and set of instructions to be followed in the budgeting system. It also

© The Institute of Chartered Accountants of Nepal 14


CAP II Paper 5: Cost and Management Accounting

lists out details of the responsibilities of different persons and the managers
involved in the process. A typical budget manual contains the following:
 Objectives and managerial policies of the business concern.
 Internal lines of authorities and responsibilities.
 Functions of the budget committee including the role of budget officer.
d) Sunk costs are historical or past costs. These are the costs which have been
created by a decision that was made in the past and cannot be changed by any
decision that will be made in the future. Investments in Plant and Machinery,
Buildings etc. are prime examples of such costs. Since sunk costs cannot be
altered by decisions made at the later stage, they are irrelevant for decision-
making.

© The Institute of Chartered Accountants of Nepal 15


CAP II Paper 5: Cost and Management Accounting

Specific Comments on the performance of the students


Batch: - June 2019
Level: - CAP-II
Subject: Cost & Management Accounting
Question No. 1
Most of the students have well attempted the questions. However there were some
lapses on the methodology they used. Some students did not prepare the cost structure
well. There was general understanding of the questions but most students focused on
the questions properly. As the question has asked the increased/decreased in number,
but student didn't responded accordingly.
Question No. 2
(a) Most students have solved the question very nicely.
(b) This question was very good. It has asked the essence of the cost accounting and
management. All of the students responded properly however there is lacking of
conceptual understanding to understand the question.
Question No. 3
(a) Most of the students attempted well. However, most students consider the
ordering cost in the incremental cost also.
(b) Most students attempted it correctly.
(c) Most students describe escalation clause properly. However, most students had
problem in describing the De- escalation clause.
Question No. 4
(a) Most students couldn't solve the question properly. They seem confused in
calculating the equivalent units. They were not able to calculate the profit
properly.
(b) Calculating net saving in production cost seemed to be quiet confusing for
students.
(c) Most students solved the questions properly. Most were confused a using the
value of scrap on the cost sheet while calculating the net factoring cost.
Question No. 5
(a) Most students calculated the total cost on both labor cost and labor hour cost. But
some were confused in giving the justification on their views.
(b) Most students solved the question wisely.
(c) There seem some lacking of concept about practical implication on installing the
costing system. Most wrote theoretical implication on the installation of costing
system.
Question No. 6
(a) Most students solved the question correctly.
(b) These seem same lacking of understanding an absorption/expected/budgeted
overhead by most of the students. They should revisit it again.
(c) None of the student solve it correctly.

© The Institute of Chartered Accountants of Nepal 16


Paper 6A: Business Communication
CAP II Paper 6: Business Communication & Marketing

Marks
All questions are compulsory.
Section -'A'
1. Read the following case carefully and answer the questions given below: (4×5=20)
Jane Rye is a student of advertising at State University and will graduate at the
end of the next term. She has a part-time job in the sales department at a local
television station. When hired, Rye thought she was very lucky to have a job
there, not only for the money but also for the work experience.
Pat Trent, the sales manager who hired her, was Rye’s immediate supervisor.
Rye was doing a very good job and received considerable support from Trent.
In fact, the sales manager had nothing but praise for Rye’s work when
reporting to top management. Trent often told her subordinate that her work
was exceptional and Trent would like to hire her on a permanent basis after
graduation to head a new media research department for the station. The job
seemed to promise a challenging and rewarding career.
While Rye was flattered by the offer, she was not interested in the position
because she found her present job unsatisfying. However, she never told Trent
her feelings about the job or the possible appointment. Because Trent had
trained Rye and had promoted her to everyone, Rye had become very loyal
and grateful to her sales manager. Thus, Rye thought she would betray Trent
if she were to refuse the job. After six weeks, however, Rye decided to quit
and work part-time at the university, but she did not know how to approach
her boss.
Rye, feeling unable to say anything to Trent, let time pass until the day she
was ready to quit to start her new job. When Rye got to work that day, the
sales manager was scheduled to leave town later that morning. Rye was forced
to go into Tent’s office while two other people were there, discussing another
matter. Trent asked Rye what she wanted, and Rye replied, ―I am resigning.‖
The sales manager was taken completely by surprise, asked Rye why she was
resigning, and wondered what was to be done with the project Rye was
handling. Rye apologized for such short notice. Rye explained that she was
taking a part-time job at the school starting tomorrow. Trent, very
disappointed in her, said, ―If you had told me sooner, I could have phased out
the project to someone else- now I’m in a bind,‖
a) How should Rye have handled her resignation?
b) Where, when, and how do you think Rye should have resigned? Do you
think Trent would have understood under different circumstances?
c) How did Trent foster Rye’s reluctance to communicate?
d) What are some possible long-term repercussions of the way Rye handled
her resignation?
Answer:
a) Rye should have resigned more gracefully and more professionally. She should have
given adequate notice to her supervisor, Mr. Trent so that he could have saved his
project by hiring another appropriate employee. She also should have talked to her

© The Institute of Chartered Accountants of Nepal 2


CAP II Paper 6: Business Communication & Marketing

supervisor in greater details about the probable difficulties he might face because of
her abrupt discontinuation in the job. She also should have talked to him the problems
and difficulties she faced during the work because of which she decided to quit.
Lastly, to save the project, she should have continued until the project is completed.

b) She should have explained the circumstances that forced her to quit, in a formal
resignation letter, may be a month prior to her resignation date. Once she provided
Trent with her legitimate reasons for quitting the job, Trent would also have reacted
professionally and gracefully. Then the story would have been different.

c) Trent also failed to understand Rye properly. He just saw what he wanted to see. He
is also a failure as supervisor, especially in regard with communication. He was so
confident on her that his blind faith on her at the end proved to be fatal to the
organization. He might have learnt a lesson that managers must try to listen to their
subordinates.

d) In this case, the organization may immediately lose some money and reputation.
Trent may also lose faith on humanity or even in his capacity to recognize people. It
may reduce the level of trust on each other in the long run. If stakeholders of
organizations are unprofessional, businesses will ultimately collapse.

2. What importance does ethical communication have in business? Discuss some


of the ways of maintaining ethical standards in business. (5+5=10)
Answer:
As a business communicator, you should understand basic ethical principles so that
you can make logical decisions when faced with dilemmas in the workplace.
Professionals in any field must deal with moral dilemmas on the job. However, just
being a moral person and having sound personal ethics may not be sufficient to
handle the ethical issues that you may face in the workplace. On the job you will face
many dilemmas, and you will want to react ethically.
Taking ethics into consideration can be painful in the short term. But in the long term
ethical behaviour makes sense and pays off. Dealing honestly with colleagues and
customers develops trust and builds stronger relationships. Many businesses today
recognize that ethical practices make good business sense. Ethical companies endure
less litigation, less resentment, and less government regulation. The following
guidelines can help you set specific ethical goals and maintain a high ethical standard.

Abiding by the Law. Know the laws in your field and follow them. Particularly
important for business communicators are issues of copyright law. Don't assume that
Internet items are in the "public domain" and free to be used. Internet items are also
covered by copyright laws.

Telling the Truth. Ethical business communicators do not intentionally make state-
ments that are untrue or deceptive. We become aware of dishonesty in business when
violators break laws, notably in advertising, packaging, and marketing. Half truths,

© The Institute of Chartered Accountants of Nepal 3


CAP II Paper 6: Business Communication & Marketing

exaggerations, and deceptions constitute unethical communication. But conflicting


loyalties in the workplace sometimes blur the line between right and wrong.

Labeling Opinions. Sensitive communicators know the difference between facts and
opinions. Facts are verifiable and often are quantifiable; opinions are beliefs held with
confidence but without substantiation. Stating opinions as if they were facts is
unethical.

Being Objective. Ethical business communicators recognize their own biases and
strive to keep them from distorting a message. Honest reporting means presenting the
whole picture and relating all facts fairly.

Using Inclusive Language. Strive to use language that includes rather than excludes.
Do not use expressions that discriminate against individuals or groups on the basis of
their gender, ethnicity, disability, or age. Language is discriminatory when it
stereotypes, insults, or excludes people.

Giving Credit. Ethical communicators give credit for ideas by (1) referring to ori-
ginators names within the text, (2) using quotation marks, and (3) documenting
sources with endnotes, footnotes, or internal references. In school or on the job,
stealing ideas or words from others is unethical.
3.
a) What is workforce diversity? Explain briefly. 5
b) Suppose you’ve applied in an XYZ Company for the programmer
position. It’s been nearly two weeks and yet you haven’t heard anything
from the company. Write a follow-up letter to Mr. John Gilbert, the
Human Resource Manager of XYZ Company regarding the confirmation
of the receipt of your application. Also add further necessary information. 5

Answer:
a) Diversity is the collective strength of experiences, skills, talents, perspectives, and
cultures that each agent and employee brings to the organization. It is very important
term in intercultural communication. It’s how we create a dynamic business
environment to serve our customers. Hence, the organization attempts to create an
inclusive working atmosphere. Inclusion is about respecting and valuing the unique
dimension each agent and employee adds to the organization. The firms recognize
that agents and employees are at their creative and productive best when they work in
an inclusive work environment.

We look at diversity differentiating three dimensions of analysis: a symbolic, social,


and individual dimension. By maintaining these dimensions, we try to avoid reducing
the complexity of diversity, which in the normal case is multi-dimensional.

Although the concept is often framed in terms of ethnic background, a broader and
more useful definition of diversity includes ―all the characteristics and experiences
that define each of us as individuals.‖ As an example, any of the companies today

© The Institute of Chartered Accountants of Nepal 4


CAP II Paper 6: Business Communication & Marketing

identifies separate dimensions of diversity in its discussions of workforce diversity,


including gender, race, age, military experience, parental status, marital status,
religious and ethnic background, geography and thinking style.

b)

Mr. John Gillbert


XYZ Company
87 Silken Road
Omega, Nepal.

Jan. 24, 2019

Dear Mr. Gillbert,

I submitted a letter of application and a resume earlier this month for the programmer
position advertised in the Times Union. To date, I have not heard from your office. I
would like to confirm receipt of my application and reiterate my interest in the job.

I am very interested in working at XYZ Company and I believe my skills and


experience would be an ideal match for this position.

If necessary, I would be glad to resend my application materials or to provide any


further information you might need regarding my candidacy. I can be reached at
(555)555-5555 or binusharma@abcd.com. I look forward to hearing from you.

Thank you for your consideration.

Sincerely,

Signature

Binita Sharma

4. Write short notes on: (5×2=10)


a) Graphics in business communication
b) Overcoming group problems
c) Information overload
d) Demerits of written communication
e) Proxemics
Answer:
a) Graphics in business communication
Graphics, one of the highly effective non-verbal tools commonly used in business
communication refers to different designs, drawings or pictures that we keep in our
power point slides, advertisements, business texts, brochures, instructions, manuals,
etc. The usefulness of graphics in business communication can never be

© The Institute of Chartered Accountants of Nepal 5


CAP II Paper 6: Business Communication & Marketing

underestimated since graphic representation of information becomes not only clear


and precise but also impressive and persuasive. It is commonly believed that a picture
is worth thousand words. Line graphs, histograms, bar charts, pie charts, figures, etc.
are the common examples of graphics.

b) Overcoming group problems


Working in groups may invite a number of problems though it is believed that team
work contributes to attaining the organizational goals and missions. The group
problems are to be settled so that the expected outcomes can be ensured. Some of the
important strategies for overcoming the group problems are:
 Working with group spirit and identity;
 Discouraging personal skepticism, lobbying, influences and thoughts;
 Training on group autonomy and group dynamism;
 Generating the sense of cooperation, endurance and collaboration.

c) Information overload
Information is the power but if one tries to send too much information at once, it may
be difficult for the listener/ reader. Too much information is as bad as too little. It is
because it reduces the receiver’s ability to concentrate effectively on the most
important messages. People facing an information overload may sometimes tend to
ignore major part of the messages, delay in responding to others, answer only
partially to the messages, respond inaccurately to the messages, spend less time with
each message, and respond only superficially to all the messages. So effective
communicators must be aware of this potential trap.

d) Demerits of written communication


 It is costly and time consuming.
 It becomes difficult to maintain privacy about written communication.
 It is rigid and doesn’t provide much scope for making changes.
 It is very formal and lacks personal touch.
 It is often too formal and structured.
 Different people may interpret it differently.
e) Proxemics
Proxemics or the use of space, is another area of cultural difference. We might see
differences in a meeting room seating arrangement depending upon the proxemics of
a culture. People in the United States have an intimate zone of less than 18 inches for
people they know very well, a social zone of 4 to 12 feet used with people with whom
they work closely, and a public distance of over 12 feet for those they do not know.
Cultural standards create different proxemics—some engage in hugging and cheek or
air kisses. In the United States a private office is considered a status symbol, whereas
in Japan there are only a few and even those are not used a great deal of the time. In
France, the boss sits in the middle of the work area, with his subordinates in offices
around him or her.

© The Institute of Chartered Accountants of Nepal 6


CAP II Paper 6: Business Communication & Marketing

Paper 6B: Marketing

© The Institute of Chartered Accountants of Nepal 7


CAP II Paper 6: Business Communication & Marketing

All questions are compulsory.


Section -'B'
5. Read the following case carefully and answer the questions given below: (4×5=20)
Ram Bahadur Thapa recently took voluntary retirement from Nepal Bank Ltd.
at the age of 56. He worked in this same bank for the last 30 years. At
retirement he received Rs. 12 lakh as gratuity. He is the father of three sons –
Shyam Bahadur (26), Harka Bahadur (20) and Gokul Bahadur (16). His wife
Sharmila (50) is still working in a secondary school as math teacher. They live
in a two-bedroom house in Sitapaila, Kathmandu.
In 1990, he had bought a Sony 20‖ TV and a Chinese VCD. These were the
only means of entertainment for the family. The TV set had grown too old and
needed replacement. The set had worn out beyond any possibility of repair.
Sharmila also proposed that the family should exchange both TV and VCD
and buy a modern flat screen TV and DVD. Ram Bahadur agreed on the
proposal to replace the TV but disagreed on buying the DVD. He argued that
the VCD is all right and serves the purpose of occasional viewing of movies.
Harka Bahadur insisted that the image and sound quality of DVD is far better
than that of the VCD. He argued ―everybody today has a DVD‖. He suggested
that his father should visit some shops in Mahabaudha and listen to the sound
and image quality of the DVDs. Ram Bahadur asked his son ―what should I
do with the old VCD, no one would buy it even for Rs. 1000‖. Harka Bahadur
suggested that it should be thrown away as garbage.
Ram Bahadur agreed to his son’s suggestion and took him to the market to
look at the DVD alternatives. The price range of the different brands varied
between Rs. 3500 for the Chinese to Rs. 9000 for Korean DVDs. Ram
Bahadur was highly impressed with the DVDs convenience and the slim look.
Both the Chinese and Korean products looked similar. He wanted to buy
cheaper one, but his son suggested buying the more expensive one. Harka
Bahadur insisted on buying a popular brand ―Samsung is so popular and has
high image; besides, it has a warranty period of two years‖. Ram Bahadur
argued ―the Chinese is not only cheap but strong, see the last VCD is still
working for the last 15 years‖. They could not decide on the brand to buy and
finally agreed that they would consult the other family members and buy that
brand which receives the highest vote.
a) How did Ram Bahadur’s family recognize their needs?
b) What are the sources of information used by Ram Bahadur’s family?
c) Explain the role played by various members in the buying process of TV
and DVD.
d) Explain the brand preference of Ram Bahadur and his son about Chinese
and Korean DVD.

© The Institute of Chartered Accountants of Nepal 8


CAP II Paper 6: Business Communication & Marketing

Answer:
a) Need is the state of felt deprivation. Consumer purchases various types of goods or
services to satisfy their needs. Need recognition is the starting point of consumer
behavior. Need is activated because of the internal stimuli and external stimuli.
Customers will recognize needs if there is change in actual or ideal state. In Ram
Bahadur’s family, the TV set had grown too old and needed replacement. The set had
worn out beyond any possibility of repair. It indicates decrease in actual state. But in
case of DVD, old VCD is still working for its occasional use, but Ram Bahadur’s son
and wife are insisting him to buy DVD. It indicates increase in ideal state. Similarly,
need of TV set is activated because of the internal stimuli and need of DVD is
activated because of the external stimuli as Ram Bahadur’s son has expressed his
view about the picture and sound quality of DVD and he also had said that everybody
had DVD in their home.
Thus, they have recognized their needs because of, decrease in actual state in case of
TV and increase of ideal state in case of DVD.

b) Information search is the second stage in consumer buying process. Customers search
information form internal source and external source. Internal source is related with
retrieval of their own memory. If information retrieved from internal source is not
enough, consumers search information from external sources. External source
includes personal source, commercial source, public source and experimental source.
In Ram Bahadur’s family, they have used both internal as well as external sources of
information. Ram Bahadur, his wife and His son Harka Bahadur have used internal
sources of information by the retrieval of their own memory. Again they have used
commercial sources of information form the shopkeeper of Mahabaudha. They have
also used the information from experimental sources by observing picture and sound
quality of the TV and DVD at the shop.

c) In consumer buying decision process, role played by different family members may
be initiator, influence, decider, purchaser etc. In Ram Bahadur’s family, Ram
Bahadur’s wife Samrila has played the role of initiator as she has proposed to
exchange both TV and VCD. She has also initiated to purchase flat screen TV and
DVD. Ram Bahadur and his son have played the role of influencers as Ram Bahadur,
in the beginning, not interested to buy DVD. In the later phase he has insisted to buy
Chinese DVD as he is the satisfied customer of Chinese VCD. It was purchased in
1990 and which was still working for its occasion use. Similarly, his son Harka
Bahadur has influenced tried to influence Ram Bahadur to purchase DVD because of
its picture and sound quality. Again Harka Bahadur has also suggested his father to
visit Mahabaudha. He has also insisted to buy Korean DVD. He has tried to influence
Ram Bahadur to purchase Korean DVD by explaining popularity of Samsung brand
and warranty period of two years. But other family members have played no role in
the given case.

d) In the given case, there is no confusion about purchasing TV set. In case of DVD
Ram Bahadur and his son Harka Bahadur have different choice. Ram Bahadur wants

© The Institute of Chartered Accountants of Nepal 9


CAP II Paper 6: Business Communication & Marketing

to purchase Chinese DVD whereas his son Harka Bahadur is in favor of Korean
DVD. Ram Bahadur is satisfied customer of existing Chinese VCD. Thus, as a
satisfied customer he wants to purchase Chinese DVD. Similarly, Harka Bahadur is in
favor of Korean DVD because of his age and personality factor. He is also inspired by
the brand consciousness because in consumer behavior, young people are highly
brand conscious than older people. In the given case, age of Ram Bahadur is 56 and
age of Harka Bahadur is only 20. Again, in consumer behavior, young people are risk
taker and matured people are risk avoider. In the given case, this risk taking
behaviour is also reflected in their brand preference while selecting Chinese or
Korean DVD.

6. Describe the impacts of macro environment in marketing. 10


Answer:
The marketing environment is everything your company must take into consideration
when developing and presenting a new product. The elements of a marketing
environment include, but are not limited to, the changing preferences of customers,
your competition, the legal, political and regulatory environment, your own resources
and budget, current trends and the overall economy. All these elements affect your
marketing decisions -- or at least they should, because all of them influence your
prospects.

Tastes and Trends


To be successful, a marketing plan should focus on consumer preferences and current
market trends. In recent days, many large retailers have decided to adapt to
consumers' increasing enthusiasm for social media by establishing corporate Twitter
accounts and opening online storefronts in Facebook. Consumers no longer need to
visit a retailer's main website to buy; some platforms allow them to make the
purchase without ever leaving Facebook. Companies that fail to take major trends
into account may find their sales lagging behind competitors'.

Budget and Economy


Your budget plainly has a role in your marketing decisions. It dictates how much
advertising you buy and where you can afford to place it. The overall economy also
has a massive influence on your marketing decisions. If you're marketing in a down
economy, your consumers won't be willing to pay a premium for your product, and
your advertising should probably point out that the product saves your customers
money, costs less than your competitor's product, or lasts a long time and is therefore
a good value. In a strong economy, your strategy probably will change. You'll be able
to charge more, and your ad message may stress the pleasure or convenience your
product offers your customers

Competitors
Prudent marketing decisions must factor in competitors -- how many you have and
how good they are at what they do will affect your marketing plans. If, for example,
your competitors are able to offer their product for a much lower price than yours,
your marketing strategy must stress the fact that your product is of a higher quality,

© The Institute of Chartered Accountants of Nepal 10


CAP II Paper 6: Business Communication & Marketing

that your warranty is better, or that your product lasts longer. If you have few or no
local competitors, you're free to expand into new markets.

Legal and Political


Changes in the political and legal environment can restrict or even end certain
marketing activities. The tobacco industry is a case in point. The U.S. government
first mandated cigarette warning labels in 1965, and, as evidence of cigarettes'
harmful health effects grew, it battled the tobacco industry more and more vigorously.
These battles culminated in the passage of a 2009 law that gave the federal
government the authority to regulate tobacco, which also includes its marketing and
labeling. In Nepal few years back the government has mandated the same warning
labels. Therefore, before offering a product, you should consider whether it is a
candidate for legal or regulatory trouble.

7.
a) What are external factors affecting price determination? 5
b) Describe the components of marketing information system. 5
Answer:
a) Price is exchange value what the buyers get and seller receive. Determining price is a
technical as well as difficult job for any producers. Generally, it is determined on the
base of total cost. In this global competition, price is affected by the both internal and
external factors. External factors are explained below:

1. Competition:
While fixing the price of the product, the firm needs to study the degree of
competition in the market. If there is high competition, the prices may be kept low
to effectively face the competition, and if competition is low, the prices may be
kept high.
2. Consumers:
The marketer should consider various consumer factors while fixing the prices.
The consumer factors that must be considered includes the price sensitivity of the
buyer, purchasing power, and so on.
3. Government control:
Government rules and regulation must be considered while fixing the prices. In
certain products, government may announce administered prices, and therefore the
marketer has to consider such regulation while fixing the prices.
4. Economic conditions:
The marketer may also have to consider the economic condition prevailing in the
market while fixing the prices. At the time of recession, the consumer may have
less money to spend, so the marketer may reduce the prices in order to influence
the buying decision of the consumers.
5. Channel intermediaries:
The marketer must consider a number of channel intermediaries and their
expectations. The longer the chain of intermediaries, the higher would be the
prices of the goods.

© The Institute of Chartered Accountants of Nepal 11


CAP II Paper 6: Business Communication & Marketing

b) The four components of Marketing Information System are described below.


Internal Database
Internal Database is also known as Internal Records System. Internal data is
electronic collection of information obtained from data sources within the company.
Like from accounting department (financial statements, records of sales, cash flows),
from operations department (production schedules, shipments, inventories), from
Sales force (reseller and competitors activities), from marketing department
(customer demographics psychographic, buying behaviour, customer satisfaction,
service problems, customer database). Sometimes may need surveys on employees.
Internal databases accessed quickly & cheaply. Internal data Problems: it is collected
not for marketing and data ages quickly, out of date.
These records provide past and current information about sales, costs, inventories,
cash flows and account receivable and payable. Many organizations maintain their
internal records from the computerized system. These records help to forecast the
future through analyzing the past and present scenario of the organizations too.
Marketing Intelligence System
Marketing Intelligence is systematic collection and analyses of publicly available
information about competitors and daily developments in marketing environment.
Marketing intelligence can be achieved through quizzing own employees(employees
knows customers better), Bench marking competitors’ products/activities, surfing the
internet/social media, visiting industry tradeshows, searching rivals parking lots/ trash
bins outside their building, knowing thought common suppliers, resellers, customers,
other companies' annual reports, publications, press releases, advertisements of
competitors, spying, employing actors(disguised shopping), talking to off duty
employees of competitors.
It collects information from external sources such as magazines, trade journals,
books, commercial publication and so on which provide the information about current
marketing-environment and changing conditions in the market. Marketing
intelligence acts as a mirror of prevailing marketing environment reflecting precisely
how things are going on in the market.
Marketing Research System
Marketing Research is collecting specific information systematically. Marketing
Research is systematic design, and analyses and reporting of data relevant to specific
marketing situation facing an organization like market potential, markets share,
effectiveness of pricing, product, distribution, promotion, advertising research, sales
research (efficiency of sales personnel, sales potentials, efficient distribution,
forecasting future sales), motivational research and attitude research etc.
It is conducted to solve specific problems of the company. Thus, it is a formal study
of specific problems, opportunities or situations. It pinpoint the problem, collects data
to solve the problem, data are analyzed, conclusions are drawn and then
recommendations are given for solving the problem. According to Kotler (2006)
marketing research is the systematic design for collection, analysis and reporting of
data and findings relevant to specific marketing situations facing by the company.
Thus, marketing research is related to any problem regarding buyer behavior, product
or brand preferences, product or brand usage, advertising awareness, sales promotion,
physical distribution, dealer behavior, competition etc.

© The Institute of Chartered Accountants of Nepal 12


CAP II Paper 6: Business Communication & Marketing

Decision Support System (Information Analysis)


Decision Support System (DSS) is also known as Information Analysis component
in MIS. Data, research may require more analysis, advanced statistical analysis.
Stores, analysis and synthesizes the information/data gathered from various sources.
DSS has three components (1) data bank (lazar, register, books, computer hard
drives) (2) statistical bank (averages, %, dispersion, cross tabulations, multivariate,
regression, factor analysis, cluster, conjoint analysis soft wares) (3) model bank
(synthesis of two or more information to prescribe the right course of actions.
Examples are BCG/GE Matrix, Markov model, queuing model, game theory,
heuristics, X2/chai-square test etc. models)

8. Write short notes on: (5×2=10)


a) Branding
b) Stages of consumer buying process
c) Public relations
d) Components of physical distribution
e) Market segmentation
Answer:
a) Branding
Branding is a process of creating a unique name, term, design, symbol, or any other
feature to the products. It helps to identify manufacturer of the product. It plays an
important role in marketing. It reflects the features of the product.

b) Stages of consumer buying process


Consumer buying process consists of following stages:
 Need recognition
 Information search
 Analysis of alternatives
 Purchase decision
 Post purchase evaluation
 Disposal

c) Public relations
Answer: Public relations are a broad set of activities used to create and maintain
favorable attitude of customers, government officials, press and society toward
company. It is achieved through effective personal relationships, presentation of a good
corporate image, social responsiveness and charity work. Tools of Public Relations are
as follows
 Media relations
 Group relations
 Lobbying
 Blogging
 Social media marketing
 Sponsorship of social events

© The Institute of Chartered Accountants of Nepal 13


CAP II Paper 6: Business Communication & Marketing

d) Components of physical distribution


I. Transportation
II. Warehousing
III. Material Handling
IV. Inventory management
V. Order Processing
VI. Information Management

e) Market Segmentation
The process of defining and subdividing a large homogenous market into clearly
identifiable segments having similar needs, wants, or demand characteristics. Its
objective is to design a marketing mix that precisely matches the expectations of
customers in the targeted segment.

© The Institute of Chartered Accountants of Nepal 14


CAP II Paper 6: Business Communication & Marketing

Specific Comments on the performance of the students


Batch: - June 2019
Level: - CAP-II
Subject: Business Communication
Question No. 1
Overall performance is ok, but requires more clarity in terms of opinion and restatement
of the case time and again needs to be reduced. Many students were found writing either
lengthy or very short answer.
Question No. 2
Most of the students have correctly addressed this question, but could have been
improved by adding diagrams.
Question No. 3
Students were found failing/performing very poor at writing a follow up letter. Students
have not more clarity in 'Business ethics'. There was lacking of formalities in follow up
letter.
Question No. 4
Some of the students were found not familiar with the term 'Kinethesis'. Due to the nature
of the question and weightage of marks, the answers were not in detail as the candidate
had to give short answer. In general, there is lacking of clarity in the specifications of
different resumes.

Subject: Marketing
Question No. 5
In general, dealing with case can be more improved. Candidates were supposed to relate
the specified problem with existing theory to deal case precisely. The students were not
conscious about the case studies. Majority of them were not serious about the issue of the
cases.
Question No. 6
Candidates were found answering superficially for the asked questions having higher
weight. Most of the students did not understand about principles of modern marketing
concept.
Question No. 7
Though the answer seems satisfactory with respect to concept, presentations of dealing
question can be improved. Students are confident in this section but confused in factor
affecting the determination of promotion mix. They are clear about the process of new
product development.
Question No. 8
Concept is clear. However, there is still room for improvement in answering.

© The Institute of Chartered Accountants of Nepal 15


Paper 7: Income Tax & VAT
CAP II Paper 7: Income Tax & VAT

Marks
Attempt all questions. Working note should form part of the answer.

1. Juntara Trades is a partnership firm with 4 partners. It is an authorized distributer of


SUPERB brand of household goods. The firm has drawn income statement relating to
Income Year (IY) 2074/75 as follows:
Particulars Amount (Rs.)
Sales 55,194,100
Less: Cost of Goods Sold 44,155,280
Opening Stock 3,219,660
Materials Purchased 46,455,030
Less: Closing Stock (5,519,410)
Gross Profit 11,038,820
Less: Administrative & Selling Expenses 4,723,196
Salary & Wages 3,311,646
Business Promotion Expenses 500,000
Repairs & Maintenance 375,000
Office Expenses 50,000
Lawyers Fee 45,000
Marketing Expenses 441,550
Net Profit 6,315,624
The accountant provides the following additional information in relation to the above
transactions:
a) Salary and Wages includes Rs. 50,000 paid for installing plant Rs. 10 lakh
purchased on Chaitra 2074.
b) The firm didn't deduct tax on salary Rs. 40,000 at the time of payment but it was
paid by the firm during Ashadh end. The deposited amount is debited to Salary
and Wages.
c) The firm was awarded a motorcycle with market value of Rs. 220,000 as it met
Sales target. The motorcycle was immediately sold to one of the employees at the
market value. The transaction has not been reflected in the income statement
above.
d) The details of depreciable assets are as follows:
Block Opening Depreciation Base (Rs.)
A 11,123,750
B 556,190
C 2,500,000
D 4,449,500
e) The sole assets in block C is a delivery van which is 100% financed by AB Bank
Ltd. The delivery van is registered in the name of the bank. The related interest
expense of Rs. 250,000 has not been paid by the firm till the end of the year and is
not considered as expenses in preparing the income statement above.
f) 20% of repairs & maintenance is related to Block B and remaining is related to
Block D.
g) The company has a policy to provide commission to persons referring customers.
The company paid Rs. 40,000 this year as referral commission to persons for
referring customers in the previous year and this has been reduced from Sales
above. Similarly, the company paid Rs. 30,000 after closure of the financial year

© The Institute of Chartered Accountants of Nepal 2


CAP II Paper 7: Income Tax & VAT

as referral commission to persons for referring customers in IY 2074/75 which is


not shown as expenses in the income statement above.
h) Lawyer‟s fee is paid to a lawyer for defending assessment of the firm
made by income tax officer in relation to IY 2073/74. The tax officer
issued an assessment order demanding additional income tax Rs.
200,000. The related liability is not provided by the firm in the above
income statement.
i) Office expenses include monthly tuition fees of children of marketing
director of Rs. 12,000 as per terms of employment.
Required: 20
i) Compute tax payable by the firm for IY 2074/75.
ii) Would the tax liability change if the firm had engaged more than 300
Nepalese employees throughout the year?
Answer:
i) Computation of tax payable by Juntara Trades (Partnership Firm) for IY 2074/75.

Particulars Section Note Detail Amount


Items to be included in
Income

Sales 7(2)(Kha) 1 55,234,100.00

Business related gift 7(2)(Nga) 220,000.00 55,454,100.00


Less: Deductible Expenses

Cost of Goods Sold 15 2 44,155,280.00

Salary and Wages 13 3 3,233,646.00


Business Promotion
Expenses 13 500,000.00

Depreciation Expenses 19 4 1,967,660.00

Interest expenses 14(1) 250,000.00


Repair and Maintenance
Expenses 16 5 338,933.30

Office Expenses 13 6 38,000.00

Lawyers Fee 13 7 45,000.00

Marketing Expenses 13 441,550.00

Referral Commission 13 8 30,000.00 51,000,069.30

Taxable Income 4,454,030.70


Schedule
1 Section
Income Tax Rate @ 25% 2(1) 1,113,507.68

© The Institute of Chartered Accountants of Nepal 3


CAP II Paper 7: Income Tax & VAT

2. No, tax liability shall not be changed if firm employees 300 Nepalese Employee throughout the
year because provision mentioned in Section 11(3) of Income Tax Act, 2058 shall only be
applicable if it is a Special Industry or Information Technology Industry.

Note : 1
Computation of amount to be included in Sales

Particulars Amount
Given 55,194,100.00
Add: Prior period Referral commission
not allowed to be deducted 40,000.00
Sales 55,234,100.00

Note : 2
Computation of Cost of Goods Sold U/S
15

Particulars Amount
Opening Stock 3,219,660.00
Add: Material Purchase 46,455,030.00
Less: Closing Stock 5,519,410.00
Cost of Goods Sold 44,155,280.00

Note : 3
Computation of Salary and Wages U/S
13

Particulars Amount
Given 3,311,646.00
Less: Salary paid for installation of plant
to be capitalized 50,000.00
Less: Tax paid on salary under this act as
the amount should be treated as
receivable from employee 40,000.00
Add: Tuition fee of children paid as per
employment term 12,000.00
Salary & Wages 3,233,646.00

Note : 4
Computation of depreciation U/S 19

Particulars Block A Block B Block C Block D Total

Opening Depreciation Base 11,123,750 556,190 2,500,000 4,449,500


Add: Absorbed Addition
Upto Poush end

© The Institute of Chartered Accountants of Nepal 4


CAP II Paper 7: Income Tax & VAT

220,000
Upto Chaitra end (2/3 of Cost)
700,000
Upto Ashad end (1/3 of Cost)
Less: Amount derived from disposal
220,000

Depreciation Base 11,123,750 556,190 2,500,000 5,149,500


Depreciation Rate 5% 25% 20% 15%

Depreciation U/S 19 556,187.50 139,047.50 500,000 772,425 1,967,660

Note : 5
Computation of repair and maintenance
U/S 16
Particulars Block B Block D Total
Depreciation Base as per Working Note
4 556,190.00 5,149,500.00

7% of Depreciation Base 38,933.30 360,465.00

Actual 75,000.00 300,000.00

Allowed U/S 16 38,933.30 300,000.00 338,933.30

Note : 6
Computation of Office Expenses allowed
U/S 13

Particulars Amount

Given 50,000.00
Less: Tuition fee to be added in
Employment expenses 12,000.00

Allowed U/S 13 38,000.00

Note :7
Legal expenses is allowed to be deducted
U/S 13

Note : 8
Computation of referral commission
allowed U/S 13
Particulars Amount

Referral commission for the year 30,000.00

Allowed U/S 13 30,000.00

© The Institute of Chartered Accountants of Nepal 5


CAP II Paper 7: Income Tax & VAT

2. Ms. Anjali Amatya, submitted the following details of income for Income
Year 2075/76.
Particulars Amount
1. House Rental income let out to XYZ Ltd. (Net Rent) (Rs.)
342,000
2. Interest Income from Development Bank in Nepal (Net 85,000
Interest)
3. Interest Income from friends (Gross) 35,000
4. Furniture rental income from Mr. Paras (Gross) 850,000
5. Windfall gain withholding tax not deducted (Gross) 100,000
6. Gift received related to investment (Gross) 225,000
7. Income from natural resources (Net) 8,500
8. Gain recovered from investment insurance (Net) 85,000
9. Recovery of bad debt 200,000
10. Income from sale of listed shares (Net) 277,500
Ms. Amatya submitted the following details of expenses:
Particulars Amount (Rs.)
1. Repairing expenses of house property let out to XYZ Ltd. (Rs.) (Rs.)
40,000
2. Interest expenses paid to borrowed funds to give loan to friends 9,000
3. Allowable depreciation for furniture let out to Mr. Paras 35,000
4. Interest expenses for borrowed to pay insurance premium 6,500
5. Life insurance premium paid 19,000
6. Expenses paid related to natural resources 9,000
7. Bad debts recovered was not allowed for deduction earlier 80,000
8. Contribution to approved retirement funds 90,000
Your assistance is required to compute her taxable income and tax liability
for Income Year 2075/76. 10
Answer:

Particulars Section Note Detail Amount


Items to be Included
in Income

House Rental Income 1 -


Interest income from
Development Bank in
Nepal 2 -
Interest income from
Friends 9(2)(Ka) 35,000.00
Furniture rental income
from Mr. Paras 9(2)(Ka) 850,000.00
Windfall Gain 92 FWH
Gift related to
investment 9(2)(Gha) 225,000.00
Income from Natural
resources 9(2)(Ka) 3 10,000.00
Gain recovered from
investment insurance 9(3)(Ka) 4 -

Recovery of bad debts 9(2)(Chha) 5 120,000.00

© The Institute of Chartered Accountants of Nepal 6


CAP II Paper 7: Income Tax & VAT

Income from sale of


listed shares 9(2)(Kha) 6 300,000.00 1,540,000.00
Less: Deductible
Expenses
Repairing expenses of
house 21 7 -
Interest expenses paid
to borrowed fund to
give loan to friends 14(1) 9,000.00
Allowable depreciation
expenses 19 35,000.00
Interest Expenses for
borrowed to pay
insurance premium 21(1)(Ga) 7 -
Expenses paid related
to natural resources 13 9,000.00 53,000.00
Assessable Income
from Investment 1,487,000.00
Less: Contribution to
Approved Retirement
Fund 63 8 90,000.00
Schedule 1
Less: Life Insurance Section
Premium 1(12) 9 19,000.00

Taxable Income 1,378,000.00

Computation of Tax Liability of Ms. Anjali Amatya for the


Income Year 2075-76 as per Schedule 1 Section 1(4) assuming to
be Single Natural Person

Particulars Detail Amount


Step 1 Higher of following
shall be taxed as per Schedule
1 Section 1(1)
a) Taxable Income - Gain on
disposal of Non Business 1,078,000.
Chargeable Assets 00
or
b) Basic Exemption as per
Schedule 1 Section 1(1) 350,000.00

1,078,000.
Higher of Step 1 00
Tax on Higher amount as
per Schedule 1 Section 1(1)

Up to Rs. 350000 -

Next Rs. 100000 @ 10% 10,000.00

Next Rs. 200000 @ 20% 40,000.00


Balance

© The Institute of Chartered Accountants of Nepal 7


CAP II Paper 7: Income Tax & VAT

128,400.00
Tax as per Step 1 178,400.00

Step 2 Balance taxable


income shall be taxed at the
rate of 7.5% 22,500.00
Total Tax Payable (Step
1+Step 2) 200,900.00
Less: Withholding Tax
On gain on disposal of listed
share 22500
Natural resource payment 1500
Net Tax Payable 176,900.00

Note: 1
As per Section 2, “Rent” means any payment made by the lessee under a lease of a tangible asset including
any premium and any other payment for the granting of the lease but excludes a natural resource payment and
house rent income of a natural person otherwise than that derived in relation to operation of private firm.

Note : 2
Interest from bank is subject to withholding tax @ 5% u/s 88(3). Such withholding payment shall be treated
as Final Withholding Payment u/s 92(1)(Nga) therefore shall not be included in Income from Investment as
per Section 9(3)(Ka).

Note: 3
Natural resource payment is subject to withholding tax @ 15% u/s 88(1).

Note: 4
Gain from investment insurance is subject to withholding ta @ 5% u/s 88(2)(Kha). Such withholding
payment shall be treated as final withholding payment u/s 92(1)(Ga) therefore shall not be included in
Income from Investment U/S 9(3)(Ka)

Note: 5
Baddebt recoverd shall be included in income only to the extent of expenses which has been allowed
previously.

Note : 6
Income from sale of listed share is subject to advance tax at the rate of 7.5% U/S 95Ka(2). Therefore gross
amount shall be included in income.

Note: 7
Any expenses which is incurred for earning income that is exempt from tax or final withholding payment
shall not be allowed to be deducted as per Section 21(1)(Ga)

© The Institute of Chartered Accountants of Nepal 8


CAP II Paper 7: Income Tax & VAT

Note: 8
Least of the following shall be allowed as per Section 63
i) Monetary Limit 300,000.00
ii) 1/3 of Assessable Income 495,666.67
iii) Actual Contribution 90,000.00
Least 90,000.00

Note : 9
Least of the following shall be allowed as per Schedule 1 Section 1(12)
i) Monetary Limit 25000
ii) Actual premium 19000
Least 19000

Note : 10
Windfall gain is subject to withholding tax @ 25% U/S 88Ka. Such withholding payment shall be treated as
final withholding payment U/S 92.

3.
a) Big Traders Pvt. Ltd. dealing with electronic items has annual turnover
Rs. 7,890,000 and taxable income Rs. 1,780,000 for IY 2074/75. The
details of tax paid by the company is Rs. 130,000 on Poush end 2074
and filed the estimated tax return on Poush 28 2074, paid Rs. 56,000 on
Chaitra 27, 2074 and a government agency has withheld tax on behalf
of Big Traders during the month of Jestha 2075 amounting Rs. 57,500.
Final installment of tax amounting Rs. 46,500 was deposited during
Ashadh 2075. Examine about the compliance of Section 94 by the
company. 5
b) What is the applicable withholding tax amount on the following: 5
i) Service fees of Rs. 50,000 paid for VAT exempted services.
ii) ABC Limited paid house rent Rs. 100,000 to landlord Mr. Sharma.
iii) MNO Ltd. paid Rs. 50,000 as freight charges to Sulabh Transport
Co.
iv) Bank of Kathmandu paid Rs. 500,000 as interest in fixed deposit
account maintained by Sagarmatha Mutual Funds.
v) ABC Ltd. provides car worth Rs. 2,500,000 to the winner in the
lottery scheme of the Co.
Answer:
a)

Computation of Tax Liability of Big Traders Private Limited for the Income Year
2074-2075
Particulars Section Amount
Taxable income from
Business 7(2) 1,780,000.00
Schedule 1 Section
Income Tax Rate @ 25% 2(1) 445,000.00

© The Institute of Chartered Accountants of Nepal 9


CAP II Paper 7: Income Tax & VAT

Computation of advance tax to be paid in each installment U/S 94


Due Date Cumulative % Installment Due Payment Deficit
Up to Poush 2074 40% 178,000.00 130,000.00 48,000.00
Upto Chaitra 2074 70% 311,500.00 186,000.00 125,500.00
Upto Ashad 2075 100% 445,000.00 290,000.00 155,000.00

As per Sub Section (1) of Section 94, a person who derives or expects to derive any assessable
income during an income-year from a business or investment shall be required to pay tax for the
year by three installments as mentioned above. However, in the given case Big Traders Private
Limited has paid advance tax lower than actual tax to be paid by Rs. 155,000

b)
Withholding Withholding
S.No. Nature of Payment Section Tax Rate Amount Tax Amount
Service fee to VAT
i exempted services 88(1)(4) 1.50% 50,000.00 750.00
House Rent to Natural No
ii Person 88(1)(5) Withholding 100,000.00 -

iii Freight Charges paid 88(1)(8) 2.50% 50,000.00 1,250.00


Interest paid to Mutual No
iv Fund by Bank 88(4)(Nga) Withholding 500,000.00 -

v Payment of lottery 88Ka 25% 2,500,000.00 625,000.00

4.
a) Himali Garments (Pvt.) Ltd. deals with export and local sales of
readymade garments. Based on the following information, determine
the taxable income and tax liability from local sales & export business. 5
Particulars Amount (Rs.)
Total Sales 20 Million
Local Sales 5 Million
Opening Stock of Garments 3 Million
Cost of Production 17 Million
Closing Stock 4 Million
Export Expenses 0.5 Million
Promotional Expenses in Nepal 0.9 Million
Depreciation 1.6 Million

b) During the income year 2074/75, Mr. Anand won DV lottery of


America and he left permanently with his family members on 25th
Poush 2075. Mr. Anand has already filed IT Return for income year
2074/75, however he has not filed IT return for income year 2075/76.
Mr. Anand has earned Rs. 1,500,000 taxable income up to 25th Poush
2075.
Can income tax authority assess the income tax liability of Mr. Anand
immediately? 5

© The Institute of Chartered Accountants of Nepal 10


CAP II Paper 7: Income Tax & VAT

c) Nepal Telecommunications Authority (NTA) hired Mr. Eric, a German


Citizen as Rural Communication Infrastructure Expert with effect from
September 17, 2018. He came to Nepal, 7 days before his joining date
on September 10, 2018. He left Nepal on December 20, 2018 and came
back to Nepal after Christmas leave on January 9, 2019, and continued
his service for the contract period. After the expiry of contract, he
converted his visa category into Tourist Visa, and went to Annapurna
Base Camp for 17 days, and returned to his home country on April 2,
2019. Comment about the residential status of Mr. Eric.
5
d) Mr. Sonu, a sole shareholder of M/s Sonu Industries Pvt. Ltd., was
worried about the performance of the Company as it incurred losses of
Rs. 5 crores during the last 4 financial years ending F/Y 2073/74. Mr.
Rakesh, an expert acquired 60% stake in the Company on Ashadh 31,
2074. The company earned Rs. 1.5 crore as profit in F/Y 2074/75. The
Company has submitted the income tax return by assessing a taxable
loss of Rs. 3.5 crore for F/Y 2074/75 under self-assessment by
adjusting the carry forward losses of Rs. 5 crore upto F/Y 2073/74 u/s
20 of Income Tax Act, 2058. The Chief Tax Officer issued an order to
pay income tax on Rs. 1.5 crore along with interest thereon. The
management of the Company seeks your advice on the said order of
Inland Revenue Office. 5
Answer:
a)
Computation of Taxable income and tax liability of of Himali Garments Private
Limited for the Income Year 2074-2075
Amt in Million
Export Domestic
Particulars Section Note Sales Sales Total
Items to be Included in Income 7(2)(Kha)
Sales 15 5 20
Less: Deductible Expenses
Cost of goods sold 15 2 12 4 16
Export Expenses 13 0.5 0 0.5
Promotional expenses in Nepal 13 0 0.9 0.9
Depreciation 19 3 1.2 0.4 1.6
Profit/Loss 1.3 -0.3 1
Taxable Income 1.3 0
Schedule
1Section
Income Tax Rate @ 20% 1(3Ka) 0.26

© The Institute of Chartered Accountants of Nepal 11


CAP II Paper 7: Income Tax & VAT

Note: 1

As per Sub Section (4) of Section 11, While calculating the income by a person operating
transactions eligible for separate benefit under this section shall calculate the such
income assuming the income derived by a separate person. Therefore combined expenses
shall be allocated in the ratio of sales i.e. 3:1.

Note: 2
Computation of Cost of Goods sold U/S 15

Particulars Amount
Opening Stock 3
Add: Cost of Production 17
Less: Closing Stock 4
Cost of Goods sold 16
Export Sales 12
Domestic Sales 4

Note: 3
Allocation of Depreciation u/s 19

Particulars Amount
Given 1.6
Export Sales 1.2
Domestic Sales 0.4

Note 4:

As per Sub Section (3) of Section 20, Subject to subsection (1) and (2), a person may
deduct an unrelieved loss with a foreign source only in calculating the person's foreign
source income and an unrelieved loss incurred in deriving nontaxable income only in
calculating the person's non-taxable income for the purposes of subsections (1) and (2).

b)
As per Section 100, assessment of tax within the same income year or after
income year but before statutory time limit of filling return is jeopardy assessment.
ii) If any of the following conditions are satisfied, the tax authority may demand
jeopardy assessment:
The person become bankrupt, is wounded-up, or goes into liquidation;
The person is about to leave Nepal indefinitely;
The person is about to leave the business;
The IRD otherwise considered it appropriate.
Mr. Anand is leaving Nepal permanently with his family members. Yes, Income
tax authority can demand jeopardy assessment as per the provision of Income tax
Act 2058 for the taxable income of Rs. 1500,000 earned by Mr. Anand before the
completion of income year or before statutory time limit of filling income tax
return

© The Institute of Chartered Accountants of Nepal 12


CAP II Paper 7: Income Tax & VAT

c)
As per Section 2(Ka Nga)(1)(Kha), natural person is resident for an Income Year if he
resides in Nepal for 183 days or more during consecutive 365 days. Here, consecutive 365
days means an Income Year.

Determination of residential status of Mr. Eric for the income year 2075-76

Number of
Income Year Period of stay Days Residential Status
10.09.2018 to
2075-2076 20.12.2018 102
09.01.2019 to Resident
02.04.2019 84
Total 186

Here, both date of arrival and date of departure shall be included while computing 183
days.
Total

d)
As per Section 57 (1) & (2) of Income Tax Act, 2058, if the ownership of any
entity changes by 50% or more during the last three income years, then there will
be change in control the Company.
And as per Section 57 (2) (Kha) of Income Tax Act, 2058, The Company is not
allowed to carry forward its accumulated losses of the period prior to such transfer
of ownership.
In this case, the ownership of M/s Sonu Industries Pvt. Ltd. was changed by 60% as
the shares of the Company was sold by old management to the new management;
therefore, the Company cannot adjust any accumulated losses for the period until
Ashad 31, 2074. Thus, the assessment order issued by the Inland Revenue Office is
correct & the Company has to pay tax on the profit of the Company earned after the
change in control.
Total

5. Write short notes of the following with reference to Income Tax Act, 2058. (4×2.5=10)
a) Residential status of an individual
b) Advance ruling
c) Deductible expenses from cash payment
d) Tax on agricultural income

Answer:
a)
As per Section 2(KaNga)(1) The residential status of natural person is
determined as below:

© The Institute of Chartered Accountants of Nepal 13


CAP II Paper 7: Income Tax & VAT

i) If an individual stays in Nepal for 183 days or more in a period of


consecutive 365 days.
ii) If normal place of abode is in Nepal.
iii) If the person in an employee of Nepal Government.
Normal place of abode is clarified by Income Tax Directives to mean
place where economic activities of a person is concentrated.
As clarified by Income Tax Directives Period of 365 days shall be taken
within an Income Year.
If the person is not resident in an income year, he is regarded as non-
resident.

b)
According to Section 76 of Income Tax Act 2058, Advance Ruling is
issued to the applicant tax payer for clarifying application of tax law on
proposed arrangement of the applicant.
The advance ruling is only applicable for the concerned applicant and is
not available for general public.

c)
According to Section 21, of Income Tax Act, 2058 any expenditure paid
through cash for more than Rs 50,000 by a taxpayer having annual
turnover for more than Rs. 20 lakh cannot be deductible. However, the
following payments are deductible even though, they exceed the cash
payment limit of Rs. 50,000.
i) Payment to GON, Constitutional bodies, public enterprises and BFI,
ii) Payment to producer of primary agricultural produces even when the
products are primarily processed,
iii) Payment of retirement contribution or retirement payment
iv) Payment made on the day of closure of banking service or on
circumstances to be paid only in cash, or
v) Payment made in such area where banking facility is not available
vi) Amount deposited into the bank account of the receiver.

d)
As per Clarification of Section 11, Agriculture business means the business of
producing crops from public or private land, or deriving rent from a tenant
using land.
Agricultural income derived by a natural person from farming within the
prescribed limit under Land Related Act, 2021 is exempt from tax. But the same
income is taxed if earned by a partnership firm or Company.

© The Institute of Chartered Accountants of Nepal 14


CAP II Paper 7: Income Tax & VAT

6. ABC Industry Pvt. Ltd. had the following transactions in Baishakh 2076.
Calculate the VAT payable/receivable from the information below. 10
Particulars Amount (Rs.)
Sales:
Local 6,000,000
Export 12,000,000
Purchases:
Clothes, Stitching, Packing Materials, loose tools for machineries 7,200,000
Special packing for export 300,000
Consultancy charges abroad 600,000
Bus for staff transportation 2,200,000
Motorcycle hire purchase 500,000
Telephone expenses 96,000
Diesel for generator 70,000
Diesel for bus 34,000
Petrol for motorcycle 20,000
Computers 100,000
Soft drinks 22,000
Additional information:
Opening VAT receivable for the month was Rs. 91,560. Diesel for bus for Rs. 22,000
and soft drinks for Rs. 8,000 was purchased through abbreviated tax invoice. Items
above are exclusive of VAT.

Answer:
Amount VAT
Particulars Remarks
(Rs.) (Rs.)
Output VAT
Local Sales 6,000,000.00 780,000.00 Full
0% for
Export Sales -
12,000,000.00 Export
Total Output VAT (A) 780,000.00
Less: Input VAT
Purchase of clothes, Stitching,
Packing Materials, loose tools for 936,000.00 Full
7,200,000.00
machineries
Special packing for export 39,000.00 Full
300,000.00
Reverse
Charging,
Payment of consultancy charges
assuming
abroad 600,000.00 78,000.00
VAT already
paid
Purchase of bus for staff Only 40%
transportation 2,200,000.00 114,400.00 Allowed
Purchase of motorcycle hire
Full
purchase 500,000.00 65,000.00
Telephone expenses 96,000.00 12,480.00 Full
Purchase of diesel for generator 70,000.00 9,100.00 Full

© The Institute of Chartered Accountants of Nepal 15


CAP II Paper 7: Income Tax & VAT

Abbreviat
ed Tax
Purchase of diesel for bus 1,560.00
34,000.00 Invoice not
allowed
Not
Purchase of petrol for motorcycle -
20,000.00 allowed
Purchase of computers 100,000.00 13,000.00 Full
VAT on
Purchase of soft drinks - Beverages
22,000.00
not allowed
Total Input tax credit (B) 1,268,540.00
Opening VAT Receivable (C) 91,560.00
Excess Input VAT (A-B-C) to
580,100.00
be carried forward
Total

7.
a) Maharjan Pvt. Ltd. working in Palpa has the following amounts of Sales and
Purchases excluding VAT. Can VAT refund be claimed in Baishakh return? 5

Amount
(Rs.)
Month Sales Purchase
Kartik 500,000 600,000
Mangsir 520,000 500,000
Poush 520,000 500,000
Magh 450,000 500,000
Falgun 320,000 500,000
Chaitra 400,000 500,000
Baishakh 350,000 500,000

b) ABC Ltd. provides you the following transaction during Ashwin to


Mangsir, 2075:

Amount (Rs.)
Particulars Ashwin Kartik Mangsir
Sales:
Taxable Sales 100,000 200,000 300,000
Zero Rated Sales 300,000 100,000 200,000
Exempted Sales 200,000 300,000 100,000
Total Sales (A) 600,000 600,000 600,000
VAT paid/payable on purchases related to :
Taxable Sales 13,000 26,000 39,000
Zero Rated Sales 12,000 3000 6,000
Exempted Sales 12,000 24000 6,000
Total VAT paid (B) 37,000 53,000 51,000
VAT paid/payable on expenses:
Overhead expenses 12,000 12,000 12,000

© The Institute of Chartered Accountants of Nepal 16


CAP II Paper 7: Income Tax & VAT

Motor Car 26,000 0 0


No credit (goods/services) 8,000 8,000 8,000
Total VAT paid (C) 46,000 20,000 20,000
Required: 5
Compute the amount of eligible VAT credit available to the company for each
month?
Assume there was opening VAT credit of Rs. 1,500 at the beginning of Ashwin,
2075.

Answer:
a)

As per Section 24(3) of VAT Act, 2052; a registered person may file a claim
for a lump sum refund, as prescribed, of the remaining excess amount after
offsetting for a continuous period of six months.
Cumulative
VAT VAT
Month Sales Purchase VAT Payable/
Payable Receivable
(Receivable)
Kartik
500,000.00 65,000.00 600,000.00 78,000.00 (13,000.00)
Mangsir
520,000.00 67,600.00 500,000.00 65,000.00 (10,400.00)
Poush
520,000.00 67,600.00 500,000.00 65,000.00 (7,800.00)
Magh
450,000.00 58,500.00 500,000.00 65,000.00 (14,300.00)
Falgun
320,000.00 41,600.00 500,000.00 65,000.00 (37,700.00)
Chaitra
400,000.00 52,000.00 500,000.00 65,000.00 (50,700.00)
Baisakh
350,000.00 45,500.00 500,000.00 65,000.00 (70,200.00)
For Baisakh tax return, there is continuous credit of Rs. 7,800 for last six
months. Company can claim a refund of Rs. 7,800 in Baishakh.

b) Computation of eligible VAT credit available to ABC Ltd for each month:

Particulars Aswin Kartik Mangsir

Percentage of Taxable and


66.67 50.00 83.33
Exempted Sales
Input tax credit (VAT
receivables):
Opening VAT credit 1,500 28,433 37,433
Input tax credit related to
13,000 26,000 39,000
taxable sales
Input tax credit related to Zero 12,000 3000 6,000

© The Institute of Chartered Accountants of Nepal 17


CAP II Paper 7: Income Tax & VAT

Rated Sales
Input tax credit related to
0 0 0
Exempted Sales
Input tax credit on Overhead
8,000 6,000 10,000
Exp. (Note -2)
Input tax credit on Motor
6,933 0 0
Vehicle (W.N.-3)
Input tax credit on No credit
0 0 0
(goods/Services)
Total eligible VAT credit
41,433 63,433 92,433
available (A)
Output Tax (VAT Payable):
Total Taxable Sales 13,000 26,000 39,000
Zero Rate Sales 0 0 0
Exempted Sales 0 0 0
Total output VAT payable
13,000 26,000 39,000
(B)
Net VAT credit
Receivable/(Payable) to be
28,433 37,433 53,433
carried forward to next
month A-B

Note 1:

Computation of available proportionate VAT credit percentage:

Particulars Aswin Kartik Mangsir


Taxable Sales (Taxable + Zero Rated Sales (a) 400,000 300,000 500,000
Exempted Sales 200,000 300,000 100,000
Total Sales (b) 600,000 600,000 600,000
% of Taxable Sales (a/b*100) 66.67 50.00 83.33

Note 2:
Computation of VAT credit allowed on overhead expenses
Particulars Aswin Kartik Mangsir
Overhead Expenses 12,000 12,000 12,000
% of Taxable Sales as per Note 1 66.67 50.00 83.33
Credit Allowed 8,000 6,000 10,000

Note 3:
Computation of VAT credit allowed on Motor Car
Particulars Aswin
Motor Car 26,000
Credit Allowed @ 40% 10,400
% of Taxable Sales as per Note 1 66.67
Credit Allowed 6,933

© The Institute of Chartered Accountants of Nepal 18


CAP II Paper 7: Income Tax & VAT

8. Write short notes on the basis of VAT Act, 2052: (4×2.5=10)


a) Reverse charge
b) Conditions for zero rate of tax
c) Collection of tax from other than registered person
d) Tax on transfer of business
Answer:
a)
Generally, VAT is collected and paid by the seller of goods. However, in some
cases, Act has cast responsibility upon the purchaser of goods to pay VAT. This
responsibility of paying VAT on purchaser is termed as „Reverse VAT‟.
Section 8(2) and 8(3) of Value Added Tax Act, 2052 has provided as follows:
i) VAT for Service Received from Foreign Party:
As per Section 8(2) of Value Added Tax Act, 2052, any person whether
registered or not in Nepal receiving services from a person who is not registered
and is outside Nepal shall have to assess and collect tax at the taxable value at
the time of payment in accordance with this Act and the Rules framed under
this Act.
ii) VAT from Housing Developers:
Section 8(3) of Value Added Tax Act, 2052, mention that If a construction of
commercial purpose building or apartment or shopping complex or similar
other structure as specified by the Department, of which value is more than
Five Million Rupees, has been made from a person who is not registered, it
shall be deemed as it has been constructed from registered person and shall
deposit the tax. In case tax is not deposited, it shall be assessed and collected
from the owner of such structure.

b)

Zero Rate Tax means VAT is applicable at zero rates. In these cases, input tax
credit is available to the person. Schedule 2 of the Value Added Tax Act, 2052
has provided zero rated VAT items as follows:

i) Export of Goods
ii) Export of Services
iii) Goods and Services imported by a person or body corporate availing
diplomatic facility or customs facility as per recommendation of Government of
Nepal
iv) If Sales Tax was earlier exempt as per treaty or agreement
v) Sales made to industries established in Special Economic Zone
vi) Battery for Solar Power on recommendation of Alternate Power Promotion
Centre
vii) Equipments for hydro power on recommendation of specified body
viii) If paintings, handicrafts, carving and similar other handicrafts produced a
cottage and small scale industry within Nepal are exported
ix) Import or local purchase of scooters used by persons with disabilities shall,
if such scooters are registered in their name in the Office of Transport
Management

© The Institute of Chartered Accountants of Nepal 19


CAP II Paper 7: Income Tax & VAT

c)
In general, VAT paid on purchase is qualified for credit if it is supported by tax
invoice in prescribed format. Here are few exemptions where tax paid on
purchase is eligible for credit without tax invoice. They are:
VAT paid against reverse charging
Taxable items purchased from GoN
Import supported by Pragyapan Patra
Wood purchased from community forest.

d)
As per Sub Section (1) of Section 5Ka, Notwithstanding anything elsewhere in
Act, no tax is levied in the transfer of business of Registered Person by sale to
another Registered Person or transferred ownership of the business to heir after
death. Such transfer or sale of business to be informed to the Department as
Prescribed.
As per Sub Section (2) of Section 5Ka, Notwithstanding anything contained in
Sub-sec.(1), tax obligation of so transferred industry of business, either
registered or requires to be registered, shall be borne by the transferee.

© The Institute of Chartered Accountants of Nepal 20


CAP II Paper 7: Income Tax & VAT

Specific Comments on the performance of the students


Batch: - June 2019
Level: - CAP-II
Subject: Income Tax & VAT
Question No. 1
Majority of students attempted the question, however very few students get half of the
marks allotted for the question. Majority of students secured marks in between 5 to 8.
Students need to understand the concepts of deductions relating to provision for CSR,
R&M expenses etc. Only few students were able to answer near correctly. Most
students even not able to compute depreciation correctly by applying correct rate.
Students are also unable to understand that the entity is special industry inadequate.
Question No. 2
Majority of students attempted the question and secured average marks and very less
students secured 5 to 7 marks. Not well prepared by students to compute TPS of
employee as given in the question. Students need to understand that for TDS annual
income shall compute first.
Question No. 3
Majority of students do not attempted the question with disclosing the provision of IT
Act, general attempt by students and secured very less marks.
Question No. 4
Majority of students attempted the question but unable to secure good marks. Only
very less student secured average marks. Majority of students secured very less
marks. Students have Lack of understanding of knowledge about residential status,
withholding tax and recent amendment. Students were more confused in the concepts
of Residential status of company and its taxability.
Question No. 5
Majority of students attempted the question but unable to answer the short notes in
line with the provision of IT Act and secured average marks. Students need to
elaborate in theory questions.
Question No. 6
Majority of students try to attempt the question one/two students try to solve the
question but majority students confused to answer the question and secured very less
marks. Students showed poor performance due to lack of knowledge about
proportionate VAT credit. Lack of knowledge of proportionate credit was verdict.
Question No. 7
Majority of students attempted the question and unable to answer as per the spirit of
question and provision of VAT Act. They secured average marks based on step
markings. Students attempt is good but need to response on specific matter asked by
question.
Question No. 8
Majority of students attempted the question and very less students able to secure good
marks. Majority of students gives general answer that is without disclosing the
provision of Act. Students need to elaborate in theory questions.

© The Institute of Chartered Accountants of Nepal 21

You might also like